Vous êtes sur la page 1sur 144

The Official SAT Online Course

Help | Profile | My Organizer | My Bookmarks | Logout

1/1

Answers and Explanations

Test Sections

Back to Score Report

Section 1

Essay
Online - Practice Test #5

Section 2

Section 3

Section 4

Section 5

Section 6

These sample essays were originally handwritten by students but are shown typed here for ease of reading. The essays are displayed exactly as students wrote them, without any corrections to spelling, punctuation, or syntax. One handwritten sample essay is provided to illustrate the need for legible and clear handwriting.

Section 8

Section 9

Exemplars:

Section 10

Essay Prompt

Think carefully about the information presented in the following excerpt and the assignment below.

eg

Copyright 2006 The College Board. All rights reserved.

nR

is

te r

Do closed doors make us creative? Plan and write an essay in which you develop your point of view on this issue. Support your position with reasoning and examples taken from your reading, studies, experience, or observations.

ed

Privacy Policy

Many people believe that closed doors make us creative. These people argue that obstacles and restrictions are necessary, for without them we would never be forced to come up with new solutions. But closed doors, either in the form of specific obstacles or a lack of opportunities, often prevent people from reaching their full creative potential.

Back to Score Report

Terms of Use

Contact Us

file://E:\\e2.htm

2006-11-12

The Official SAT Online Course

1/23

Help | Profile | My Organizer | My Bookmarks | Logout

Answers and Explanations

Test Sections

Back to Score Report

Section 1

View Answers and Explanations


Online - Practice Test #5

Section 2

Section 3

Section 4

The garden that had remained ------- for months was now pleasantly enlivened by the budding shoots of its perennial flowers.
(A)

Section 5

redolent

Section 6

Section 8

(B)

dormant

Section 9

(C) exuberant

Section 10

(D) compliant

(E)

trenchant

nR

(A)

Explanation for Incorrect Answer A : Choice (A) is incorrect. Redolent means aromatic or fragrant. If one were to insert this term into the text, the sentence would read The garden that had remained redolent for months was now pleasantly enlivened by the budding shoots of its perennial flowers. A garden in full bloom can be redolent, but the phrase now pleasantly enlivened suggests that the garden was inactive before the flowers began to grow. An inactive garden is most likely not fragrant.

Explanation for Incorrect Answer C : Choice (C) is incorrect. Exuberant means plentiful or producing in abundance. If one were to insert this term into the text, the sentence would read The garden that had remained exuberant for months was now pleasantly enlivened by the budding shoots of its perennial flowers. The phrase now pleasantly enlivened suggests that the garden was inactive before the flowers began to grow. An inactive garden would most likely not be described as exuberant, or plentiful.

Explanation for Incorrect Answer D : Choice (D) is incorrect. Compliant means willing to conform. If one were to insert this term into the text, the sentence would read The garden that had remained compliant for months was now pleasantly enlivened by the budding shoots of its perennial flowers. It is illogical to suggest that a garden could be compliant, or willing to conform.

Explanation for Incorrect Answer E : Choice (E) is incorrect. Trenchant means keen or sharply perceptive. If one were to insert this term into the text, the sentence would read The garden that had remained trenchant for months was now pleasantly enlivened by the budding shoots of its perennial flowers. It is illogical to suggest that a garden could be trenchant, or sharply perceptive.

After several months of training, the ------- young spaniel was finally ------- enough to be walked safely without a leash.

eager . . unruly

eg

is t

er

Explanation for Correct Answer B : Choice (B) is correct. Dormant means inactive or not growing. If one were to insert this term into the text, the sentence would read The garden that had remained dormant for months was now pleasantly enlivened by the budding shoots of its perennial flowers. A dormant, or inactive, garden would become active as its perennial flowers began to grow.

ed

ANSWERS

AND EXPLANATIONS

file://E:\\e3.htm

2006-11-12

The Official SAT Online Course

2/23

(B)

placid . . defiant

(C) clever . . helpful

(D) boisterous . . docile

(E)

vigilant . . convinced

ANSWERS

AND EXPLANATIONS

Explanation for Correct Answer D : Choice (D) is correct. Boisterous means rowdy and undisciplined. Docile means easily managed. If one were to insert these terms into the text, the sentence would read After several months of training, the boisterous young spaniel was finally docile enough to be walked safely without a leash. The word finally indicates that after several months of training, the spaniels behavior changed. It is logical to suggest that a boisterous dog could become docile after several months of training. A dog that is docile, or easily managed, could likely be walked safely without a leash.

Explanation for Incorrect Answer C : Choice (C) is incorrect. Clever means smart and quick-witted. Helpful means providing assistance. If one were to insert these terms into the text, the sentence would read After several months of training, the clever young spaniel was finally helpful enough to be walked safely without a leash. Although the first term fits within the context of the sentence, the second term does not. A dog can certainly be clever, but it does not make logical sense to say that a dog is providing assistance by walking without a leash.

nR

(A)

Explanation for Incorrect Answer E : Choice (E) is incorrect. Vigilant means watchful and alert. Convinced means certain or persuaded. If one were to insert these terms into the text, the sentence would read After several months of training, the vigilant young spaniel was finally convinced enough to be walked safely without a leash. Although a dog can be vigilant, it does not make sense in this context to say that the dog was convinced enough to be walked without a leash. It is not clear what the dog has been convinced of or why convincing was necessary in order to walk the dog without a leashor, indeed, that dogs can be convinced, a term usually applied to humans.

------- as Marios misdeed was, his grandmother, always blind to his faults, pretended to be unaware of it.

Accidental

(B)

Apt

(C) Random

(D) Flagrant

(E)

Covert

eg is

Explanation for Incorrect Answer B : Choice (B) is incorrect. Placid means calm and undisturbed. Defiant means bold and disobedient. If one were to insert these terms into the text, the sentence would read After several months of training, the placid young spaniel was finally defiant enough to be walked safely without a leash. Although a dog can certainly be placid, or calm, it is unlikely that a defiant dog could be walked safely without a leash. Furthermore, one would not expect training to cause a dog to be defiant, or disobedient.

te

re

Explanation for Incorrect Answer A : Choice (A) is incorrect. Eager means enthusiastic or impatient. Unruly means undisciplined and willful. If one were to insert these terms into the text, the sentence would read After several months of training, the eager young spaniel was finally unruly enough to be walked safely without a leash. Although a dog can certainly be eager, it is unlikely that an unruly dog could be walked safely without a leash. Furthermore, one would not expect training to cause a dog to be unruly, or undisciplined.

file://E:\\e3.htm

2006-11-12

The Official SAT Online Course

3/23

ANSWERS

AND EXPLANATIONS

Explanation for Correct Answer D : Choice (D) is correct. Flagrant means conspicuously or obviously offensive. If one were to insert this term into the text, the sentence would read Flagrant as Marios misdeed was, his grandmother, always blind to his faults, pretended to be unaware of it. The structure of the sentence indicates that Marios grandmother was certainly aware of Marios misdeed, even though she pretended to be unaware. The missing term should describe a misdeed that is very obvious. The term flagrant accurately describes an action that is obviously offensive.

Explanation for Incorrect Answer A : Choice (A) is incorrect. Accidental means occurring unexpectedly or by chance. If one were to insert this term into the text, the sentence would read Accidental as Marios misdeed was, his grandmother, always blind to his faults, pretended to be unaware of it. The structure of the sentence indicates that Marios grandmother was certainly aware of Marios misdeed, even though she pretended to be unaware. The missing term should describe a misdeed that is very obvious. An action that is accidental would not necessarily be obvious.

nR eg

(A)

Explanation for Incorrect Answer C : Choice (C) is incorrect. Random means lacking a definite pattern or plan. If one were to insert this term into the text, the sentence would read Random as Marios misdeed was, his grandmother, always blind to his faults, pretended to be unaware of it. The structure of the sentence indicates that Marios grandmother was certainly aware of Marios misdeed, even though she pretended to be unaware. The missing term should describe a misdeed that is very obvious. An action that is random, or does not follow a pattern or plan, would not necessarily be obvious.

Explanation for Incorrect Answer E : Choice (E) is incorrect. Covert means secret or not openly shown. If one were to insert this term into the text, the sentence would read Covert as Marios misdeed was, his grandmother, always blind to his faults, pretended to be unaware of it. The structure of the sentence indicates that Marios grandmother was certainly aware of Marios misdeed, even though she pretended to be unaware. The missing term should describe a misdeed that is very obvious. A covert, or secret, action is the opposite of an obvious action. Marios grandmother would most likely not be aware of a covert misdeed.

Despite his ------- desire to show off, he remained at heart a very ------- person.

uncharacteristic. . demonstrative

(B)

inexplicable. . hedonistic

(C) occasional. . reticent

(D) continual. . transparent

(E)

blatant. . exhibitionistic

ANSWERS

AND EXPLANATIONS

Explanation for Correct Answer C : Choice (C) is correct. Occasional means happening only sometimes. Reticent means quiet and reserved. If one were to insert these terms into the text, the sentence would read Despite his occasional desire to show off, he remained at heart a very reticent person. The word Despite indicates that the missing terms will describe contrasting behaviors. Someone who is reticent, or quiet and reserved, would not frequently show off but might do so from time to time.

is t

er

ed

Explanation for Incorrect Answer B : Choice (B) is incorrect. Apt means appropriate or suitable. If one were to insert this term into the text, the sentence would read Apt as Marios misdeed was, his grandmother, always blind to his faults, pretended to be unaware of it. A misdeed is inappropriate behavior and would not likely be described as apt.

file://E:\\e3.htm

2006-11-12

The Official SAT Online Course

4/23

Explanation for Incorrect Answer A : Choice (A) is incorrect. Uncharacteristic means not typical. Demonstrative means displaying feelings openly. If one were to insert these terms into the text, the sentence would read Despite his uncharacteristic desire to show off, he remained at heart a very demonstrative person. The word Despite indicates that the missing terms will describe contrasting behaviors. There is no inherent contrast between the terms uncharacteristic and demonstrative, because showing off is not necessarily uncharacteristic behavior for someone who displays his feelings openly.

Explanation for Incorrect Answer B : Choice (B) is incorrect. Inexplicable means incapable of being explained. Hedonistic means concerned primarily with happiness and pleasure. If one were to insert these terms into the text, the sentence would read Despite his inexplicable desire to show off, he remained at heart a very hedonistic person. The word Despite indicates that the missing terms will describe contrasting behaviors. Even if the desire to show off is inexplicable, or incapable of being explained, there is no inherent contrast between having a desire to show off and being hedonistic. Showing off is not necessarily related to being primarily concerned with happiness and pleasure.

nR

(A)

The employer blamed the staff member's lack of productivity on ------- rather than incompetence, claiming that the man knew how to do his job but was too lazy to apply himself.

infatuation

(B)

tension

(C) indigence

(D) indolence

(E)

ineptitude

ANSWERS

Explanation for Correct Answer D : Choice (D) is correct. Indolence is a tendency to be lazy. If one were to insert this term into the text, the sentence would read The employer blamed the staff members lack of productivity on indolence rather than incompetence, claiming that the man knew how to do his job but was too lazy to apply himself. The employers claim that the staff member was competent but too lazy to apply himself indicates that the staff members lack of productivity was the result of laziness. The term indolence precisely describes the laziness that the employer claimed was responsible for the staff members lack of productivity.

Explanation for Incorrect Answer A : Choice (A) is incorrect. Infatuation is a foolish or extravagant love or admiration for something. If one were to insert this term into the text, the sentence would read The employer blamed the staff members lack of productivity on

eg

AND EXPLANATIONS

is te

Explanation for Incorrect Answer E : Choice (E) is incorrect. Blatant means completely obvious. Exhibitionistic means acting in a way that will attract attention. If one were to insert these terms into the text, the sentence would read Despite his blatant desire to show off, he remained at heart a very exhibitionistic person. The word Despite indicates that the missing terms will describe contrasting behaviors. There is no contrast between having a blatant desire to show off and being an exhibitionistic person. On the contrary, someone who wants to attract attention would probably be likely to show off.

re d

Explanation for Incorrect Answer D : Choice (D) is incorrect. Continual means going on without interruption. Transparent means obvious and free from pretense. If one were to insert these terms into the text, the sentence would read Despite his continual desire to show off, he remained at heart a very transparent person. The word Despite indicates that the missing terms will describe contrasting behaviors. There is no inherent contrast between having a continual desire to show off and being a transparent person, and a person could be obvious in his desire to show off.

file://E:\\e3.htm

2006-11-12

The Official SAT Online Course

5/23

infatuation rather than incompetence, claiming that the man knew how to do his job but was too lazy to apply himself. The employers claim that the staff member was competent but too lazy to apply himself indicates that the staff members lack of productivity was the result of laziness. The missing term should describe this laziness. Infatuation, or a foolish love, does not describe laziness.

Explanation for Incorrect Answer B : Choice (B) is incorrect. Tension is stress. If one were to insert this term into the text, the sentence would read The employer blamed the staff members lack of productivity on tension rather than incompetence, claiming that the man knew how to do his job but was too lazy to apply himself. The employers claim that the staff member was competent but too lazy to apply himself indicates that the staff members lack of productivity was the result of laziness. The missing term should describe this laziness. Tension, or stress, does not describe laziness.

Explanation for Incorrect Answer C : Choice (C) is incorrect. Indigence is extreme poverty. If one were to insert this term into the text, the sentence would read The employer blamed the staff members lack of productivity on indigence rather than incompetence, claiming that the man knew how to do his job but was too lazy to apply himself. The employers claim that the staff member was competent but too lazy to apply himself indicates that the staff members lack of productivity was the result of laziness. The missing term should describe this laziness. Indigence, or extreme poverty, does not describe laziness.

(A)

pedantry. . chance

(B)

gallantry. . whimsy

nR

(E)

(C) humility. . fortune

(D) bravado. . accident

effrontery. . discretion

ANSWERS

Explanation for Correct Answer C : Choice (C) is correct. Humility is the quality of being humble and not overly proud. Fortune means luck or chance. If one were to insert these terms into the text, the sentence would read The audience recognized the officers characteristic humility when he attributed his achievements to fortune rather than bravery. The sentence structure indicates that the first missing term will describe the behavior demonstrated in the officers discussion of his achievements. Attributing achievements to fortune, or luck, instead of bravery is an example of humility.

Explanation for Incorrect Answer A : Choice (A) is incorrect. Pedantry means to pay extreme attention to rules. Chance means luck or an unpredictable advantage. If one were to insert these terms into the text, the sentence would read The audience recognized the officers characteristic pedantry when he attributed his achievements to chance rather than bravery. The sentence structure indicates that the first missing term will describe the behavior demonstrated in the officers discussion of his achievements. Attributing achievements to chance, or luck, is not an example of pedantry, or extreme attention to rules.

Explanation for Incorrect Answer B :

eg

The audience recognized the officers characteristic ------- when he attributed his achievements to ------- rather than bravery.

AND EXPLANATIONS

is te r

Explanation for Incorrect Answer E : Choice (E) is incorrect. Ineptitude is incompetence and a lack of sense. If one were to insert this term into the text, the sentence would read The employer blamed the staff members lack of productivity on ineptitude rather than incompetence, claiming that the man knew how to do his job but was too lazy to apply himself. The term ineptitude is not logical in this context. Ineptitude means incompetence. The employer did not blame the staff members lack of productivity on incompetence but, rather, claimed that the staff member knew how to do his job.

ed

file://E:\\e3.htm

2006-11-12

The Official SAT Online Course

6/23

Choice (B) is incorrect. Gallantry is very noble and brave behavior. Whimsy is a whim or a sudden impulse. If one were to insert these terms into the text, the sentence would read The audience recognized the officers characteristic gallantry when he attributed his achievements to whimsy rather than bravery. The sentence structure indicates that the first missing term will describe the behavior demonstrated by the officers discussion of his achievements. There is not necessarily anything noble or brave in the officers claim that his achievements are due to whimsy, or sudden impulse.

Explanation for Incorrect Answer D : Choice (D) is incorrect. Bravado means pretending to be brave. Accident is an unplanned and unexpected event. If one were to insert these terms into the text, the sentence would read The audience recognized the officers characteristic bravado when he attributed his achievements to accident rather than bravery. Attributing his achievements to accident instead of bravery is not an example of the officer pretending to be brave. Someone demonstrating bravado would probably attribute his achievements to bravery.

Explanation for Incorrect Answer E : Choice (E) is incorrect. Effrontery means offensive boldness. Discretion is the ability to make responsible decisions. If one were to insert these terms into the text, the sentence would read The audience recognized the officers characteristic effrontery when he attributed his achievements to discretion rather than bravery. There is no reason to believe that the audience would consider the officer to be offensively bold when attributing his achievements to discretion, or responsible choices.

The strong ------- the professor was able to establish with his students made him ------- confidant for those on campus seeking advice beyond the purely academic.

(B)

rapport. . a respected

(D) community. . an unusual

(E)

ambition. . a valued

ANSWERS

nR

Explanation for Correct Answer B : Choice (B) is correct. Rapport means a relationship of trust. Respected means thought highly of and appreciated. If one were to insert these terms into the text, the sentence would read The strong rapport the professor was able to establish with his students made him a respected confidant for those on campus seeking advice beyond the purely academic. A confidant is someone one trusts with secrets or private matters. If the professor established a rapport, or relationship of trust, with his students, it makes sense that the students would respect his advice on personal matters.

Explanation for Incorrect Answer A : Choice (A) is incorrect. Program means an academic plan or curriculum. Occasional means happening only sometimes. If one were to insert these terms into the text, the sentence would read The strong program the professor was able to establish with his students made him an occasional confidant for those on campus seeking advice beyond the purely academic. A confidant is someone one trusts with secrets or private matters. While the professor could establish a strong program with the help of his students, there is not necessarily any reason to believe that the professors program would cause his students to seek him out to discuss personal matters.

Explanation for Incorrect Answer C : Choice (C) is incorrect. Confidence is trust in a person or thing. Unappreciated means not recognized or not valued. If one were to insert these terms into the text, the sentence would read The strong confidence the professor was able to establish with his students made him an unappreciated confidant for those on campus seeking advice beyond the purely academic. A confidant is someone one trusts with secrets or private matters. If the professor was able to establish confidence, or trust, with his students, it is reasonable to suggest that they would appreciate his advice on personal matters.

eg

AND EXPLANATIONS

is t

(C) confidence. . an unappreciated

er

(A)

program. . an occasional

ed

file://E:\\e3.htm

2006-11-12

The Official SAT Online Course

7/23

Explanation for Incorrect Answer D : Choice (D) is incorrect. Community means a group of people with a connection or something in common. Unusual means not ordinary. If one were to insert these terms into the text, the sentence would read The strong community the professor was able to establish with his students made him an unusual confidant for those on campus seeking advice beyond the purely academic. A confidant is someone one trusts with secrets or private matters. If the professor had established a strong community, it would not be unusual for his students to seek his advice on personal matters.

Explanation for Incorrect Answer E : Choice (E) is incorrect. Ambition is the desire to achieve a specific thing. Valued means appreciated and regarded highly. If one were to insert these terms into the text, the sentence would read The strong ambition the professor was able to establish with his students made him a valued confidant for those on campus seeking advice beyond the purely academic. While the second term fits the meaning of the sentence, the first term does not. A professor could be a valued confidant, or person trusted with private matters, but it does not make logical sense to say that the professor established ambition with his students.

After David left him waiting for the third consecutive time, Kirk realized that the same behavior he had initially valued as spontaneous and carefree was, in fact, simply -------.
(A)

capricious

(C) extraneous

(D) captivating

(E)

inscrutable

ANSWERS

AND EXPLANATIONS

nR

Explanation for Correct Answer A : Choice (A) is correct. Capricious means irresponsibly unpredictable. If one were to insert this term into the text, the sentence would read After David left him waiting for the third consecutive time, Kirk realized that the same behavior he had initially valued as spontaneous and carefree was, in fact, simply capricious. The phrase initially valued suggests that after being left waiting three times in a row, Kirk changed his opinion of Davids behavior. It would make sense to say that Kirk would no longer value Davids behavior if he came to see it as capricious, or irresponsibly unpredictable.

Explanation for Incorrect Answer B : Choice (B) is incorrect. Incontrovertible means indisputable or not open to question. If one were to insert this term into the text, the sentence would read After David left him waiting for the third consecutive time, Kirk realized that the same behavior he had initially valued as spontaneous and carefree was, in fact, simply incontrovertible. The phrase initially valued indicates that after being left waiting three times in a row, Kirk changed his opinion of Davids behavior. There is no reason to believe that Kirk would not have initially believed that Davids behavior was incontrovertible, or not open to question.

Explanation for Incorrect Answer C : Choice (C) is incorrect. Extraneous means irrelevant and not essential. If one were to insert this term into the text, the sentence would read After David left him waiting for the third consecutive time, Kirk realized that the same behavior he had initially valued as spontaneous and carefree was, in fact, simply extraneous. The phrase initially valued suggests that after being left waiting three times in a row, Kirk changed his opinion of Davids behavior. If Davids behavior affected Kirks opinion, it is illogical to say that Davids behavior was extraneous, or irrelevant.

Explanation for Incorrect Answer D : Choice (D) is incorrect. Captivating means irresistibly appealing. If one were to insert this term into the text, the sentence would read After David had left him waiting for the third consecutive time, Kirk realized that the same behavior he had initially valued as spontaneous and carefree was, in fact, simply captivating. The phrase initially valued indicates that after being left waiting three times in a

eg

is te r

ed

(B)

incontrovertible

file://E:\\e3.htm

2006-11-12

The Official SAT Online Course

8/23

row, Kirk no longer valued Davids behavior. It is illogical to suggest that being left waiting caused Kirk to consider Davids behavior to be captivating, or irresistibly appealing.

Explanation for Incorrect Answer E : Choice (E) is incorrect. Inscrutable means mysterious and not readily understood. If one were to insert this term into the text, the sentence would read After David had left him waiting for the third consecutive time, Kirk realized that the same behavior he had initially valued as spontaneous and carefree was, in fact, simply inscrutable. The phrase the same behavior indicates that while Kirk s opinion of Davids behavior changed, Davids behavior was always the same. Therefore, the missing term should be another way to describe spontaneous and carefree behavior. Behavior that is spontaneous and carefree is not necessarily inscrutable, or mysterious.

The author of the passage uses quotation marks in line 13 in order to indicate that

nR
(B)

(A)

this theory is the one with which the author most nearly agrees

(C) a common word is being used to describe a unique biological process

(D) a word is being used in a humorous way

this theory is less scientifically valid than the other theories in the passage

(E)

a direct quotation from another source is being used

ANSWERS

Explanation for Correct Answer C : Choice (C) is correct. In the passage, three theories are given concerning the original purpose of the human appendix. In lines 12-16, the third and final theory is given: the appendix may attract body infections in order to localize the infection in one spot that is not critical to body functioning. The use of the quotation marks around attract implies that there are probably technical terms to describe this attraction, but that the author is instead using a term that the general reader will be likely to understand. The reader knows that a bodily organ cannot really attract, or draw the attention of, an infection, but the use of this familiar verb helps clarify the concept.

Explanation for Incorrect Answer A : Choice (A) is incorrect. There is no indication that the author of the passage supports the third theory any more than the other two theories, nor would the use of quotation marks necessarily serve this purpose.

Explanation for Incorrect Answer B : Choice (B) is incorrect. There is no indication that this third theory is any less

eg

What was most likely the original purpose of the human appendix? Experts can only theorize on its use. It may have Line had the same purpose it has in present5 day herbivores, where it harbors colonies of bacteria that help in the digestion of cellulose. Another theory suggests that tonsils and the appendix might manufacture the antibody-producing white 10 blood cells called B lymphocytes; however, B lymphocytes could also be produced by bone marrow. The third theory is that the appendix may attract body infections in order to localize the 15 infection in one spot that is not critical to body functioning.

AND EXPLANATIONS

is te re

file://E:\\e3.htm

2006-11-12

The Official SAT Online Course

9/23

scientifically valid than the other two theories. Using a non-technical term like attracts to clarify a concept does not suggest that that concept is less than valid.

Explanation for Incorrect Answer D : Choice (D) is incorrect. While quotation marks might in certain cases connote humor, that is not the case here. The tone of the sentenceand of the whole passageis objective and dispassionate.

Explanation for Incorrect Answer E : Choice (E) is incorrect. While marking a direct quotation from another source is one of the main uses of quotation marks, there is no indication here that the author of the passage is deriving the word attract from another source.

10

How does the theory described in lines 3-7 primarily differ from the other two theories described in the passage?
(A)

It pertains only to plants.

(B)

It concerns a physical process that occurs in more than one area of the human body.

(C) It is a theory supported by more experts in the field than are the other two theories.

(D) It is concerned with the prevention of disease.

(E)

It makes reference to a process presently occurring in other animals.

ANSWERS

AND EXPLANATIONS

One hazard in historical study is the necessity of dividing the whole into segments, since not everything can be Line examined simultaneously. Common ways 5 of dividing history are by period, country, topic, artistic or political movement, or theme. Each of these can be justified, but
file://E:\\e3.htm

nR

Explanation for Incorrect Answer A : Choice (A) is incorrect. None of the three theories described in the passage pertains only to plants. The first theory mentions plants, but only in discussing the digestion of cellulose by herbivores.

Explanation for Incorrect Answer B : Choice (B) is incorrect. It is the second theory (lines 7-12), not the theory in lines 3-7, that deals with a process possibly occurring in two areas of the human body: the production of lymphocytes in the appendix and the tonsils.

Explanation for Incorrect Answer C : Choice (C) is incorrect. There is no indication that any of the three theories described in the passage enjoys more scientific support than do the others. The only mention of experts is in line 3 and pertains to all the theories.

Explanation for Incorrect Answer D : Choice (D) is incorrect. Both the second and third theories could be said to pertain to disease preventionthe second by aiding in the production of antibodies, the third by isolating infectionsso disease prevention cannot be said to be represented in the first theory only.

eg

is

Explanation for Correct Answer E : Choice (E) is correct. The theory described in lines 3-7 states that the human appendix may originally have harbored colonies of bacteria to aid in digestion, which is its purpose in present-day herbivores. Neither of the other theories in the passagein lines 7-12 and 12-16refers to present-day animals, herbivorous or carnivorous.

te r

ed

2006-11-12

The Official SAT Online Course

10/23

all have their shortcomings. When divisions are made according to country, 10 the interconnections among events occurring in two or more countries may go unnoticed or remain unexplored. Division into time periods may interrupt or obscure ongoing developments, or may give undue 15 emphasis to some event or type of activity (especially war or politics) as crucial in marking the end or beginning of a period or movement.
11

Which of the following is most analogous to the hazard the author sees in the division of historical study?
(A)

A lawyer accepts cases in too many different areas of legal practice.

(B)

A teacher must cope with large class sizes and is unable to give students sufficient individual instruction.

(C) A biologist studies large areas of forest but fails to examine in depth the nesting site of a specific bird species.

(D) An artist produces works in many different media, but does not excel in any one medium.

(E)

A doctor diagnoses one ailment but overlooks elements of the patients overall health.

ANSWERS

AND EXPLANATIONS

nR eg

Explanation for Correct Answer E : Choice (E) is correct. The author argues in this passage that dividing history into segments for instructional purposes is always risky. No matter how history is divided, distortions of emphasis will occur. Focusing on one part of historyan international period or a single nations history, for instanceleaves larger connections unexplored. This type of hazard is analogous to the challenge doctors face when assessing a patients health. A doctor might make a specific diagnosis associated with an ailing portion of a patients body in order to recommend treatment, but may inadvertently overlook larger, more global elements of the patients health.

Explanation for Incorrect Answer A : Choice (A) is incorrect. The author argues in this passage that dividing history into segments for instructional purposes is always risky. No matter how history is divided, distortions of emphasis will occur. Focusing on one part of historyan international period or a single nations history, for instanceleaves larger connections unexplored. A lawyer accepting cases in too many different areas of legal practice would be a reverse of the hazard in historical studies: the lawyer would be spread too thin by too wide a focus, instead of missing the bigger picture because of a specializing too narrowly.

Explanation for Incorrect Answer B : Choice (B) is incorrect. The author argues in this passage that dividing history into segments for instructional purposes is always risky. No matter how history is divided, distortions of emphasis will occur. Focusing on one part of historyan international period or a single nations history, for instanceleaves larger connections unexplored. Although coping with a large class size could be similar to dealing with large portions of history, the inability to give individual students attention is not the same type of hazard the author mentions in historical studies. The authors point is that too much attention is given to individual sections of history at the expense of the bigger picture.

Explanation for Incorrect Answer C : Choice (C) is incorrect. The author argues in this passage that dividing history into segments for instructional purposes is always risky. No matter how history is divided, distortions of emphasis will occur. Focusing on one part of historyan international period or a single nations history, for instanceleaves larger connections unexplored. A biologist studying large areas of forest at the expense of focusing on a specific bird species would be the reverse of the hazard in historical studies: the biologist would overlook details because of too wide a focus,

is

te

re d

file://E:\\e3.htm

2006-11-12

The Official SAT Online Course

11/23

instead of overlooking the big picture because of a narrow focus.

Explanation for Incorrect Answer D : Choice (D) is incorrect. The author argues in this passage that dividing history into segments for instructional purposes is always risky. No matter how history is divided, distortions of emphasis will occur. Focusing on one part of historyan international period or a single nations history, for instanceleaves larger connections unexplored. An artist working in many different media would be the reverse of the hazard in historical studies: the artist would be spread too thin by too wide a focus, instead of missing the bigger picture because of specializing too narrowly.

12

The author implies which of the following about war and politics in historical studies (line 16)? (A) They make the study of international movements difficult.
(B)

They serve primarily as a convenience to the reader.

(C) They are more helpful to use in defining periods than in defining movements.

(D) They are equally important to historians and to readers.

(E)

They are commonly used to define historical periods.

Explanation for Incorrect Answer A : Choice (A) is incorrect. In this passage, the author mentions several ways in which history is divided for instructional purposes. War and politics are used as examples of events or activities used to define historical periods. While the author says that all such divisions have their shortcomings, he or she does not imply that these particular divisionswar and politicsmake the study of international movements difficult. Such a division might, it could be argued, make the study of international movements easier than would, say, division according to nation.

nR

Explanation for Incorrect Answer B : Choice (B) is incorrect. In this passage, the author mentions several ways in which history is divided for instructional purposes. War and politics are used as examples of events or activities used to define historical periods. While the author says that all such divisions have their shortcomings, he or she does not imply that these particular divisionswar and politicsserve primarily as a convenience to the reader. In fact, the author seems to imply that such division may not be a convenience, but may actually inconvenience the reader by distorting the importance of particular events.

Explanation for Incorrect Answer C : Choice (C) is incorrect. In this passage, the author mentions several ways in which history is divided for instructional purposes. War and politics are used as examples of events or activities commonly used to define historical periods. The author does not imply that these particular divisionswar and politicsare more useful in defining periods than in defining movements. Rather, the author states that all such divisions introduce certain hazards and oversights into historical study.

Explanation for Incorrect Answer D : Choice (D) is incorrect. In this passage, the author mentions several ways in which history is divided for instructional purposes. War and politics are used as examples of events or activities commonly used to define historical periods. The author does not discuss the importance of these particular divisionswar and politicsto either the historian who writes the history or the reader who reads history; the author says only that such divisions are common and potentially hazardous.

eg

is

Explanation for Correct Answer E : Choice (E) is correct. In this passage, the author states that dividing history into time periods may overemphasize some event or type of activity commonly used to mark the end or beginning of a period or movement. The author includes war or politics as examples of types of activities that are used to mark the border of a historical period, that is, the end or beginning of a period or movement.

te

re

ANSWERS

AND EXPLANATIONS

file://E:\\e3.htm

2006-11-12

The Official SAT Online Course

12/23

The following passages discuss a type of film called film noir, which, according to most film historians, had its high point around the time of the Second World War (1939-1945).

Passage 1

Even though films now called film noir by critics have been made in Hollywood since 1939, film noir as a genre Line did not exist until 1946. In that year an 5 exhibition of American movies was held in Paris, and French film critics got their first look at what had been going on in Hollywood since the advent of World War II. Among the films shown were Laura; 10 The Maltese Falcon; Murder, My Sweet; Double Indemnity; and The Woman in the Window. Those five films shared enough traits that critic Nino Frank gave them a new classification: film noir, or literally, 15 black film. The traits they shared were both stylistic and thematic. They were dark in both look and mood. Their primary action took place at night on rainswept city streets, in narrow ash-can 20 alleys, in claustrophobic diners, and in dingy, shadowy hotel rooms with neon signs flashing outside the windows, rooms in which, as hard-boiled author Nelson Algren once put it, every bed you rent 25 makes you an accessory to somebody elses shady past. The characters in these films were bookies, con men, killers, cigarette girls, crooked cops, down-andout boxers, and calculating, scheming, 30 and very deadly women. The well-lit, singing and tap-dancing, happy-ending world of the 1930s had in ten short years become a hostile, orderless place in which alienation, obsession, and paranoia ruled. 35 The universe seemed to conspire to defeat and entrap the inhabitants who wandered blindly through it. They were victims of fate, their own worst enemies who, looking for a score, ended by defeating 40 themselves. The five films mentioned earlier that were shown at the 1946 exhibition were the ones the French critiqued. These high-budget studio productions most 45 commonly come to the publics mind when the word noir is mentioned because they

nR

eg i

st

er e

file://E:\\e3.htm

2006-11-12

The Official SAT Online Course

13/23

are cited most often in the spate of contemporary books that have recently been published on the subject. But the 50 noir cycle, although kick-started by the success of those high-budget productions, actually had its roots in the B movie, in particular, in the B crime movie. Film noir was made to order for the B, or low55 budget, part of the movie double bill1. It

70

75

80

85

90

95

It may be that noir began in a way of photographing that was as economical as it was moody (less light meant less money on decoran important wartime consideration when studios faced limits on construction material). Where did noir come from? Its an intriguing question and one still not adequately answered, despite the quantity of writing that wallows in that noir mood. Dont rule out the influence of German film from the twenties, if only because there were, by the early forties, so many European refugees (writers, directors, camera operators, designers, actors) working in Hollywood. Dont forget the impact of French films of the late thirties, especially those of Marcel Carne. His Le Jour Se Leve (1939, called Daybreak in the United States) was such a success that it was remade in Hollywood in 1947 as The Long Night. Finally, dont underestimate the influence Citizen Kane had on anyone whose art and craft was cinematography. The film was a box office flop, but filmmakers were absorbed by it. A landmark in so much, Kane is a turning point in the opening up of a noir sensibility. Equally, dont forget that from the

nR

eg

is

te

re

Passage 2

was cheaper to produce because it required less lighting and smaller casts and usually entailed story lines that required limited-scale setsan attractive 60 quality to film studios operating on reduced wartime budgets. Film noir was character-driven, and its story lines, which were unusual and compact, could often be told in the 60 to 80 minutes required of B 65 pictures.

file://E:\\e3.htm

2006-11-12

The Official SAT Online Course

14/23

1 In the 1940s, a film showing would typically include a longer, more expensively-produced film (an A film) and a shorter, less-expensively produced film (a B film)in other words, a double bill.

2 Film actresses who were popular during the 1940s.

13

nR

(A)

In lines 1 through 4 of Passage 1, the author suggests that films now called film noir by critics

were not classified as film noir when first made

(B)

were reminiscent of earlier European films

(C) were uplifting in mood and theme

(D) were intended to contrast with films of the 1930s

(E)

were disliked by many French film critics

ANSWERS

Explanation for Correct Answer A : Choice (A) is correct. The passage states that while films now classified as film noir had been made since 1939, film noir as a genre did not exist until 1946. Thus the films now called film noir by critics were not classified as film noir when they were first made, since the term was not coined until after the Paris exhibition.

eg i

Jones, and Shirley Temple coping in the absence of men. That, too, looks like a 120 film noir. But the mood is entirely that of innocent, ardent, flawless hope, and assurance that when the war ends everything will revert to calm and order.

AND EXPLANATIONS

st e

re
2

forties onward, Los Angeles was much beset by psychoanalysis, and the growing intellectual interest in guilt, depression, and nightmare. Dont eliminate the 100 impact, the memory, or the mere thought of a wars damage. I want to stress how deeply noir impulses lay in the common imagination that of the audience as well as the 105 filmmakers. Mildred Pierce, for instance which appeared in 1945 when many American women were running businesses of their own just to survive while the men were away at warsighs and seems to 110 say, It doesnt make any difference, why bother, for there is something malign in human nature or luck that will undermine enterprise and hope. Of course, not every film was so bleak. Look instead at 115 David Selznicks very beautiful and touching Since You Went Away, which is all about Claudette Colbert, Jennifer

file://E:\\e3.htm

2006-11-12

The Official SAT Online Course

15/23

Explanation for Incorrect Answer B : Choice (B) is incorrect. The first four lines of the passage state that films now called film noir were made before film noir had been recognized as its own genre. So while the passage does speak of films earlier than 1946, nothing is suggested here about the relationship between these American films and earlier European films.

Explanation for Incorrect Answer C : Choice (C) is incorrect. The first four lines of the passage state that films now called film noir were made before film noir had been recognized as its own genre. It stands to reason, then, that since these films were film noir in everything but namesince the name had not yet been inventedthey were anything but uplifting in mood and theme. Rather, they were almost certainly bleak in mood and theme.

Explanation for Incorrect Answer D : Choice (D) is incorrect. The first four lines of the passage state that films now called film noir were made before film noir had been recognized as its own genre. While it is suggested in lines 30-32 that certain films of the 1930s (well-lit, singing and tap-dancing, happy-ending) did contrast markedly with the film noir, there is nothing in the passage to indicate that either type of film was intended to contrast with the other.

14

(A)

visual appearance

(C) characters

(D) theme

nR
(E)

music

ANSWERS

Explanation for Correct Answer E : Choice (E) is correct. The passage discusses many similarities among the five films mentioned in lines 9 through 12; they shared enough traits that critic Nino Frank gave them a new classification. The passage states that the films were similar in look and mood, that they shared thematic traits, and that many of them featured the same types of characters. While the music in these films may or may not be similar, music is not discussed in the passage, and so no inferences can be drawn from the passage about it.

Explanation for Incorrect Answer A : Choice (A) is incorrect. The passage states that the five films mentioned in lines 9 through 12 were dark in both look and mood, which implies that they were indeed similar in visual appearance.

Explanation for Incorrect Answer B : Choice (B) is incorrect. The passage states that the five films mentioned in lines 9 through 12 were dark in both look and mood; further, the world of these films was a hostile, orderless place in which alienation, obsession, and paranoia ruled. Together, these descriptions imply that the films were indeed similar in emotional effect.

Explanation for Incorrect Answer C : Choice (C) is incorrect. The passage explicitly states that the films shared similar types of characters. In lines 26- 29, several character types are listed, including

eg

(B)

emotional effect

AND EXPLANATIONS

is t

It can be inferred that the films listed in lines 9 through 12 were similar in each of the following ways EXCEPT:

er

ed

Explanation for Incorrect Answer E : Choice (E) is incorrect. The first four lines of the passage state that films now called film noir were made before film noir had been recognized as its own genre. If these films were later termed film noir, a term coined by French critics, it stands to reason that French critics liked rather than disliked them.

file://E:\\e3.htm

2006-11-12

The Official SAT Online Course

16/23

crooked cops, down-and-out boxers, and calculating, scheming, and very deadly women.
Explanation for Incorrect Answer D : Choice (D) is incorrect. The passage states in line 16 that the traits the films shared were both stylistic and thematicin other words, that they had similarities in theme.

15

In paragraph 2, the author says that the films discussed in paragraph 1 are NOT typical of their genre in regard to
(A)

setting

(B)

budget

(C) country of origin

(D) plot

(E)

lighting

ANSWERS

AND EXPLANATIONS

16

nR eg

(A)

Explanation for Incorrect Answer A : Choice (A) is incorrect. Although paragraph 2 states that typical film noir entailed story lines that required limited-scale sets, this is not mentioned as a difference between the typical film noir and the films mentioned in paragraph 1. The author s point is not that these settings were different from those of the films mentioned, but that film noir did not, in general, require a large budget to produce.

Explanation for Incorrect Answer C : Choice (C) is incorrect. Paragraph 2 says nothing about film noirs country of origin, though the passage as a whole calls film noir an American phenomenon first named by French film critics. Rather, paragraph 2 discusses the low budgets of the typical film noir.

Explanation for Incorrect Answer D : Choice (D) is incorrect. Although paragraph 2 states that typical film noir plots were character-driven and unusual and compact, this is not mentioned as a difference between the typical film noir and the films mentioned in paragraph 1. The authors point is not that these plots were different from those of the films mentioned, but that film noir did not, in general, require a high budget to produce.

Explanation for Incorrect Answer E : Choice (E) is incorrect. Paragraph 2 says nothing about film noirs lighting, though the passage as a whole indicates that the typical film noir was dimly lighted. Rather, paragraph 2 discusses the low budgets of typical film noir.

The author of Passage 1 uses the quotation in lines 24-26 primarily in order to

critique a writer

(B)

recount an incident

(C) evoke a place

(D) describe a character

(E)

summarize a plot

is

te

re

Explanation for Correct Answer B : Choice (B) is correct. Paragraph 2 discusses the budgets of film noir films. The author argues that although the high-budget productions mentioned in the first paragraph helped kick-start film noir, the roots of film noir were in B movies. B movies, the passage states, were low-budget, cheaper to produce films used as the less expensive half of double features. The higher budget is the only difference presented here between the films mentioned in paragraph 1 and typical films of the film noir genre.

file://E:\\e3.htm

2006-11-12

The Official SAT Online Course

17/23

ANSWERS

AND EXPLANATIONS

Explanation for Correct Answer C : Choice (C) is correct. In the second half of the first paragraph, the author describes the characteristics of film noir, including setting: the claustrophobic diners and dingy, shadowy hotel rooms where film noir is commonly set. The author quotes Nelson Algrens description of the hotels in which every bed you rent makes you an accessory to somebody elses shady past in order to evoke strongly the typical places where action occurs in film noir.

Explanation for Incorrect Answer A : Choice (A) is incorrect. In the second half of the first paragraph, the author describes the characteristics of film noir, including setting. The author uses Nelson Algrens quotation to help evoke the types of places typically found in film noir. There is no critique of Algrens writing. In fact, the author seems to agree wholeheartedly with Algrens description.

Explanation for Incorrect Answer B : Choice (B) is incorrect. In the second half of the first paragraph, the author describes the characteristics of film noir, including setting. The author uses Nelson Algrens quotation to help evoke the types of places found in film noir. The quote does not recount a specific incident, but, rather, refers to film noir settings in general.

17

nR
(A)

The author of Passage 1 suggests that the spate of contemporary books (lines 4748) on film noir

(B)

discusses only five films

focuses on non-crime films

(C) focuses on relatively costly noir films

(D) is inaccurate in their historical data

(E)

is from big-budget publishing companies

ANSWERS

Explanation for Correct Answer C : Choice (C) is correct. The author states that the spate of contemporary books on film noir cites most often the five films shown at the 1946 exhibition. These five films, consequently, have dominated public perception of film noir. The author argues that this perception of film noir may be inaccurate, however, because the films most discussed in the books were much more costly to produce than were most film noir films. According to the author, film noir films were very often lowbudget B movies.

Explanation for Incorrect Answer A : Choice (A) is incorrect. The author states that the spate of contemporary books on film noir cites most often the five films shown at the 1946 exhibition. The author does not state that these five films are the only films discussed in contemporary writing about film noir.

eg

Explanation for Incorrect Answer E : Choice (E) is incorrect. In the second half of the first paragraph, the author describes the characteristics of film noir, including setting. The author uses Nelson Algrens quotation to help evoke the types of places generally found in film noir, not to summarize the plot of any specific film.

AND EXPLANATIONS

is

te

Explanation for Incorrect Answer D : Choice (D) is incorrect. In the second half of the first paragraph, the author describes the characteristics of film noir, including setting. The author uses Nelson Algrens quotation to help evoke the type of places generally found in film noir, not to describe any specific character, though character types are discussed in the first paragraph of the passage.

re

file://E:\\e3.htm

2006-11-12

The Official SAT Online Course

18/23

Explanation for Incorrect Answer B : Choice (B) is incorrect. The author states that the spate of contemporary books on film noir cites most often the five films shown at the 1946 exhibition. Although the author states that the roots of film noir were in the B crime movie and that the five high-budget films are uncharacteristic, this does not imply that the five films were not crime films themselves.

Explanation for Incorrect Answer D : Choice (D) is incorrect. The author states that the spate of contemporary books on film noir cites most often the five films shown at the 1946 exhibition. Although the author argues that focusing on these five films produces a slightly inaccurate understanding of film noir, the historical data is not termed inaccurate. The author agrees that these films were shown at the exhibition and that the exhibition was vital to the establishment of film noir as a genre.

Explanation for Incorrect Answer E : Choice (E) is incorrect. Although the author states that the spate of contemporary books focuses on films that were high-budget studio productions, nothing is said in the passage about the companies that published the books.

18

The authors of both passages imply that contemporary writing about film noir
(A)

comes mainly from French film critics

(B)

has failed to describe the origins of film noir accurately

(D) is inferior to earlier writing on film noir

(E)

has dramatically improved the understanding of film noir

ANSWERS

AND EXPLANATIONS

19

nR

Explanation for Correct Answer B : Choice (B) is correct. Both authors argue that contemporary writing on film noir fails to describe its origins accurately. The author of Passage 1 argues that the spate of contemporary books on film noir inaccurately portrays film noirs origins by focusing on five high-budget films shown at the 1946 exhibition. According to the author, this focus de-emphasizes the fact that film noir actually had its roots in the B movie, which were much lower-budget productions. The author of Passage 2 states that the quantity of writing that wallows in that noir mood has not adequately answered the question, Where did noir come from? Thus, both authors think that many modern writers ignore or miss the point about the real origins of film noir.

Explanation for Incorrect Answer A : Choice (A) is incorrect. Neither author mentions the national origin of contemporary writing about film noir. The author of Passage 1 discusses the importance French film critics played in establishing the film noir genre in 1946, but says nothing in specific about contemporary writing by French critics.

Explanation for Incorrect Answer C : Choice (C) is incorrect. Both authors state that contemporary writing about film noir mischaracterizes the origins of film noir, not the mood of film noir. The authors seem to agree on the dark mood of film noir.

Explanation for Incorrect Answer D : Choice (D) is incorrect. While both authors criticize contemporary writing as wrongheaded about the origins of film noir, neither author compares the quality of this writing with that of earlier writing.

Explanation for Incorrect Answer E : Choice (E) is incorrect. Neither author states that contemporary writing has improved the understanding of film noir. On the contrary, they both argue that contemporary writing has inadequately described the origins of film noir.

Both passages imply that the development of film noir can be attributed in part to

eg is t

er

ed

(C) mischaracterizes the film noir mood

file://E:\\e3.htm

2006-11-12

The Official SAT Online Course

19/23

(A)

the presence of European filmmakers in Hollywood

(B)

the influential writing of French film critics

(C) economic restraints resulting from World War II

(D) United States filmmakers dissatisfaction with high-paid actors

(E)

the popularity of Citizen Kane

ANSWERS

AND EXPLANATIONS

Explanation for Correct Answer C : Choice (C) is correct. Both passages highlight the relatively low cost of making most film noir films. They both state also that this low cost was central to film noir production because low cost was consistent with economic restraints resulting from World War II. Passage 1 states that the limited-scale sets of film noir were an attractive quality to film studios operating on reduced wartime budgets. Passage 2 states that the low lighting that contributed to the mood of film noir was also cheaper, an important wartime consideration when studios faced limits on construction material.

20

nR eg
(A)

Explanation for Incorrect Answer E : Choice (E) is incorrect. Although Passage 2 states that Citizen Kane was influential in the development of film noir, Passage 1 does not mention Citizen Kane at all.

The phrase wallows in in line 75 is closest in meaning to

indulges in

(B)

conforms to

(C) criticizes

(D) explores

(E)

reveals

ANSWERS

AND EXPLANATIONS

Explanation for Correct Answer A : Choice (A) is correct. In lines 72-75, Passage 2 discusses the abundant material written about film noir. The author is saying that writers have failed to explain accurately the origins of film noir, despite the large quantity of writing that wallows in, or is excessively involved with, film noir. In other words, the writing has been insufficient despite its indulgence in the topic.

Explanation for Incorrect Answer B : Choice (B) is incorrect. In lines 72-75, Passage 2 discusses the abundant material written about film noir. The passage is not arguing that the writing conforms to, or shares, the film noir mood, but that it has insufficiently explained film noirs origins despite its excessive involvement with the topic.

is

Explanation for Incorrect Answer D : Choice (D) is incorrect. Neither passage discusses high-paid actors or United States filmmakers attitudes towards them.

te

Explanation for Incorrect Answer B : Choice (B) is incorrect. Although Passage 1 discusses the importance of French film critics in the establishment of film noir as a genre, Passage 2 speaks of European film directors but does not mention French film critics at all.

re

Explanation for Incorrect Answer A : Choice (A) is incorrect. Although Passage 2 claims the presence of European filmmakers in Hollywood was influential in the development of film noir, Passage 1 speaks of French film critics but makes no mention of European filmmakers in Hollywood.

file://E:\\e3.htm

2006-11-12

The Official SAT Online Course

20/23

Explanation for Incorrect Answer C : Choice (C) is incorrect. In lines 72-75, Passage 2 discusses the abundant material written about film noir. The passage is not stating that the writing criticizes, or disapproves of, film noir, but that it has insufficiently explained film noirs origins despite its excessive involvement with the topic.

Explanation for Incorrect Answer D : Choice (D) is incorrect. In lines 72-75, Passage 2 discusses the abundant material written about film noir. The passage is not simply stating that this material explores film noir, but that is has insufficiently explained film noirs origins, despite its excessive involvement with the topic.

Explanation for Incorrect Answer E : Choice (E) is incorrect. In lines 72-75, Passage 2 discusses the abundant material written about film noir. The passage is not stating that this material has revealed or exposed, anything about film noir. On the contrary, it states that the writing has failed to sufficiently explain film noirs origins despite its excessive involvement with the topic.

21

The author of Passage 2 mentions Since You Went Away in line 116 primarily to
(A)

indicate that not all films in the 1940s expressed the same attitude

(B)

illustrate the popularity of film noir actors

(C) demonstrate that most film noir dealt with WWII

nR

Explanation for Incorrect Answer B : Choice (B) is incorrect. Although the author mentions that Since You Went Away starred certain popular film actors, the authors point is that the innocent and hopeful attitude of Since You Went Away is different from the bleak mood of other films mentioned in the passage. The films cast is not necessarily relevant to the argument about the films different attitude.

Explanation for Incorrect Answer C : Choice (C) is incorrect. Although the author states that Since You Went Away deals with women during World War II, the authors point is that that the innocent and hopeful attitude of Since You Went Away is different from the bleak mood of other films mentioned in the passage. The author does not say that any other film noir films dealt with World War II, though it is likely that many did.

Explanation for Incorrect Answer D : Choice (D) is incorrect. Although the author states that Since You Went Away portrayed women dealing with the absence of men during World War II, the authors point is that the innocent and hopeful attitude of Since You Went Away is different from the bleak mood of other films mentioned in the passage. The author does not say that any other film noir films dealt with the absence of men.

Explanation for Incorrect Answer E : Choice (E) is incorrect. The author does not state that David Selznick was influential in defining film noir, but only that Selznicks film Since You Went Away had a different, more hopeful mood, than the bleak mood of other films mentioned in the passage.

eg

Explanation for Correct Answer A : Choice (A) is correct. The first half of the fifth paragraph discusses the apathetic and hopeless mood of many film noir films. The author then mentions Since You Went Away to point out that not every film of the time was so bleak. The author argues that Since You Went Away looks like a film noir, even though its mood is entirely that of innocent, ardent, flawless hope, an attitude quite different from film noir films previously discussed in the passage.

is te

ANSWERS

AND EXPLANATIONS

re

(E)

point out David Selznicks influence in defining film noir

(D) show that the absence of men was a major film noir theme

file://E:\\e3.htm

2006-11-12

The Official SAT Online Course

21/23

22

The second paragraph of Passage 2 is best described as


(A)

a catalog of possible sources of the film noir mood

(B)

a description of the mood that characterizes film noir

(C) a refutation of several misconceptions about film noir

(D) an argument for redefining the term film noir

(E)

a list of the major film noir films

ANSWERS

AND EXPLANATIONS

Explanation for Correct Answer A : Choice (A) is correct. Paragraph 2 begins with the question, Where did noir come from? The paragraph then suggests different possible sources for film noir by exhorting the reader not to rule out or forget, or underestimate various influences. The paragraph catalogs German film from the twenties, French films of the late thirties, and Citizen Kane as influential in the formation of the film noir genre.

23

nR

(A)

Explanation for Incorrect Answer E : Choice (E) is incorrect. Although paragraph 2 does mention several films, these films are not presented as major film noir films. Instead, the paragraph lists them as possible sources for the style that became film noir.

The quote It doesnt makeenterprise and hope in lines 110-113 is primarily meant to

give an example of dialogue from Mildred Pierce

(B)

summarize what critics thought about Mildred Pierce

(C) show how Mildred Pierce differs from other films made in 1945

(D) characterize the sentiments expressed in Mildred Pierce

(E)

demonstrate the influence of Mildred Pierce on Since You Went Away

ANSWERS

Explanation for Correct Answer D : Choice (D) is correct. The passage argues that the noir impulses reflected the common sentiments of people in general during the 1940s. The quotation in lines 110-113 characterizes the attitude expressed by the film Mildred Pierce. This attitude, the passage argues, was present in the imagination of many people at that time.

Explanation for Incorrect Answer A : Choice (A) is incorrect. The quotation in lines 110-113 is not actual dialogue from Mildred Pierce. The quotation is, according to the author, a characterization of the

eg is

Explanation for Incorrect Answer D : Choice (D) is incorrect. Paragraph 2 does not attempt to define the term film noir. The paragraph assumes a general definition of film noir, then lists, or catalogs, several possible sources of film noir.

AND EXPLANATIONS

te r

Explanation for Incorrect Answer C : Choice (C) is incorrect. Paragraph 2 does not address misconceptions about film noir. The paragraph states that determining the origins of film noir is difficult, and then lists possible sources for consideration. The sources are presented as possibilities, not as misconceptions.

ed

Explanation for Incorrect Answer B : Choice (B) is incorrect. Paragraph 2 does not address the film noir mood specifically. Instead, it lists possible answers to the question, Where did noir come from? The paragraph presents sources of film noir, not elements of its mood.

file://E:\\e3.htm

2006-11-12

The Official SAT Online Course

22/23

general attitude the film presents. This attitude, the passage argues, was shared by many people during the 1940s.

Explanation for Incorrect Answer B : Choice (B) is incorrect. The quotation in lines 110-113 does not refer to comments critics made about Mildred Pierce. The quotation is, according to the author, a characterization of the general attitude the film presents. This attitude, the passage argues, was shared by many people during the 1940s.

Explanation for Incorrect Answer C : Choice (C) is incorrect. The quotation is a characterization of the general attitude the film presentsan attitude, the passage argues, that was shared by many people during the 1940s. Although the passage does state that not every film was so bleak, the paragraph argues that this attitude was the characteristic of many film noir films. The quotation is used to express a general sentiment in many film noir films, not a difference between Mildred Pierce and other films.

Explanation for Incorrect Answer E : Choice (E) is incorrect. Although the paragraph discusses both Mildred Pierce and Since You Went Away, the quotation is used to help distinguish between the two films. The quotation in lines 110-113 characterizes the attitude expressed by the film Mildred Pierce. This attitude, the passage argues, was present in the imagination of many people at that time. The paragraph explicitly states that this attitude was NOT shared by Since You Went Away.

The word impulses in line 103 is closest in meaning to


(A)

incentives

(B)

stimulants

(C) fantasies

(D) transformations

(E)

feelings

nR

Explanation for Correct Answer E : Choice (E) is correct. The passage argues that noir impulses reflected the common sentiments of people in the 1940s. The author implies that these impulses, or feelings, were shared by both filmmakers and filmgoers and that such feelings were present in the common imagination of the time. The quotation that the author uses to support this pointlines 110-113is an expression of a pessimistic feeling about life.

Explanation for Incorrect Answer A : Choice (A) is incorrect. The passage argues that the noir impulses reflected the common sentiments of people in the 1940s. These impulses were the feelings people had about their lives, not incentives, or motivating factors, to impel them to do something.

Explanation for Incorrect Answer B : Choice (B) is incorrect. The passage argues that the noir impulses reflected the common sentiments of people in the 1940s. These impulses were the feelings people had about their lives, not stimulants or factors that would stimulate them to do something.

Explanation for Incorrect Answer C : Choice (C) is incorrect. The passage argues that the noir impulses reflected the common sentiments of people in the 1940s. These impulses were the feelings, not fantasies, many people had about their lives. In fact, the grim feelings expressed in the paragraph are the opposite of what is usually thought of as a fantasy, or pleasant notion or daydream.

Explanation for Incorrect Answer D : Choice (D) is incorrect. The passage argues that the noir impulses reflected the common sentiments of people in the 1940s. These impulses were the feelings people had about their lives, not any type of transformation, or profound change, they might have experienced.

eg

ANSWERS

AND EXPLANATIONS

is t

er

ed

24

file://E:\\e3.htm

2006-11-12

The Official SAT Online Course

23/23

Back to Score Report

Copyright 2006 The College Board. All rights reserved.

Privacy Policy

Terms of Use

Contact Us

nR eg

is

te

re

d
file://E:\\e3.htm

2006-11-12

The Official SAT Online Course

1/17

Help | Profile | My Organizer | My Bookmarks | Logout

Answers and Explanations

Test Sections

Back to Score Report

Section 1

View Answers and Explanations


Online - Practice Test #5

Section 2

Section 3

Section 4

Which of the following triples

does NOT satisfy

Section 5

Section 6

(A)
(B)
(C)
(D)
(E)

Section 8

Section 9

Section 10

Explanation for Correct Answer C :

Choice (C) is correct. If

ed

then

ANSWERS

AND EXPLANATIONS


does not

satisfy

Explanation for Incorrect Answer A :

eg

Choice (A) is not correct. If

is t

nR

satisfy

Explanation for Incorrect Answer B :

Choice (B) is not correct. If

Explanation for Incorrect Answer D :

Choice (D) is not correct. If

Explanation for Incorrect Answer E :

Choice (E) is not correct. If

satisfy

er

then

Therefore, the triple

The question asks which triple does NOT satisfy

then

The question asks which triple does NOT satisfy

then

The question asks which triple does NOT

then

The question asks which triple does NOT

Susan had

trading cards. After giving away

cards and receiving

cards, she

had

cards. What is the value of

(A)
(B)
(C)

file://E:\\e4.htm

2006-11-12

The Official SAT Online Course

2/17

(D)
(E)

ANSWERS

AND EXPLANATIONS
cards, she

Explanation for Correct Answer A : trading cards. After giving away Choice (A) is correct. Susan had

had

cards; then, after receiving

cards, she had

cards. Since

Susan ended up with

cards, it follows that

Subtracting

from each side of this equation and rearranging gives

Explanation for Incorrect Answer B : Choice (B) is not correct. The value of

is

not

Explanation for Incorrect Answer C : Choice (C) is not correct. The value of

is

not

Explanation for Incorrect Answer D : Choice (D) is not correct. The value of

is

Explanation for Incorrect Answer E : Choice (E) is not correct. The value of

is

te

then

re
If

is a positive integer divisible by

and if


not

not

value of

(A)
(B)
(C)
(D)

nR

(E)

ANSWERS

Explanation for Correct Answer B :

eg is

AND EXPLANATIONS
is divisible by

Choice (B) is correct. If

In addition,

satisfies the condition

d
Therefore,

what is the greatest possible

because

There are no integers greater

that are divisible by and less than than that satisfies both conditions. possible integer

is the greatest

Explanation for Incorrect Answer A : is not divisible by Choice (A) is not correct. The integer

Therefore,

cannot be the greatest possible integer less than

that is divisible by

Explanation for Incorrect Answer C : is not divisible by Choice (C) is not correct. The integer

Therefore,

cannot be the greatest possible integer less than

that is divisible by

Explanation for Incorrect Answer D : is not divisible by Choice (D) is not correct. The integer

Therefore,

cannot be the greatest possible integer less than

that is divisible by

Explanation for Incorrect Answer E : is not divisible by Choice (E) is not correct. The integer

Therefore,

cannot be the greatest possible integer less than

that is divisible by

file://E:\\e4.htm

2006-11-12

The Official SAT Online Course

3/17

(A)
(B)
(C)
(D)
(E)

ANSWERS

nR

Explanation for Correct Answer E : Choice (E) is correct. The sales for Company

eg

AND EXPLANATIONS
are represented by the dashed line,

so the sales for the six months for that company were

is t

and

month decrease in sales for Company

Explanation for Incorrect Answer A :

Choice (A) is not correct. This choice,

for the month of August. The question asks and Company between Company for the largest one-month decrease in sales for Company

Explanation for Incorrect Answer B :

Choice (B) is not correct. This choice,

increase in sales for Company decrease in sales for Company

Explanation for Incorrect Answer C :

Choice (C) is not correct. This choice,

decrease in sales for Company

decrease in sales for Company

Explanation for Incorrect Answer D :

Choice (D) is not correct. This choice,

increase in sales for Company

decrease in sales for Company

er ed

was

for the and Company The graph above shows the monthly sales for Company What was the largest one-month decrease in sales for Company second half of during this period of time?

Therefore, the largest one-

was the difference in sales

was the largest one-month

The question asks for the largest one-month

was the largest one-month

The question asks for the largest one-month

was the largest one-month

The question asks for the largest one-month

file://E:\\e4.htm

2006-11-12

The Official SAT Online Course

4/17

For parties, the number of cases of juice a caterer orders is directly proportional to cases for a party with the number of people attending. If the caterer orders people people attending, how many cases would she order for a party with attending?
(A)
(B)
(C)
(D)
(E)

ANSWERS

AND EXPLANATIONS

Explanation for Correct Answer E : represent the number of cases of juice the caterer Choice (E) is correct. Let

would order for a party with

people attending. Then

and

Explanation for Incorrect Answer A :

people is

eg

Explanation for Incorrect Answer B : represent the number of cases of juice the caterer Choice (B) is not correct. Let

would order for a party with

nR

Explanation for Incorrect Answer C :

for

Choice (C) is not correct. This choice is

not

is

te

Choice (A) is not correct. This choice is

re


and

but the correct number of cases for

people attending. Then

people is

Explanation for Incorrect Answer D : represent the number of cases of juice the caterer Choice (D) is not correct. Let

d
would order for a party with

but the correct number of cases

people attending. Then

and

not

Which of the following is an element of both the set of positive odd integers and the set of prime numbers?

(A)
(B)
(C)
(D)
(E)

file://E:\\e4.htm

2006-11-12

The Official SAT Online Course

5/17

ANSWERS

AND EXPLANATIONS
is greater than

Explanation for Correct Answer B : is odd. In addition, Choice (B) is correct. The integer

and

has no positive integer factors other than itself and

so

is a prime number.

is an element of both the set of positive odd integers and the set of Therefore, prime numbers.

Explanation for Incorrect Answer A : is even, not odd. Choice (A) is not correct. The number

Explanation for Incorrect Answer C : Choice (C) is not correct. Since

the number

is not prime.

Explanation for Incorrect Answer D : Choice (D) is not correct. The number

is even, not odd. In addition, since

the number

is not prime.

Explanation for Incorrect Answer E : Choice (E) is not correct. Since

the number

(B)
(C)
(D)
(E)

nR

ANSWERS

Explanation for Correct Answer C : Choice (C) is correct. The length of the garden is

more than the width, it follows that the width of the garden is

eg

AND EXPLANATIONS
feet. Since the length is

is t

not

(A)

er

feet), not

feet more than its width. If the length of the The length of a rectangular garden is feet, what is the area of the garden in square feet? garden is

ed

is not prime.

feet

feet. Therefore,

the area of the garden, in square feet, is

Explanation for Incorrect Answer A : Choice (A) is not correct. The length of the garden is

feet. The width of the

garden is

feet less than the length, or

feet. Therefore, the area of the

garden, in square feet, is

Explanation for Incorrect Answer B : Choice (B) is not correct. The width of the garden is

feet, but the length is

feet. Therefore, the area of the garden, in square feet, is

not

Explanation for Incorrect Answer D : Choice (D) is not correct. The length of the garden is

feet, but the width is

feet. Therefore, the area of the garden, in square feet, is

not

Explanation for Incorrect Answer E : Choice (E) is not correct. The length of the garden is

feet. The width of the

garden is

feet less than the length (

feet more (

feet).

Therefore, the area of the garden, in square feet, is

not

file://E:\\e4.htm

2006-11-12

The Official SAT Online Course

6/17

On the number line above, the tick marks are equally spaced. What is the value of

(A)
(B)
(C)
(D)
(E)

ANSWERS

AND EXPLANATIONS

equal subintervals, the distance between two adjacent tick marks is

Explanation for Incorrect Answer A : Choice (A) is not correct. The value of

eg is t

er

Explanation for Correct Answer E : From and is the distance between Choice (E) is correct. The value of the figure, we can see that this is the same as the distance between two adjacent into to tick marks. Since the equally spaced tick marks divide the interval from

ed

is the distance between two adjacent

tick marks. Since the equally spaced tick marks divide the interval from

to

into

not

nR
not

equal subintervals, the distance between two adjacent tick marks is

Explanation for Incorrect Answer B : Choice (B) is not correct. The value of

is the distance between two adjacent

tick marks. Since the equally spaced tick marks divide the interval from

to

into

equal subintervals, the distance between two adjacent tick marks is

Explanation for Incorrect Answer C : Choice (C) is not correct. This choice is the value of the value of

but the question asks for

Explanation for Incorrect Answer D : Choice (D) is not correct. The value of

is the distance between two adjacent

tick marks. Since the equally spaced tick marks divide the interval from

to

into

equal subintervals, the distance between two adjacent tick marks is

not

This choice could be the result of miscounting the subintervals between

and

Jamal has some coins in his pocket. Some of these coins are quarters, and none of

file://E:\\e4.htm

2006-11-12

The Official SAT Online Course

7/17

the quarters in his pocket are dated earlier than be true?


(A)

Which of the following must

None of the coins in Jamals pocket are dated earlier than

(B)

Some of the coins in Jamals pocket are dated earlier than

(C) Some of the coins in Jamals pocket are dated

or later.

(D) Most of the coins in Jamals pocket are either quarters or dated earlier than

(E)

Most of the coins in Jamals pocket are not quarters.

ANSWERS

AND EXPLANATIONS

Explanation for Correct Answer C : Choice (C) is correct. Jamal has some quarters in his pocket, and none of them are Therefore, all of the quarters in his pocket must be dated dated earlier than or later. This means that the statement Some of the coins in Jamals or later MUST be true. pocket are dated

but we have no information about the other coins. Therefore, earlier than the statement None of the coins in Jamals pocket are dated earlier than is NOT necessarily true.

Explanation for Incorrect Answer B : Choice (B) is not correct. We know that none of the quarters in Jamals pocket are

dated earlier than

10

nR

Explanation for Incorrect Answer D : Choice (D) is not correct. We know that Jamal may have some coins in his pocket that are not quarters, but we do not know how many coins are quarters and how many coins are not quarters. We also do not know anything about the dates of the coins that are not quarters. Therefore, the statement Most of the coins in may or may not Jamals pocket are either quarters or dated earlier than be true.

Explanation for Incorrect Answer E : Choice (E) is not correct. Although we know that Jamal has some quarters in his pocket and possibly some coins that are not quarters, we do not know how many of the coins are quarters and how many are not quarters. Therefore, the statement Most of the coins in Jamals pocket are not quarters is NOT necessarily true.

eg i

either. Therefore, the statement Some of the coins in Jamals pocket are

st

and we know that Jamal may have some coins in his dated earlier than pocket that are not quarters. We know that none of the quarters are dated earlier and it is possible than none of the other coins are dated earlier than than

is NOT necessarily true.

er

ed

Explanation for Incorrect Answer A : Choice (A) is not correct. It may be true that some of the coins in Jamals pocket are not quarters. We know that none of the quarters in Jamals pocket are dated

The circumference of the circle with center of the shaded region?

shown above is

What is the area

(A)

file://E:\\e4.htm

2006-11-12

The Official SAT Online Course

8/17

(B)
(C)

(D)
(E)

ANSWERS

AND EXPLANATIONS
times its diameter. Thus, Hence, the radius of the

Explanation for Correct Answer B : Choice (B) is correct. The circumference of a circle is has diameter the circle, which has circumference

circle is

and the area of the entire circle is

Since the two diameters

in the figure are perpendicular, the shaded region is

of the circle.

Therefore, the area of the shaded region is

Explanation for Incorrect Answer A :

Choice (A) is not correct.

is one quarter of the circumference, but the question

asks for the area of one quarter of the circle.

Explanation for Incorrect Answer D :

Choice (D) is not correct.

nR
circle has area

for the area of one quarter of the circle.

Explanation for Incorrect Answer E : Choice (E) is not correct. The shaded area is one quarter of the circle, but since the

11

eg is

Explanation for Incorrect Answer C : is the radius of the circle, but the question asks for the Choice (C) is not correct. area of one quarter of the circle.

is one quarter of the diameter, but the question asks

the area of one quarter of the circle is

te r

Thus, if

and If be true?

are positive integers, and if

(A)
(B)
(C)
(D)
(E)

ANSWERS

AND EXPLANATIONS
then at least one of the

Explanation for Correct Answer D : Choice (D) is correct. If the product of two numbers is

ed

which of the following must

numbers must be equal to

then either

or

is a positive integer, (or both). Since Therefore, it must be true that

It follows that

file://E:\\e4.htm

2006-11-12

The Official SAT Online Course

9/17

Explanation for Incorrect Answer A : and Choice (A) is not correct. If

then

and

are positive

integers such that

yet it is not true that

Explanation for Incorrect Answer B : and Choice (B) is not correct. If

then

and

are positive

integers such that

yet it is not true that

Explanation for Incorrect Answer C : and Choice (C) is not correct. If

then

and

are positive

integers such that

yet it is not true that

Explanation for Incorrect Answer E : and Choice (E) is not correct. If

then

and

are positive

integers such that

yet it is not true that

12

is divided into three nonoverlapping In the figure above, regular pentagon triangles. Which of the following is true about the three triangles?
(A)

They have equal areas.

(B)

They have equal perimeters.

(C) They are similar.

and

nR
(E)

(D) They are isosceles.

ANSWERS

They each have at least one angle of measure

Explanation for Correct Answer D : Choice (D) is correct. A regular pentagon has five sides of equal length, so

eg

AND EXPLANATIONS

is

te

re d

so

are each isosceles, and the two sides of equal length in

have

the same length as the two sides of equal length in

The five angles of a

regular pentagon are of equal measure, so

has the same measure as

Thus,

and

are congruent. Therefore,

and

is also isosceles.

Explanation for Incorrect Answer A : and Choice (A) is not correct.

have the same area, but the area of

is greater.

is the largest angle of

is the longest

side of

Let

on

be the point such that

has the same length as

a side of the pentagon. It can be shown that Therefore,

is congruent to

Explanation for Incorrect Answer B : and Choice (B) is not correct.

are of equal perimeter, but the

perimeter of

is greater.

is the largest angle of

so

is

the longest side of

The perimeter of

is

file://E:\\e4.htm

2006-11-12

The Official SAT Online Course

10/17

whereas the perimeter of

is

Explanation for Incorrect Answer C : and Choice (C) is not correct.

are similar; in fact, they are

congruent. But

is not similar to the other two triangles. The angles of

are of measure

and

whereas the other two triangles have

angles of measure

and

Explanation for Incorrect Answer E : Choice (E) is not correct. None of the three triangles has any angles of measure

13

If
I.

or

which of the following must be true?

II.

III.

(A)

III only

(C) I and III only

(D) II and III only

(E)

I, II, and III

nR eg
ANSWERS
Choice (B) is correct. If

Explanation for Correct Answer B :

definition of absolute value, it is also true that

of

and II must both be true.

AND EXPLANATIONS
or

is te

(B)

I and II only

re

then

d
However, statement III could be false. The value of

In addition, by the

Therefore, statements I

could be greater than

but it could be less. For example, if

then

but

Explanation for Incorrect Answer A :

Choice (A) is not correct. It is not true that

must be greater than

The value

could be greater than

but it could be less. For example, if

then

but

Explanation for Incorrect Answer C : Choice (C) is not correct. Statement I must be true, but statement II must also be

true, and statement III could be false. The value of

could be greater than

but it could be less. For example, if

then

but

Explanation for Incorrect Answer D : Choice (D) is not correct. Statement II must be true, but statement I must also be

true, and statement III could be false. The value of

could be greater than

but it could be less. For example, if

then

but

file://E:\\e4.htm

2006-11-12

The Official SAT Online Course

11/17

Explanation for Incorrect Answer E : Choice (E) is not correct. Statements I and II must be true, but statement III could

be false. The value of

could be greater than

but it could be less. For

example, if

then

but

14

In the figure above,

and

If

is an integer, what is the least

possible value of
(A)
(B)
(C)
(D)
(E)

ANSWERS

AND EXPLANATIONS

Explanation for Correct Answer E : Choice (E) is correct. The sum of the measures of the three angles of a triangle is Substituting it follows that Since so

for

gives

equivalent to

nR eg

integer and

least possible value of

Explanation for Incorrect Answer A : were equal to Choice (A) is not correct. If

The value of

so

it must be true that

Since

is

te

Therefore,

re

then


this equation is

it follows that

d
which implies that

Since

is an

is also an integer. Therefore, the

is the least integer greater than

which is

would equal

would then be

However,

cannot be

cannot equal

Explanation for Incorrect Answer B : were equal to Choice (B) is not correct. If

then

would equal

The value of

would then be

However,

so

cannot be

Therefore,

cannot equal

Explanation for Incorrect Answer C : were equal to Choice (C) is not correct. If

then

would equal

The value of

would then be

However,

so

cannot be

Therefore,

cannot equal

Explanation for Incorrect Answer D : were equal to Choice (D) is not correct. If

then

would equal

The value of

would then be

However,

so

cannot be

Therefore,

cannot equal

15

For all positive integers

and

let

be defined by

If

and

are positive integers, which of the following CANNOT be the value of

file://E:\\e4.htm

2006-11-12

The Official SAT Online Course

12/17

(A)
(B)
(C)
(D)
(E)

ANSWERS

AND EXPLANATIONS
cannot be equal to

Explanation for Correct Answer B :

Choice (B) is correct. The value of

If

then

There are no positive integers

and

such that

There are nonnegative integers that work (for

example, integers.

and

), but the question states that

and

are positive

Explanation for Incorrect Answer A :

Choice (A) is not correct. If

then

Explanation for Incorrect Answer C :

re d

then

asks which number CANNOT be the value of

Choice (C) is not correct. If

and

The question

The

question asks which number CANNOT be the value of

Explanation for Incorrect Answer D :

question asks which number CANNOT be the value of

nR eg

Explanation for Incorrect Answer E :

Choice (E) is not correct. If

question asks which number CANNOT be the value of

16

and

is

Choice (D) is not correct. If

then

te
and

The

then

The

The functions

are defined above. For how many values of

is it true that

(A)

None

(B)

One

(C) Two

(D) Three

(E)

More than three

ANSWERS

AND EXPLANATIONS
then

Explanation for Correct Answer A :

Choice (A) is correct. If

This implies

that

so that

There are no real numbers

that satisfy

this equation.

file://E:\\e4.htm

2006-11-12

The Official SAT Online Course

13/17

Another way to see that

has no solutions is to graph

and

in the

-plane and observe that these two graphs do not intersect.

Therefore, there are no values

for which

Explanation for Incorrect Answer B :

Choice (B) is not correct. If there were one value of

for which

were

true, then there would be a real solution of the equation

However, this

equation has no real solutions.

Explanation for Incorrect Answer C :

Choice (C) is not correct. If there were two values of

for which

were

true, then there would be two real solutions of the equation

However,

this equation has no real solutions.

Explanation for Incorrect Answer D :

Choice (D) is not correct. If there were three values of

for which

were true, then there would be at least one real solution of the equation

However, this equation has no real solutions. (In any case, the number

of solutions of a quadratic equation cannot be greater than two.)

Explanation for Incorrect Answer E : Choice (E) is not correct. If there were more than three values of

equation

However, this equation has no real solutions. (In any case,

the number of solutions of a quadratic equation cannot be greater than two.)

nR

eg

The function

is
defined for

17

te
for which

following gives all values of

(A)

(B)

(C)

(D)

and

(E)

and

ANSWERS

AND EXPLANATIONS
for which

Explanation for Correct Answer D :

Choice (D) is correct. The values of

re

were true, then there would be at least one real solution of the

for which

is graphed above. Which of the

is positive?

is positive are exactly those

file://E:\\e4.htm

2006-11-12

The Official SAT Online Course

14/17

values on the

-axis for which the graph of

lies above the

-axis. The graph

of

lies above the

-axis for

and

Therefore,

and

are all the values of

for which

is positive.

Explanation for Incorrect Answer A :

Choice (A) is not correct. For example,

is positive for

which is not in

the interval

(Also,

is not positive for many values of

in the

interval

Explanation for Incorrect Answer B :

Choice (B) is not correct. It is true that

is positive for all values of

such

that

but

is also positive for all values of

such that

Explanation for Incorrect Answer C : Choice (C) is not correct. For example,

is in the interval

yet

is not positive. (Also,

is positive for all values of

in the interval

Explanation for Incorrect Answer E :

Choice (E) is not correct.

is negative exactly for those values of

of

for which

is positive.

18

er ed

is

intervals

and

However, the question asks for all the values


If line

in the

line

in the

-axis, what is an equation of line

(B)
(C)

nR

(D)

(E)

ANSWERS

Explanation for Correct Answer A :

eg

(A)

AND EXPLANATIONS
is a point on line

is t

is reflected in the

In the

-plane, an equation of line

is the reflection of

Choice (A) is correct. If

then it is true that

When line

-axis, the point

is reflected

to the point

which is on line

Since

the coordinates of

point

satisfy the relationship

Therefore, an

equation of line

is

Lines

and

are graphed in the figure below.

file://E:\\e4.htm

2006-11-12

The Official SAT Online Course

15/17

Explanation for Incorrect Answer B : for is found by substituting Choice (B) is not correct. An equation of line This incorrect answer may be the result of mistakenly in the equation

multiplying the entire right-hand side of

by

Explanation for Incorrect Answer C : for is found by substituting Choice (C) is not correct. An equation of line This incorrect answer may be the result of mistakenly in the equation

substituting

for

instead of

for

in the equation

Explanation for Incorrect Answer D :

Choice (D) is not correct.

is an equation of a line perpendicular to line

not of the line that is the reflection of line

in the

-axis.

Explanation for Incorrect Answer E :

Choice (E) is not correct.

is an equation of a line perpendicular to line

not of the line that is the reflection of line

in the

-axis.

19

For which of the following values for solution?

(A)
(B)

(C)

(D)

nR
ANSWERS

(E)

Explanation for Correct Answer A : then the system of equations becomes Choice (A) is correct. If

eg i

AND EXPLANATIONS

st er

then

will the system of equations above have no

ed

This system has no solution: if

must

be equal to

not

Explanation for Incorrect Answer B : then the system of equations becomes Choice (B) is not correct. If

This system has the solution

and

Explanation for Incorrect Answer C : then the system of equations becomes Choice (C) is not correct. If

file://E:\\e4.htm

2006-11-12

The Official SAT Online Course

16/17

This system has the solution

and

Explanation for Incorrect Answer D : then the system of equations becomes Choice (D) is not correct. If

This system has the solution

and

Explanation for Incorrect Answer E : then the system of equations becomes Choice (E) is not correct. If

This system has the solution

and

20

I.

nR

II.

customers purchased from a The table above shows the number of items hardware store over a -hour period. Which of the following can be determined from the information in the table?
The average (arithmetic mean) number of items purchased per customer

The median number of items purchased per customer

III.

The mode of the number of items purchased per customer

(A)

None

(B)

I and II only

(C) I and III only

(D) II and III only

(E)

I, II, and III

ANSWERS

Explanation for Correct Answer D : Choice (D) is correct. To determine the average number of items purchased per customer, it is necessary to know the total number of items purchased. This total of the to find the average. However, for can then be divided by

eg

AND EXPLANATIONS

is

te

re

d
file://E:\\e4.htm

2006-11-12

The Official SAT Online Course


customers, the exact number of items purchased is not given in the table. Fewer or as few as customers bought as many as could mean these than items. Therefore, I cannot be determined.

17/17

The median number of items purchased per customer is the average of the number customers when all customers are rank and of items purchased by the

and ordered by the number of items purchased. The items, so the median is rank ordering each purchased determined.

customers in this Therefore, II can be

The mode is the number of items purchased by the greatest number of customers. items each, and no larger customers purchased because The mode is Therefore, III can be group purchased a number of items different from determined.

Summarizing, only II and III can be determined from the information in the table. Therefore, choice (D) is the correct answer.

Explanation for Incorrect Answer A : Choice (A) is not correct. The median number of items purchased per customer and the mode of the number of items purchased per customer can be determined from the information in the table.

nR

eg

is

Explanation for Incorrect Answer E : Choice (E) is not correct. The median and the mode can be determined from the information in the table, but the mean cannot.

te

Explanation for Incorrect Answer C : Choice (C) is not correct. The mode can be determined from the information in the table, but the mean cannot.

re d

Privacy Policy

Explanation for Incorrect Answer B : Choice (B) is not correct. The median can be determined from the information in the table, but the mean cannot.

Back to Score Report

Copyright 2006 The College Board. All rights reserved.

Terms of Use

Contact Us

file://E:\\e4.htm

2006-11-12

The Official SAT Online Course

1/24

Help | Profile | My Organizer | My Bookmarks | Logout

Answers and Explanations

Test Sections

Back to Score Report

Section 1

View Answers and Explanations


Online - Practice Test #5

Section 2

Section 3

Section 4

Section 5

Section 6

The border between the two properties, never ------- by legal means, had long been the subject of ------- between the antagonistic neighbors. (A) determined . . concord
(B)

Section 8

undermined . . hostility

Section 9

(C) verified . . consonance

Section 10

(D) quantified . . diversion

(E)

established . . disputation

Explanation for Incorrect Answer A : Choice (A) is incorrect. To determine means to settle or decide authoritatively. Concord is a state of agreement. If one were to insert these terms into the text, the sentence would read The border between the two properties, never determined by legal means, had long been the subject of concord between the antagonistic neighbors. Antagonistic neighbors, or neighbors who express hostility and opposition, would probably not have a state of agreement concerning a border that had not been "determined." On the contrary, an undetermined border would most likely cause arguments between antagonistic neighbors.

nR

Explanation for Incorrect Answer B : Choice (B) is incorrect. To undermine means to weaken or subvert. Hostility is an attitude of ill will and opposition. If one were to insert these terms into the text, the sentence would read The border between the two properties, never undermined by legal means, had long been the subject of hostility between the antagonistic neighbors. Although antagonistic neighbors might be hostile to one another, it is somewhat illogical to say that the property border had never been undermined, or weakened, by legal means.

Explanation for Incorrect Answer C : Choice (C) is incorrect. To verify means to establish formally. Consonance is a state of harmony and agreement. If one were to insert these terms into the text, the sentence would read The border between the two properties, never verified by legal means, had long been the subject of consonance between the antagonistic neighbors. Antagonistic neighbors, or neighbors who express hostility and opposition, would most likely not have consonance, or a state of harmony, concerning a border that had never been formally established. On the contrary, a border that has not been verified, or established, would probably cause arguments between antagonistic neighbors.

Explanation for Incorrect Answer D : Choice (D) is incorrect. To quantify means to measure or determine in terms of amount or magnitude. Distraction is something that draws attention away from something else. If one were to insert these terms into the text, the sentence would read The border between the two properties, never quantified by legal means, had long been the subject of distraction between the antagonistic neighbors.

eg

is t

Explanation for Correct Answer E : Choice (E) is correct. To establish means to agree to institute something permanently. Disputation is the act of arguing. If one were to insert these terms into the text, the sentence would read The border between the two properties, never established by legal means, had long been the subject of disputation between the antagonistic neighbors. It is logical to suggest that antagonistic neighbors, or neighbors who express hostility and opposition, might argue over a border that had never been "established."

er ed

ANSWERS

AND EXPLANATIONS

file://E:\\e5.htm

2006-11-12

The Official SAT Online Course

2/24

Although a property border might be a distraction for antagonistic neighbors, it is somewhat illogical to suggest that the border could be quantified, or determined in terms of amount or magnitude.

Even though Charlie was in apparently good health, the doctor prescribed for him some ------- medication due to his familial history of high blood pressure. (A) presumptive
(B)

predictive

(C) preliminary

(D) premeditated

(E)

preventative

ANSWERS

AND EXPLANATIONS

nR

Explanation for Incorrect Answer A : Choice (A) is incorrect. Presumptive means based on an assumption. If one were to insert this term into the text, the sentence would read Even though Charlie was in apparently good health, the doctor prescribed for him some presumptive medication due to his familial history of high blood pressure. While the doctor might have presumed that Charlie should be taking medication due to his familys history of high blood pressure, it does not make sense to say that the medication itself was presumptive, or based on an assumption that the doctor made about Charlies health.

Explanation for Incorrect Answer B : Choice (B) is incorrect. Predictive means making a prediction. If one were to insert this term into the text, the sentence would read Even though Charlie was in apparently good health, the doctor prescribed for him some predictive medication due to his familial history of high blood pressure. It does not make sense to describe medication as predictive. Medication cannot predict a patients health.

Explanation for Incorrect Answer C : Choice (C) is incorrect. Preliminary means coming before something else. If one were to insert this term into the text, the sentence would read Even though Charlie was in apparently good health, the doctor prescribed for him some preliminary medication due to his familial history of high blood pressure. The phrase Even though sets up a contrast between Charlies apparent good health and his familys history of high blood pressure. The missing term should describe a medication that the doctor would prescribe for Charlie even in the absence of symptoms. For medication to be preliminary, or coming before something else, the doctor would have to know that Charlie would later need other medications. Charlie may or may not develop high blood pressure, so the medication would not necessarily be preliminary.

Explanation for Incorrect Answer D : Choice (D) is incorrect. Premeditated means considered beforehand. If one were to insert this term into the text, the sentence would read Even though Charlie was in apparently good health, the doctor prescribed for him some premeditated medication due to his familial history of high blood pressure. While the doctors decision to prescribe medication for Charlie may have been premeditated, or considered beforehand, it does not make sense to say that the medication itself was premeditated.

eg

is

te re d

Explanation for Correct Answer E : Choice (E) is correct. Preventative means preventing something. If one were to insert this term into the text, the sentence would read Even though Charlie was in apparently good health, the doctor prescribed for him some preventative medication due to his familial history of high blood pressure. The phrase Even though sets up a contrast between Charlies apparent good health and his familys history of high blood pressure. The missing term should describe a medication that the doctor would prescribe for Charlie even in the absence of symptoms. It would make sense for the doctor to prescribe preventative medication to help Charlie avoid developing the high blood pressure that runs in his family.

file://E:\\e5.htm

2006-11-12

The Official SAT Online Course

3/24

Though earlier anatomists had touched on the idea, Paul Broca was the first to --------- fully the modern notion that specific behaviors are controlled by particular areas of the human brain. (A) articulate
(B)

derogate

(C) represent

(D) refute

(E)

iterate

ANSWERS

AND EXPLANATIONS

Explanation for Correct Answer A : Choice (A) is correct. To articulate means to express a concept. If one were to insert this term into the text, the sentence would read Though earlier anatomists had touched on the idea, Paul Broca was the first to articulate fully the modern notion that specific behaviors are controlled by particular areas of the human brain. While earlier anatomists had touched on the idea that certain behaviors are controlled by particular areas of the brain, Paul Broca was the first to fully articulate, or express, the concept.

nR

(A)

Explanation for Incorrect Answer C : Choice (C) is incorrect. To represent means to serve as a sign or symbol of something. If one were to insert this term into the text, the sentence would read Though earlier anatomists had touched on the idea, Paul Broca was the first to represent fully the modern notion that specific behaviors are controlled by particular areas of the human brain. The fact that earlier anatomists touched on the notion that certain behaviors are controlled by particular areas of the brain is not logically connected to the idea that Paul Broca represented, or was a symbol of, that notion.

Explanation for Incorrect Answer D : Choice (D) is incorrect. To refute means to prove wrong. If one were to insert this term into the text, the sentence would read Though earlier anatomists had touched on the idea, Paul Broca was the first to refute fully the modern notion that specific behaviors are controlled by particular areas of the human brain. Nothing in the sentence suggests that earlier anatomists had partially refuted, or proved wrong, the idea that certain behaviors are controlled by particular areas of the brain, so it would not make sense to say that Paul Broca fully refuted the idea.

Explanation for Incorrect Answer E : Choice (E) is incorrect. To iterate means to say or do something repeatedly. If one were to insert this term into the text, the sentence would read Though earlier anatomists had touched on the idea, Paul Broca was the first to iterate fully the modern notion that specific behaviors are controlled by particular areas of the human brain. Although Paul Broca could have been the first to fully state the idea that specific areas of the human brain control specific behaviors, nothing in the sentence suggests that he stated this idea repeatedly.

Designed as a gathering place, the new student lounge was appropriately ------- with tables, chairs, and even sofas where groups could assemble comfortably.

indicated

(B)

appointed

(C) denuded

eg

is t

Explanation for Incorrect Answer B : Choice (B) is incorrect. To derogate means to belittle or to cause to seem inferior. If one were to insert this term into the text, the sentence would read Though earlier anatomists had touched on the idea, Paul Broca was the first to derogate fully the modern notion that specific behaviors are controlled by particular areas of the human brain. Nothing in the sentence suggests that either the earlier anatomists or Paul Broca in any way derogated, or belittled, the idea that certain behaviors are controlled by particular areas of the brain.

er ed

file://E:\\e5.htm

2006-11-12

The Official SAT Online Course

4/24

(D) conflated

(E)

venerated

ANSWERS

AND EXPLANATIONS

Explanation for Correct Answer B : Choice (B) is correct. To appoint means to provide with furnishings or equipment. If one were to insert this term into the text, the sentence would read Designed as a gathering place, the new student lounge was appropriately appointed with tables, chairs, and even sofas where groups could assemble comfortably. A student lounge that is intended to be a gathering place would most likely be appointed, or furnished, with tables, chairs, and sofas that would allow groups to gather together in comfortable surroundings.

Explanation for Incorrect Answer A : Choice (A) is incorrect. To indicate means to show or make known. If one were to insert this term into the text, the sentence would read Designed as a gathering place, the new student lounge was appropriately indicated with tables, chairs, and even sofas where groups could assemble comfortably. It does not make sense to say that the lounge was indicated, or shown, by its tables, chairs, and sofas.

Explanation for Incorrect Answer E : Choice (E) is incorrect. To venerate means to honor or regard something with respect. If one were to insert this term into the text, the sentence would read Designed as a gathering place, the new student lounge was appropriately venerated with tables, chairs, and even sofas where groups could assemble comfortably. Although a gathering place might have tables, chairs, and sofas, it is not clear how these pieces of furniture would cause the student lounge to be venerated, or regarded with respect.

nR

(A)

Named in honor of the schools founder, the Richard Brownstone Community Service Fellowship is one of the highest awards ------- Brownstone School graduates.

conscripted to

(B)

redeemed for

(C) conferred on

(D) relegated to

(E)

deprived of

ANSWERS

Explanation for Correct Answer C : Choice (C) is correct. To confer means to give to or to bestow upon. If one were to insert this term into the text, the sentence would read Named in honor of the schools founder, the Richard Brownstone Community Service Fellowship is one of the highest awards conferred on Brownstone School graduates. It makes sense to say that an award would be conferred on, or given to, graduates of Brownstone School.

eg

Explanation for Incorrect Answer D : Choice (D) is incorrect. To conflate means to join or combine. If one were to insert this term into the text, the sentence would read Designed as a gathering place, the new student lounge was appropriately conflated with tables, chairs, and even sofas where groups could assemble comfortably. Although a gathering place might have tables, chairs, and sofas, it does not make sense to say that the student lounge was conflated, or joined, with these pieces of furniture.

AND EXPLANATIONS

is t

er

Explanation for Incorrect Answer C : Choice (C) is incorrect. To denude means to make bare. If one were to insert this term into the text, the sentence would read Designed as a gathering place, the new student lounge was appropriately denuded with tables, chairs, and even sofas where groups could assemble comfortably. The term denuded does not make sense in this context. If the student lounge was denuded, or made bare, it would not have any furniture.

ed

file://E:\\e5.htm

2006-11-12

The Official SAT Online Course

5/24

Explanation for Incorrect Answer A : Choice (A) is incorrect. To conscript means to enroll into service. If one were to insert this term into the text, the sentence would read Named in honor of the schools founder, the Richard Brownstone Community Service Fellowship is one of the highest awards conscripted to Brownstone School graduates. The term conscripted does not make sense in this context. It is illogical to suggest that an award could be enrolled into service.

Explanation for Incorrect Answer B : Choice (B) is incorrect. To redeem means to exchange for something of value. If one were to insert this term into the text, the sentence would read Named in honor of the schools founder, the Richard Brownstone Community Service Fellowship is one of the highest awards redeemed for Brownstone School graduates. The term redeemed does not make sense in this context. It does not make sense to say that an award can be redeemed, or exchanged, for graduates of the Brownstone School.

The following passage on monarch butterflies was written in 2000 by a naturalist and insect specialist.

The migration and the winter gathering of monarch butterflies are among the most spectacular of all natural Line phenomena, unique in the insect world. 5 Lincoln Brower wrote of his feeling on a warm March morning as he watched tens of thousands of these butterflies explode from their resting places on the trees at an overwintering site in Mexico: Flying 10 against the azure sky and past the green boughs of the oyamels, this myriad of dancing embers reinforced my earlier conclusion that this spectacle is a treasure comparable to the finest works of art that 15 our world culture has produced over the past 4000 years. But even as I write this paragraph, the winter gathering places of the monarch are being destroyed by illegal loggingindeed, all of the oyamel 20 forests in Mexico are threatened by legal and illegal logging. If the logging

nR

eg

is

te re

Explanation for Incorrect Answer E : Choice (E) is incorrect. To deprive means to take away or withhold. If one were to insert this term into the text, the sentence would read Named in honor of the schools founder, the Richard Brownstone Community Service Fellowship is one of the highest awards deprived of Brownstone School graduates. It does not make sense to suggest that the award is deprived of, or withheld from, graduates of Brownstone School.

Explanation for Incorrect Answer D : Choice (D) is incorrect. To relegate means to assign to a place of insignificance. If one were to insert this term into the text, the sentence would read Named in honor of the schools founder, the Richard Brownstone Community Service Fellowship is one of the highest awards relegated to Brownstone School graduates. It does not make sense to suggest that presenting graduates of Brownstone School with an award is an action of assigning something to a place of insignificance. If some of the graduates are presented with one of the highest awards, it is illogical to describe either the award or the recipients as insignificant.

file://E:\\e5.htm

2006-11-12

The Official SAT Online Course

6/24

25

30

35

40

45

50

55

60

65

70

continues at its present rate, all of the overwintering sites in Mexico will be gone by the first decades of the twenty-first century. So desperate is the situation that the Union for the Conservation of Nature and Natural Resources has recognized the monarch migration as an endangered biological phenomenon and has designated it the first priority in their effort to conserve the butterflies of the world. All efforts to preserve the overwintering sites in Mexico have failed. In August of 1986, the Mexican government issued a proclamation designating these sites as ecological preserves. Five of the 12 known sites were to receive complete protection. Logging and agricultural development were to be prohibited in their core areas, a total area of only 17 square miles, and only limited logging was to be permitted in buffer zones surrounding the cores, a total of another 43 miles. The proclamation was largely ignored. One of the 5 protected sites has been clear-cut, some buffer zones have been more or less completely destroyed, and trees are being cut in all of the core areas. As Brower told me, guards that were appointed to protect the monarch colonies have not prevented illegal logging but have barred tourists, film crews, and scientists from witnessing logging activities. It is incomprehensible to me that a way cannot be found to protect a mere 60 square miles of land that are home to one of the worlds most spectacular biological phenomena. If the monarchs are to survive, the oyamel forests in which they spend the winter must remain intact. Even minor thinning of the core areas causes high mortality among the butterflies, because the canopy of the intact forest serves as a protective blanket and umbrella for them. Within a dense stand of trees, the temperature does not drop as low as it does elsewhere, enabling the monarchs to survive freezing weather under the blanket of trees. Thinning the trees puts holes in the umbrella that protects the

nR

eg

is

te r

ed

file://E:\\e5.htm

2006-11-12

The Official SAT Online Course

7/24

monarchs, letting them get wet during 75 winter storms. A wet butterfly loses its resistance to freezing and dies. Even a dry butterfly loses precious calories as its body heat radiates out to the cold night sky through holes in the canopy.
6

The quotation in lines 9-16 serves primarily as a


(A)

detailed explanation of the cultural significance of a place

(B)

personal observation about artistic awareness

(C) dramatic portrayal of an impressive event

(D) scientific account of a rare phenomenon

(E)

conclusive argument for the artistic importance of spectacle

ANSWERS

AND EXPLANATIONS

nR eg

(A)

Explanation for Incorrect Answer A : Choice (A) is incorrect. The quotation vividly describes the winter gathering of monarchs in Mexico. Although Lincoln Brower does claim that the event is a treasure comparable to the finest works of art that our world culture has produced, he does not claim that the winter gathering place itself has any cultural significance.

Explanation for Incorrect Answer B : Choice (B) is incorrect. The quotation does recount a personal observation of Lincoln Browers, but the observation is about how spectacular the winter gathering of monarchs is, not about artistic awareness in general.

Explanation for Incorrect Answer D : Choice (D) is incorrect. The author of the passage does claim that the winter gathering of monarchs is a rare phenomenon, unique in the insect world. But the quotation is a dramatic portrayal using vivid imagery, not a scientific account of the event.

Explanation for Incorrect Answer E : Choice (E) is incorrect. Although the quotation by Lincoln Brower does state that the winter gathering of monarchs is a spectacle comparable to the finest works of art our world culture has produced, Brower does not develop a conclusive argument for its artistic importance. He simply thinks it is an extremely beautiful natural event.

The author views the efforts cited in line 33 as

understandably futile

(B)

necessarily limited

(C) scientifically misguided

(D) largely undesirable

(E)

unjustifiably ineffective

ANSWERS

AND EXPLANATIONS

is

te

re

Explanation for Correct Answer C : Choice (C) is correct. The author begins the passage by stating that the migrations and winter gathering of monarch butterflies are among the most spectacular of all natural phenomena. To support this statement powerfully, the author uses Lincoln Browers vivid and dramatic description of his personal viewing of the monarchs. Browers portrayal of the monarchs as this myriad of dancing embers flying against the azure sky gives the reader the sense that the winter gathering of monarchs is an impressive event.

file://E:\\e5.htm

2006-11-12

The Official SAT Online Course

8/24

Explanation for Correct Answer E : Choice (E) is correct. The efforts in line 33 refer to failed attempts to preserve the overwintering sites [of monarchs] in Mexico from the destruction brought on by unchecked legal and illegal logging. The author clearly believes that these efforts have been ineffective, reporting that one of the five protected sites has been clear-cut, some buffer zones have been more or less completely destroyed, and trees are being cut down in all of the core areas. The author openly states that such ineffectiveness is unjustifiable: It is incomprehensible to me that a way cannot be found to protect a mere 60 square miles of land that are home to one of the worlds most spectacular biological phenomena.

Explanation for Incorrect Answer A : Choice (A) is incorrect. The efforts in line 33 refer to failed attempts to preserve the overwintering sites [of monarchs] in Mexico from the destruction brought on by unchecked legal and illegal logging. While the author clearly believes that these efforts have been futile, he or she would not agree that this futility is understandable. The author cannot understand this futility: It is incomprehensible to me that a way cannot be found to protect a mere 60 square miles of land that are home to one of the worlds most spectacular biological phenomena.

Explanation for Incorrect Answer C : Choice (C) is incorrect. The efforts in line 33 refer to failed attempts to preserve the overwintering sites [of monarchs] in Mexico from the destruction brought on by unchecked legal and illegal logging. The author does not comment on the scientific foundation underlying the efforts to protect the sites.

nR

(A)

Explanation for Incorrect Answer D : Choice (D) is incorrect. The efforts in line 33 refer to failed attempts to preserve the overwintering sites [of monarchs] in Mexico from the destruction brought on by unchecked legal and illegal logging. The author argues that the efforts have been ineffective, not undesirable. On the contrary, the author clearly desires stronger efforts to protect the sites: It is incomprehensible to me that a way cannot be found to protect a mere 60 square miles of land that are home to one of the worlds most spectacular biological phenomena.

The third paragraph is best described as

an account of a natural struggle for survival

(B)

a comparison between two types of environments

(C) a description of a disruption in an ecological system

(D) a demonstration of successful efforts to preserve an environment

(E)

a guideline for opposing the destruction of a crucial habitat

ANSWERS

Explanation for Correct Answer C : Choice (C) is correct. The third paragraph describes the effects that destroying trees in the oyamel forest has on monarch butterflies. When trees are cut down, holes are created in the protective canopy that usually keeps the monarchs warm and dry. When the monarchs become cold and wet they often freeze to death. Such events are a disruption in the natural oyamel forest ecological system caused by the unchecked logging.

eg

AND EXPLANATIONS

is te

re

Explanation for Incorrect Answer B : Choice (B) is incorrect. The efforts in line 33 refer to failed attempts to preserve the overwintering sites [of monarchs] in Mexico from the destruction brought on by unchecked legal and illegal logging. The author might agree that the efforts have been limited, but not that this limitation is necessary. The author actually believes that it should be rather easy to protect the land: It is incomprehensible to me that a way cannot be found to protect a mere 60 square miles of land that are home to one of the worlds most spectacular biological phenomena.

file://E:\\e5.htm

2006-11-12

The Official SAT Online Course

9/24

Explanation for Incorrect Answer A : Choice (A) is incorrect. The description of the monarchs dying in the cold, wet night is certainly a struggle for survival. The monarchs struggle, however, is not natural; it is caused by unchecked human loggers.

Explanation for Incorrect Answer B : Choice (B) is incorrect. The third paragraph describes only one environment: the oyamel forest that is being destroyed by loggers.

Explanation for Incorrect Answer D : Choice (D) is correct. The third paragraph does not describe successful efforts to preserve an environment. It describes the effects that the destruction of an environmentthe oyamel foresthas on the wintering monarchs.

Explanation for Incorrect Answer E : Choice (E) is incorrect. The paragraph offers no guideline for opposing the destruction of the oyamel forest. It simply describes the effects that the destruction has on the wintering monarchs.

The tone of the passage could best be characterized as


(A)

indifferent

(B)

hostile

(C) concerned

ANSWERS

AND EXPLANATIONS

In this excerpt from a British novel published in 1938, a woman describes staying with her employer at a fashionable hotel in the resort city of Monte Carlo.

nR

Explanation for Incorrect Answer A : Choice (A) is incorrect. The author is not at all indifferent to the plight of the monarch butterfly. The author is very concerned with preserving the wintering sites of the monarch, explicitly stating: It is incomprehensible to me that a way cannot be found to protect a mere 60 square miles of land that are home to one of the worlds most spectacular biological phenomena.

Explanation for Incorrect Answer B : Choice (B) is incorrect. The passage does not exhibit a hostile tone. Mainly the author is concerned about the plight of the monarch butterfly. The author is critical of the failed efforts to preserve the overwintering sites, but the tone is troubled, not hostile.

Explanation for Incorrect Answer D : Choice (D) is incorrect. The tone of passage is concerned, not bewildered, or confused. The author fully understands that the monarchs are in danger of losing their overwintering habitat to logging.

Explanation for Incorrect Answer E : Choice (E) is incorrect. The tone of the passage is concerned, not complimentary. The author does not compliment the efforts to preserve the monarchs overwintering sites, but rather criticizes them for their failure.

eg

Explanation for Correct Answer C : Choice (C) is correct. The passage exhibits a concerned tone. The author begins by describing the spectacular beauty of wintering monarchs. Then the author states that even as I write this paragraph, the winter gathering places of the monarch are being destroyed by illegal logging. The passage goes on to describe the failing attempts being made to preserve the overwintering sites and the dying monarchs struggle to survive in the damaged forests. Overall, the author demonstrates a deep concern for the survival of the monarchs.

is

te re

(E)

complimentary

(D) bewildered

file://E:\\e5.htm

2006-11-12

The Official SAT Online Course

10/24

I wonder what my life would be today, if Mrs. Van Hopper had not been a snob. Line Funny to think that the course of 5 my existence hung like a thread upon that quality of hers. Her curiosity was a disease, almost a mania. At first I had been shocked, wretchedly embarrassed when I watched people laugh behind her 10 back, leave a room hurriedly upon her entrance, or even vanish behind a Service door on the corridor upstairs. For many years now she had come to the hotel Cote dAzur, and, apart from bridge, her one 15 pastime, which was notorious by now in Monte Carlo, was to claim visitors of distinction as her friends had she but seen them once at the other end of the postoffice. Somehow she would manage to 20 introduce herself, and before her victim had scented danger she had proffered an invitation to her suite. Her method of attack was so downright and sudden that there was seldom opportunity to escape. 25 At the Cote dAzur she staked a claim upon a certain sofa in the lounge, midway between the reception hall and the passage to the restaurant, and she would have her coffee there after luncheon and 30 dinner, and all who came and went must pass her by. Sometimes she would employ me as a bait to draw her prey, and, hating my errand, I would be sent across the lounge with a verbal message, 35 the loan of a book or paper, the address of some shop or other, the sudden discovery of a mutual friend. It seemed as though notables must be fed to her,

nR

and though titles were preferred by her, 40 any face once seen in a social paper served as well. Names scattered in a gossip column, authors, artists, actors and their kind, even the mediocre ones, as long as she had learnt of them in print. 45 I can see her as though it were but yesterday, on that unforgettable afternoonnever mind how many years agowhen she sat on her favourite sofa in the lounge, debating her method of 50 attack. I could tell by her abrupt manner,

eg i

1

st e

re

and the way she tapped her lorgnette


file://E:\\e5.htm

d
2

2006-11-12

The Official SAT Online Course

11/24

55

60

65

70

75

80

85

against her teeth, that she was questing possibilities. I knew, too, when she had missed the sweet and rushed through dessert, and she had wished to finish luncheon before the new arrival and so install herself where he must pass. Suddenly she turned to me, her small eyes alight. Go upstairs quickly and find that letter from my nephew. You remember, the one written on his honeymoon, with the snapshot. Bring it down right away. I saw then that her plans were formed, and the nephew was to be the means of introduction. Not for the first time I resented the part that I must play in her schemes. Like a jugglers assistant I produced the props, then silent and attentive I waited on my cue. This newcomer would not welcome intrusion, I felt certain of that. In the little I had learnt of him at luncheon, a smattering of hearsay garnered by her ten months ago from the daily papers and stored in her memory for future use, I could imagine, in spite of my youth and inexperience of the world, that he would resent this sudden bursting in upon his solitude. Why he should have chosen to come to the Cote dAzur at Monte Carlo was not our concern, his problems were his own, and anyone but Mrs. Van Hopper would have understood. Tact was a quality unknown to her, discretion too, and because gossip was the breath of life to her this stranger must be served for her dissection.

1 Titles here refers to members of the European nobility.


2 Eyeglasses on the end of a short handle.

10

nR eg

(A)

The passage is narrated from the point of view of

an employee of the Cote dAzur hotel

(B)

an observer who is uninvolved in the action

(C) Mrs. Van Hopper

(D) a participant who is remembering the scene at a later time

(E)

a tourist who has just met Mrs. Van Hopper

ANSWERS

AND EXPLANATIONS

is te r

ed

file://E:\\e5.htm

2006-11-12

The Official SAT Online Course

12/24

Explanation for Correct Answer D : Choice (D) is correct. We know that the narrator is a participant because she narrates the action in the first personSometimes she [Mrs. Van Hopper] would employ me as a bait to draw her prey, for example. Further, there are clear signs in the passage that the narrator is recalling the action from the distance of some years: I can see her as though it were but yesterday; never mind how many years ago. Together, these clues tell the reader that the person narrating is someone who stayed with Mrs. Van Hopper years before at the Cote dAzur hotel and is remembering the action.

Explanation for Incorrect Answer A : Choice (A) is incorrect. It is clear from context that the setting is the Cote dAzur hotel, but it is also clear that the narrator works for Mrs. Van Hopper, not for the hotel: Mrs. Van Hopper might very well send a hotel employee on a personal errand as she does in paragraph 4, but no hotel employee could know as much about or spend as much time with Mrs. Van Hopper as does this narrator.

Explanation for Incorrect Answer B : Choice (B) is incorrect. Certainly the narrator is an observer of Mrs. Van Hoppers various strategies for trapping people, but we know at least as early as line 31 (Sometimes she would employ me) that the narrator is also involved in the action.

Explanation for Incorrect Answer E : Choice (E) is incorrect. The last paragraph of the passage does focus on someone who, willingly or not, is about to meet Mrs. Van Hopper, but the passage clearly is not narrated from this persons point of view.

11

The disease mentioned in line 7 is best described as


(A)

total embarrassment at another persons behavior

nR
(B)

a refusal to speak to anyone who is not wealthy

(C) an intense need to avoid public notice

(D) a violent tendency to assault strangers

(E)

a relentless drive to meet well-known people

ANSWERS

Explanation for Correct Answer E : Choice (E) is correct. In the second paragraph of the passage, the narrator says that Mrs. Van Hoppers curiosity was a disease. The narrator then sets about describing this curiosity, which turns out to be an abnormal interest in making the acquaintance of well-known people. The paragraph describes various of Mrs. Van Hoppers techniques for meeting celebrities she does not know and claiming them as her friendsbehavior that clearly embarrasses the narrator. The narrator says that this disease, or sickness, verges on mania, or an unreasonable enthusiasma relentless drive, in other words, to meet people who have been written about in gossip columns.

Explanation for Incorrect Answer A : Choice (A) is incorrect. While the narrator acknowledges being wretchedly embarrassed at Mrs. Van Hoppers importunate behavior with strangers, the disease referred to in line 7 is clearly attributed by the narrator to Mrs. Van Hopper, who shows no embarrassment at all in preying on celebrities.

Explanation for Incorrect Answer B : Choice (B) is incorrect. In the first paragraph, the narrator describes Mrs. Van

eg

AND EXPLANATIONS

is te r

ed

Explanation for Incorrect Answer C : Choice (C) is incorrect. It is unlikelythough not impossiblethat Mrs. Van Hopper would be speaking of herself in the third person. Further, when Mrs. Van Hopper engages directly with the narrator in paragraph 4, we can be fairly certain that a separate participant is involved.

file://E:\\e5.htm

2006-11-12

The Official SAT Online Course

13/24

Hopper as a "snob," and a refusal to speak to anyone who is not wealthy could be considered a specific kind of snobbery. But it is clear from context that the disease is not Mrs. Van Hoppers refusal to speak, but rather her insistence on speaking to the famous. While it can be inferred that most of those who frequent this expensive hotel are wealthy, it is celebrity rather than money that Mrs. Van Hopper is drawn to; this is her disease.

Explanation for Incorrect Answer C : Choice (C) is incorrect. Far from avoiding public notice, Mrs. Van Hopper seats herself daily in an area of the hotel where she is almost certain to be noticed, much to the embarrassment of the narrator.

Explanation for Incorrect Answer D : Choice (D) is incorrect. While Mrs. Van Hopper certainly does assault people in the figurative sense of approaching them without their permission, there is no indication that she is violent towards them, or even hostile. Rather, Mrs. Van Hoppers technique would more likely be directed towards ingratiating herself with the celebrities she stalks than towards harming them.

12

In context, employ (line 32) most nearly means


(A)

service

(B)

use

(C) attract

(E)

hire

ANSWERS

AND EXPLANATIONS

nR

Explanation for Incorrect Answer A : Choice (A) is incorrect. In this sentence, the narrator is describing being sent by Mrs. Van Hopper on useless errands whose sole purpose is to attract the attention of someone Mrs. Van Hopper wants to befriend. In this context, it makes little sense to say that the narrator is being serviced, or provided with a service, when in fact she is providing a humiliating service for Mrs. Van Hopper.

Explanation for Incorrect Answer C : Choice (C) is incorrect. In this sentence, the narrator is describing being sent by Mrs. Van Hopper on useless errands whose sole purpose is to attract the attention of someone Mrs. Van Hopper wants to befriend. In this context, it makes little sense to say that the narrator is being attracted, or having her attention solicited, when in fact she is being sent by Mrs. Van Hopper as "bait" to attract the latter's "prey."

Explanation for Incorrect Answer D : Choice (D) is incorrect. In this sentence, the narrator is describing being sent by Mrs. Van Hopper on useless errands whose sole purpose is to attract the attention of someone Mrs. Van Hopper wants to befriend. In this context, it does not make sense to say that the narrator is being devoted, or dedicated, as bait to attract Mrs. Van Hoppers prey.

Explanation for Incorrect Answer E : Choice (E) is incorrect. In this sentence, the narrator is describing being sent by Mrs. Van Hopper on useless errands whose sole purpose is to attract the attention of someone Mrs. Van Hopper wants to befriend. Although the narrator is clearly hired, or employed, by Mrs. Van Hopper, in this sentence she is speaking of being sent on useless errands in order to attract the attention of someone Mrs. Van Hopper wants to befriend; she is being used.

eg

is

Explanation for Correct Answer B : Choice (B) is correct. In this sentence, the narrator is describing being sent by Mrs. Van Hopper on useless errands whose sole purpose is to attract the attention of someone Mrs. Van Hopper wants to befriend. The narrator feels, under these circumstances, that she is being employed as baitin other words, used.

te re

(D) devote

file://E:\\e5.htm

2006-11-12

The Official SAT Online Course

14/24

13

It is clear from context that by debating (line 49), the narrator means (A) asking advice about
(B)

considering strategies for

(C) talking aloud to herself about

(D) taking notes on

(E)

arguing about

ANSWERS

AND EXPLANATIONS

Explanation for Correct Answer B : Choice (B) is correct. In this part of the passage, the narrator is speaking of one unforgettable afternoon when Mrs. Van Hopper is indulging in her usual predatory behavior towards celebrities; on this occasion, she has one special victim in mind. She has installed herself in her customary seat, and is debating, or deliberating, about which method of attack (or way of entrapping the stranger) to use. It is clear from context that she is reviewing methods in her head and not aloudthe narrator can tell by Mrs. Van Hoppers actions what the latter is thinking about.

Explanation for Incorrect Answer A : Choice (A) is incorrect. In this part of the passage, it is clear from context that Mrs. Van Hopper is reviewing methods of attackthat is, ways of entrapping a certain strangerin her head and not aloud. She is clearly thinking silently and not asking advice about ways to proceed. The narrator can tell by Mrs. Van Hoppers actions what the latter is thinking about.

14

nR eg

(A)

Explanation for Incorrect Answer D : Choice (D) is incorrect. In this part of the passage, it is clear from context that Mrs. Van Hopper is reviewing methods of attackthat is, ways of entrapping a certain strangerin her head and not aloud. She is clearly thinking silently and not taking notes on ways to proceed. The narrator can tell by Mrs. Van Hoppers actions what the latter is thinking about.

Explanation for Incorrect Answer E : Choice (E) is incorrect. In this part of the passage, it is clear from context that Mrs. Van Hopper is reviewing methods of attackthat is, ways of entrapping a certain strangerin her head and not aloud. She is clearly thinking silently and not arguing with someone else about ways to proceed. The narrator can tell by Mrs. Van Hoppers actions what the latter is thinking about.

It can be inferred that Mrs. Van Hopper sends her companion to retrieve something from upstairs primarily with the goal of

communicating with a relative

(B)

having some time alone

(C) keeping information from the companion

(D) arranging a meeting for the companion

(E)

establishing a connection with a stranger

ANSWERS

AND EXPLANATIONS

Explanation for Correct Answer E : Choice (E) is correct. We know from context that Mrs. Van Hopper is obsessed with meeting celebrities, and that she has used the narrator as bait to draw the intended prey. In the paragraph directly preceding the quotation, Mrs. Van Hopper is debating her method of attack. When, after this, she speaks, it is

is

Explanation for Incorrect Answer C : Choice (C) is incorrect. In this part of the passage, it is clear from context that Mrs. Van Hopper is reviewing methods of attackthat is, ways of entrapping a certain strangerin her head and not aloud. She is clearly thinking silently and not talking aloud to herself about ways to proceed. The narrator can tell by Mrs. Van Hoppers actions what the latter is thinking about.

te r

ed

file://E:\\e5.htm

2006-11-12

The Official SAT Online Course

15/24

clearly because she has hit upon an idea for meeting the stranger: she sends the narrator to get something. In the next paragraph, the narrator says, I saw then thatthe nephew was to be the means of introduction. Clearly, the errand is intended to establish a connection with a stranger.

Explanation for Incorrect Answer A : Choice (A) is incorrect. While it is true that Mrs. Van Hopper sends the narrator to get a communication from a relativethat is, the postcard from the nephew the goal of the errand is clearly not communicating with a relative, but getting acquainted with another hotel guest, a new arrival.

Explanation for Incorrect Answer B : Choice (B) is incorrect. While it is true that sending the narrator to get something from upstairs will leave Mrs. Van Hopper momentarily alone, being alone is clearly not the latters goal; rather, she is sending the narrator to retrieve an object that will help Mrs. Van Hopper get acquainted with another hotel guest, a new arrival.

Explanation for Incorrect Answer C : Choice (C) is incorrect. While one might send a companion on an errand in order to keep him or her from finding something out, that is clearly not the case here. Mrs. Van Hopper is not interested in what the narrator knows or does not know; rather, she is sending the narrator to retrieve an object that will help Mrs. Van Hopper get acquainted with another hotel guest, a new arrival.

Explanation for Incorrect Answer D : Choice (D) is incorrect. While the long-term goal of the errand may be to arrange a meeting, the meeting is clearly meant to be between Mrs. Van Hopper and the new arrival, not between the companion and someone else.

15

In lines 64-70 (I saw...cue), the narrator imagines herself as a


(A)

playwright

(C) stagehand

(D) criminal

nR
(E)

magician

ANSWERS

Explanation for Correct Answer C : Choice (C) is correct. In the sentences cited, the narrator refers to the part she must play in Mrs. Van Hoppers schemes: Like a jugglers assistant I produced the props, then silent and attentive I waited on my cue. A stage jugglers assistant, who must assist the performer by providing props (or articles to be used on stage) on cue (or when signaled) is a kind of stagehand.

Explanation for Incorrect Answer A : Choice (A) is incorrect. In the sentences cited, the narrator refers to the part she must play in Mrs. Van Hoppers schemes: Like a jugglers assistant I produced the props, then silent and attentive I waited on my cue. While the imagery does have to do with the theatre, the narrator is imagining herself not as a writer, but, rather, as an assistant, or stagehand.

Explanation for Incorrect Answer B : Choice (B) is incorrect. In the sentences cited, the narrator refers to the part she must play in Mrs. Van Hoppers schemes: Like a jugglers assistant I produced the props, then silent and attentive I waited on my cue. While the imagery does have to do with the theatre, and while actors do wait on cues, the narrator is imagining herself as someone who assists a performer, producing props and waiting on my cuein other words, a stagehand.

Explanation for Incorrect Answer D : Choice (D) is incorrect. In the sentences cited, the narrator refers to the part

eg

(B)

actor

AND EXPLANATIONS

is t

er

file://E:\\e5.htm

ed

2006-11-12

The Official SAT Online Course

16/24

she must play in Mrs. Van Hoppers schemes: Like a jugglers assistant I produced the props, then silent and attentive I waited on my cue. There is no reference here to criminality; rather, the narrator is imagining herself as someone who assists a performerin other words, a stagehand.

Explanation for Incorrect Answer E : Choice (E) is incorrect. In the sentences cited, the narrator refers to the part she must play in Mrs. Van Hoppers schemes: Like a jugglers assistant I produced the props, then silent and attentive I waited on my cue. While the imagery does have to do with stage juggling, there is no real reference to magic: a juggler is not really a magician. Further, the narrator is imagining herself as someone who assists a performerin other words, a stagehand.

16

On the whole, the passage is developed in which of the following ways?


(A)

a single extended episode is narrated

(B)

a dialogue is recounted

(C) a physical description is followed by a summary

(D) a general description is followed by a specific example

(E)

a character description is followed by a monologue

ANSWERS

AND EXPLANATIONS

17

nR

(A)

Explanation for Incorrect Answer A : Choice (A) is incorrect. While a single episode is narrated within the passagethe last half or so of the passage takes place on one unforgettable afternoonthe passage as a whole is not composed of a single extended narration. Rather, the first half of the passage is taken up with a general description of Mrs. Van Hoppers behavior.

Explanation for Incorrect Answer B : Choice (B) is incorrect. While there is what might be termed dialogue in the passagethe direct quotation from Mrs. Van Hopper in the fourth paragraphthis speech is only a very small part of the passage as a whole and does not explain the passages development.

Explanation for Incorrect Answer C : Choice (C) is incorrect. The description that makes up the first part of the passage is about a certain pattern of behavior and cannot properly be called a physical description, as of an object or a place. Further, the latter part of the passage, since it narrates a single event, cannot properly be called a summary.

Explanation for Incorrect Answer E : Choice (E) is incorrect. While the first part of the passage, in which Mrs. Van Hoppers customary behavior is described, might be called a kind of character description, there is only one speech in the passageMrs. Van Hoppers instructions in the fourth paragraphso the passage cannot be said to end with a monologue, or long speech.

In context, garnered (line 74) is closest in meaning to

gathered

(B)

earned

assumed

eg i

st

Explanation for Correct Answer D : Choice (D) is correct. The passage begins with a general description of Mrs. Van Hoppers customary behaviordeliberately and methodically stalking celebrity visitors to the Cote dAzur hotel. The reader is both led to believe and told directly that this pattern has recurred for years. About halfway through the passage, however, the narrator begins to focus on a specific instance of this behavior: on that unforgettable afternoonSuddenly she turned to me, her small eyes alight. This narrative takes up the rest of the passage. So the method of development of the passage is that of a general description followed by a specific example.

er

ed

file://E:\\e5.htm

2006-11-12

The Official SAT Online Course

17/24

(C)
(D) inferred

(E)

harvested

ANSWERS

AND EXPLANATIONS

Explanation for Correct Answer A : Choice (A) is correct. In this sentence, the narrator of the passage is speaking of Mrs. Van Hoppers having garnered information about the "newcomer from daily papers. In context, it makes sense to say that Mrs. Van Hopper gathered, or acquired, this information from her newspaper reading.

Explanation for Incorrect Answer B : Choice (B) is incorrect. In this sentence, the narrator of the passage is speaking of Mrs. Van Hoppers having garnered information about the "newcomer from daily papers. In context, it makes no sense to say that Mrs. Van Hopper earned, or merited, this information from her newspaper reading.

New York in the wake of World War II was a city on the verge of momentous changeseconomic, social, and political. Line For almost a century it had been a 5 preeminent manufacturing and port city, absorbing the unskilled millions who flocked there from Europe, and had yielded great fortunes for the astute and daring. The Depression of the 1930s had 10 exacted a terrible toll, and leaders conferred anxiously on how to avoid a repetition of those doleful days as the war economy wound down. Even before the war, experts had 15 been cautioning New York that it was losing industry and business to other locales. A 1939 study showed the Citys percentage of wage earners down in fiftyeight industries. 20 At the close of World War II, the great port of New York remained the

nR

eg

Explanation for Incorrect Answer E : Choice (E) is incorrect. In this sentence, the narrator of the passage is speaking of Mrs. Van Hoppers having garnered information about the "newcomer from daily papers. In context, it makes little sense to say that Mrs. Van Hopper harvested, or gathered a crop from, this information in her newspaper reading.

is

te r

Explanation for Incorrect Answer D : Choice (D) is incorrect. In this sentence, the narrator of the passage is speaking of Mrs. Van Hoppers having garnered information about the "newcomer from daily papers. In context, it makes little sense to say that Mrs. Van Hopper inferred, or concluded, this information from her newspaper reading.

ed

Explanation for Incorrect Answer C : Choice (C) is incorrect. In this sentence, the narrator of the passage is speaking of Mrs. Van Hoppers having garnered information about the "newcomer from daily papers. In context, it makes little sense to say that Mrs. Van Hopper assumed, or took upon herself, this information from her newspaper reading.

file://E:\\e5.htm

2006-11-12

The Official SAT Online Course

18/24

largest general cargo port in the country and the second largest in the world, deferring only to Rotterdam. Week in and 25 week out forty thousand longshoremen worked the ninety-six piers encircling Manhattan and the eighty lining the Brooklyn waterfront, manually loading and
30

35

45

50

55

60

65

70

nR

eg

is

te

re

40

unloading sixteen million tons a year . Hundreds of tugs and barges plied the harbor and nearby rivers, guiding the big ships to their berths and ferrying cargo. Looking ahead, the new Port Authority of New York cleared forty old piers in Brooklyn and replaced them with fourteen spacious, modern piers, the best in the world. In the mid-1950s Malcolm McLean of Winston-Salem invented a method of shipping that was to revolutionize cargo ports and make the new piers obsolete. His brainchild was the truck-sized sealed containers that slashed loading and unloading time to almost nothing. A small crew of men could use cranes to load the gargantuan containers, filling a ship twenty times faster than the old gangs grappling with crates, boxes, and bales. The impregnable containers guaranteed against waterfront theft or breakage, eliminating altogether the dockside pilferage that previously had consumed up to 15 percent of some cargoes. Ships that had traditionally spent a week in port could now finish their loading in a day. The new containers required huge storing areas, far larger than were available in either Manhattan or Brooklyn. When the City of New York proposed modernizing its East River piers to handle the containers, the Port Authority said it would have to clear all the land from the river west to Third Avenue to do so. Each berth for a ship carrying containers of cargo needed fifty acres of surrounding land, compared to an old-fashioned berth of 195,000 square feet. The Port Authority erected container ports at Port Newark and Port Elizabeth, New Jersey, with their vast stretches of empty land. The old piers of Manhattan and Brooklyn languishedrotting, deserted white

file://E:\\e5.htm

2006-11-12

The Official SAT Online Course

19/24

elephants. The ranks of the longshoremen, once forty thousand 75 strong, dwindled to nine thousand. In the new technical, mechanized world of container shipping, man-hours fell from 40 million man-hours right after the war to 13.5 million in 1983.
* The island of Manhattan is bounded by the Hudson River and the state of New Jersey to the west and by the East River and the New York City borough of Brooklyn on the east.
In line 5, preeminent is closest in meaning to
(A)

18

knowledgeable

(B)

outstanding

(C) growing

(D) abnormal

ANSWERS

AND EXPLANATIONS

nR

Explanation for Incorrect Answer A : Choice (A) is incorrect. The sentence is clearly saying that New York had been a preeminent, or outstanding, hub for industries that were declining in importance. It makes little sense to say that the city had been knowledgeable, or knowing.

Explanation for Incorrect Answer C : Choice (C) is incorrect. The first sentence of the paragraph says that post-World War II New York was poised for important changes; the second sentence goes on to discuss the citys longtime importance as a shipping and manufacturing center. While it makes a certain amount of sense to call New York a growing industrial center, or one increasing in size or importance, the comparison between past and present is implied by the context: New York had been a preeminent, or outstanding, hub for industries that were now declining in importance.

Explanation for Incorrect Answer D : Choice (D) is incorrect. The sentence is clearly saying that New York had been a preeminent, or outstanding, hub for industries that were declining in importance. It makes little sense in context to say that the city had been abnormal, or deviating from normal, as an industrial center.

Explanation for Incorrect Answer E : Choice (E) is incorrect. The first sentence of the paragraph says that post-World War II New York was poised for important changes; the second sentence goes on to discuss the citys longtime importance as a shipping and manufacturing center. Notorious means widely but unfavorably known; so while it might make sense to call attention to New Yorks fame as an industrial center, there is no reason to assume that its fame was accompanied by something negative.

eg

Explanation for Correct Answer B : Choice (B) is correct. The first sentence of the paragraph says that post-World War II New York was poised for important changes; the second sentence goes on to discuss the citys longtime importance as a shipping and manufacturing center. The comparison between the past and present is implicit: New York had been a preeminent, or outstanding, hub for industries that were now declining in importance.

is te

re

(E)

notorious

file://E:\\e5.htm

2006-11-12

The Official SAT Online Course

20/24

19

In context, the unskilled millions (line 6) apparently refers to people who (A) made great fortunes
(B)

were unprepared for the Depression of the 1930s

(C) took jobs in shipping and manufacturing

(D) were uneasy about the U.S. economy

(E)

left New York to find work

ANSWERS

AND EXPLANATIONS

Explanation for Correct Answer C : Choice (C) is correct. In these lines, the author is speaking of the century-long importance of New York as a manufacturing and port city, a city that had both absorbed the unskilled millions who flocked there from Europe and yielded great fortunes for the astute and daring. It is clear from context that the overriding topic is the New York City economy, which had been thriving for a century before World War II but which was expected to falter somewhat in the post-War years. In this reading, the astute and daring are those who made great sums of money by speculation, and the unskilled millions the European emigrants who had been employedor absorbedin manufacturing and seaport trades during the citys period of greatest importance.

Explanation for Incorrect Answer B : Choice (B) is incorrect. While this paragraph does mention the Depression, it does so in the context of the Depressions having exacted a terrible tollor had a disastrous effecton New York Citys economy. It is clear from context that the unskilled millions had found rather than lost jobs in the several decades before the Depression of the 1930s.

20

nR

(A)

Explanation for Incorrect Answer D : Choice (D) is incorrect. While this paragraph does imply that various people were uneasy about the New York economy in the wake of World War II, it is clearly business leaders, not the unskilled millions of workers who had found employment in the preceding century, who are experiencing this anxiety.

Explanation for Incorrect Answer E : Choice (E) is incorrect. While there is a suggestion in this paragraph that New York City was poised to lose jobs in the post-World War II period, there is no discussion of people who left the area in order to find work.

The third paragraph is best described as a description of

a process that would soon be obsolete

(B)

a blueprint for changing an area

(C) a plan that drew much criticism

(D) a decline in the importance of an industry

(E)

an event that foreshadowed future happenings

ANSWERS

Explanation for Correct Answer A : Choice (A) is correct. The third paragraph of the passage clearly discusses the state of New York Citys shipping industry at the close of World War II. But as the rest of the passage makes clear, this was an industry whose circumstances were about to change drasticallyand for the worsein the decade of the 1950s; shipping as it was carried on the 1940s was about to become obsolete.

eg is te r

AND EXPLANATIONS

Explanation for Incorrect Answer A : Choice (A) is incorrect. In these lines, the author is speaking of the century-long importance of New York as a manufacturing and port city, a city that had both absorbed the unskilled millions who flocked there from Europe and yielded great fortunes for the astute and daring. It is clear from context that the latter those who made great fortunesare separate from the European emigrants, who clearly were employedor absorbed bythe manufacturing and seaport trades.

ed

file://E:\\e5.htm

2006-11-12

The Official SAT Online Course

21/24

Explanation for Incorrect Answer B : Choice (B) is incorrect. The only real discussion of changing an area occurs in the fifth paragraph of the passage, not in the third; and this changethe decline of the old New York pierscomes about gradually and unintentionally, not according to a blueprint.

Explanation for Incorrect Answer C : Choice (C) is incorrect. While it might be inferable that the shipping revolution described in the fourth paragraph would draw criticism, there is no indication that the description given in the third paragraph did so; further, it is more a description of a process than it is a plan.

Explanation for Incorrect Answer D : Choice (D) is incorrect. While the third paragraph of the passage does indeed discuss the shipping industryan industry, as the rest of the passage tells us, that would soon declinethis paragraph discusses New York shipping at its height, not at the time of its decline in importance.

Explanation for Incorrect Answer E : Choice (E) is incorrect. While the last sentence in the third paragraph might be construed as a foreshadowing or indication of future eventsthe clearing of the old piers to make room for the new piers, which would themselves become obsoletethe great majority of this paragraph is devoted to discussing the shipping industry of New York at its height, not in its decline.

21

The author considers Malcolm McLean to be


(A)

(B)

an opportunist

(C) an eccentric

(D) an obstructionist

ANSWERS

nR

Explanation for Correct Answer E : Choice (E) is correct. An innovator is someone who introduces important changes. Clearly the author of the passage sees McLean, who invented a revolutionary method of shipping that made the old methods (and piers) obsolete, as an innovator.

Explanation for Incorrect Answer A : Choice (A) is incorrect. A dreamer is someone who lives in a world of unreality. A dreamer may have great ideas, but they are usually of an impractical nature. Clearly the author of the passage sees McLean, whose invention was both revolutionary and practical, as an innovator rather than as a dreamer.

Explanation for Incorrect Answer B : Choice (B) is incorrect. An opportunist is someone who takes advantage of opportunities, but who does so with little thought for principle or ethics. While the author of the passage clearly considers McLean someone who seized opportunities, there is no indication in the passage that McLean was unprincipled or unethical.

Explanation for Incorrect Answer C : Choice (C) is incorrect. An eccentric is someone who behaves in an unconventional manner. While there is a conception in popular culture of inventors being eccentric, there is no indication in this passage that the author considers McLean or his behavior unconventional.

Explanation for Incorrect Answer D : Choice (D) is incorrect. An obstructionist is someone who deliberately interferes with progress. Far from considering McLean as an obstacle to progress, the author clearly sees McLean as someone who aided or sped progressan innovator.

eg

(E)

an innovator

AND EXPLANATIONS

is

te

a dreamer

re

d
file://E:\\e5.htm

2006-11-12

The Official SAT Online Course

22/24

22

The passage mentions each of the following as an advantage of container shipping EXCEPT:
(A)

large storing areas

(B)

reduced loading time

(C) theft deterrence

(D) personnel reduction

(E)

breakage prevention

ANSWERS

AND EXPLANATIONS

Explanation for Correct Answer A : Choice (A) is correct. While the passage does indeed discuss large storing areas in the last paragraph, this characteristic of container shipping is presented more as a disadvantage than an advantage. Container shipping needed huge storing areas, far larger than were available in either Manhattan or Brooklyn. Further, the plan to modernize New Yorks piers proved impractical, leaving the old piers languishing and rotting.

23

nR

(A)

Explanation for Incorrect Answer D : Choice (D) is incorrect. The author of the passage mentions personnel reduction in a favorable light in paragraph 4: A small crew of men could use cranes to load the gargantuan containers, filling a ship twenty times faster than the old gangs grappling with crates, boxes, and bales.

Explanation for Incorrect Answer E : Choice (E) is incorrect. The author of the passage presents breakage prevention as an advantage of container shipping: The impregnable containers guaranteed against waterfront theft or breakage, implying that both theft and breakage were problems associated with older shipping practices.

According to the passage, the City of New York was unable to carry out its pier modernization plan because which of the following was lacking?

Money

(B)

Space

(C) Time

(D) Materials

(E)

Motivation

ANSWERS

Explanation for Correct Answer B : Choice (B) is correct. The first two sentences of the last paragraph make it clear that there was not sufficient space available to fully adapt New York's piers to the new container shipping system: The new containers required huge storing areas, far larger than were available in either Manhattan or Brooklyn. When the City of New York proposed modernizing its East River piers to handle the containers, the Port Authority said it would have to clear all the land from the river west to Third

eg

Explanation for Incorrect Answer C : Choice (C) is incorrect. The author of the passage presents theft deterrence as an advantage of container shipping: The impregnable containers guaranteed against waterfront theft . . . eliminating the dockside pilferage that previously had consumed up to 15 percent of some cargoes.

AND EXPLANATIONS

is t

er

Explanation for Incorrect Answer B : Choice (B) is incorrect. The author of the passage presents reduced loading time as a decided advantage of container shipping, in that it slashed loading and unloading time to almost nothingShips that had traditionally spent a week in port could now finish their loading in a day.

ed

file://E:\\e5.htm

2006-11-12

The Official SAT Online Course

23/24

Avenue to do so.

Explanation for Incorrect Answer A : Choice (A) is incorrect. The passage does not mention a lack of financial resources as being behind the failure of the pier modernization plan.

Explanation for Incorrect Answer C : Choice (C) is incorrect. The only mention of time in connection with container shipping in the passage is of how much time was saved in comparison to the older shipping methods; there is no discussion of there not being sufficient time to modernize the New York piers.

Explanation for Incorrect Answer D : Choice (D) is incorrect. The passage does not mention a lack of materials as being behind the failure of the pier modernization plan.

Explanation for Incorrect Answer E : Choice (E) is incorrect. The passage illustrates that the City of New York was, on the contrary, highly motivated to modernize its piers, but that it lacked the space required to do so.

(A)

one mans vision for the future

(B)

the contrasts between two large industries

(C) the rapid growth of a city

(E)

the results of political corruption

ANSWERS

AND EXPLANATIONS

nR

Explanation for Correct Answer D : Choice (D) is correct. The last few sentences of the passage create an unmistakable image of the desolation brought about by modernization: the useless piers, languishing like white elephants; the loss of shipping jobs created by automation. These are bleak images of the down side of technological progress.

Explanation for Incorrect Answer A : Choice (A) is incorrect. While the proposal to modernize New Yorks piers might conceivably be called a vision for the future, the proposal is a failed one, and there is no attempt to connect it to one man.

Explanation for Incorrect Answer B : Choice (B) is incorrect. While the paragraph might be said to contain certain contrasts between container shipping and older shipping methods, these are not two large [and separate] industries.

Explanation for Incorrect Answer C : Choice (C) is incorrect. The paragraph mentions New York and two cities in New Jersey, but it is not concerned with their rapid growth.

Explanation for Incorrect Answer E : Choice (E) is incorrect. The dilemma presented in the paragraph is not described as being the result of political corruption, but of modernization and automation.

eg

is

te

(D) a negative aspect of modernization

re

24

In the final paragraph of the passage, the author presents an example of

Back to Score Report

Copyright 2006 The College Board. All rights reserved.

Privacy Policy

Terms of Use

Contact Us

file://E:\\e5.htm

2006-11-12

The Official SAT Online Course

24/24

nR

eg is te r

file://E:\\e5.htm

ed

2006-11-12

The Official SAT Online Course

1/12

Help | Profile | My Organizer | My Bookmarks | Logout

Answers and Explanations

Test Sections

Back to Score Report

Section 1

View Answers and Explanations


Online - Practice Test #5

Section 2

Section 3

Section 4

Section 5

Section 6

books but bookcases in a house. Each bookcase contains at least There are books. Which of the following could be the total number of books not more than bookcases? in all
(A)
(B)
(C)
(D)
(E)

Section 8

Section 9

Section 10

ANSWERS

AND EXPLANATIONS

Explanation for Correct Answer D : bookcases contained the maximum number Choice (D) is correct. If each of the

re d

te

of books,

then the total number of books would be

bookcase contained the minimum number of books,

books would be

bookcases must be at least is in this range.

nR

Explanation for Incorrect Answer A : Choice (A) is not correct. Since each bookcase contains at least

eg

total number of books in all

total number of books cannot be

Explanation for Incorrect Answer B : Choice (B) is not correct. Since each bookcase contains at least

Therefore, the total number of books in all

is

but no more than

If each

then the total number of

Of the choices, only

books, the

bookcases must be at least

Therefore, the

books, the

total number of books in all

bookcases must be at least

Therefore, the

total number of books cannot be

Explanation for Incorrect Answer C : Choice (C) is not correct. Since each bookcase contains at least

books, the

total number of books in all

bookcases must be at least

Therefore, the

total number of books cannot be

Explanation for Incorrect Answer E : Choice (E) is not correct. Since each bookcase contains no more than

books,

the total number of books in all

bookcases can be no more than

Therefore, the total number of books cannot be

file://E:\\e6.htm

2006-11-12

The Official SAT Online Course

2/12

In the figure above, lines

and

are parallel. If

what is the value of

(A)

(B)

(C)
(D)
(E)

Explanation for Correct Answer E : Choice (E) is correct. Use the properties of parallel lines cut by a transversal.

nR

Alternate interior angles are supplementary. Thus, in the figure above,

eg

Since

is t

er

it follows that

ed

ANSWERS

AND EXPLANATIONS

Since vertical

angles are of equal measure, it follows that

of equal measure. Thus,

and

supplementary, so

Corresponding angles are also

The angles labeled

and

are

Therefore,

Explanation for Incorrect Answer A : Choice (A) is not correct. Although from the figure, it may appear that

the figure is not drawn to scale.

Explanation for Incorrect Answer B : Choice (B) is not correct. This incorrect answer may be the result of estimating angle measures from the figure instead of reasoning from geometric principles.

Explanation for Incorrect Answer C : Choice (C) is not correct. Although from the figure, it may appear that

and

the figure is not drawn to scale.

Explanation for Incorrect Answer D : Choice (D) is not correct. This incorrect answer may be the result of estimating angle measures from the figure instead of reasoning from geometric principles.

file://E:\\e6.htm

2006-11-12

The Official SAT Online Course

3/12

Graphed above is the amount that a computer shop charges for a repair job as a function of the number of hours required to do the job. Which of the following is most consistent with the information in the graph?
(A)

The shop charges the same amount for any job of any length.

(B)

The shop charges at an hourly rate for work with no fixed initial amount.

(C) The shop charges a fixed initial amount plus an hourly rate for work, starting at the beginning of the first hour.

(E)

The shop charges at an hourly rate for work beginning at the start of the hours of work. first hour. This rate decreases after

ANSWERS

AND EXPLANATIONS

nR

(A)

Explanation for Incorrect Answer A : Choice (A) is not correct. If the shop charged the same amount for any job of any length, then the graph would be a horizontal line, which it is not.

Explanation for Incorrect Answer B : Choice (B) is not correct. If the shop charged at an hourly rate for work with no fixed initial amount, then the graph would be a line through the origin, which it is not.

Explanation for Incorrect Answer C : Choice (C) is not correct. If the shop charged a fixed initial amount plus an hourly rate for work, starting at the beginning of the first hour, then the graph would show hour, but these two charges are the same. a greater charge for 2 hours than for

Explanation for Incorrect Answer E : Choice (E) is not correct. If the shop charged at an hourly rate for work beginning hours of work, then at the start of the first hour, and the rate decreased after hours, but it does not. the slope of the graph would decrease after

For how many values of

None

eg

Explanation for Correct Answer D : hours, which Choice (D) is correct. The graph is a horizontal line segment up to hours or less of work. Then corresponds to a fixed amount charged for the first the graph is a line segment with positive slope, corresponding to an hourly rate for hours. work beyond

is t

is the equation above true?

er

ed

hours or less of work. The (D) The shop charges a fixed amount for the first hours. shop charges at an hourly rate for work beyond

file://E:\\e6.htm

2006-11-12

The Official SAT Online Course

4/12

(B)

One

(C) Two

(D) Four

(E)

More than four

ANSWERS

AND EXPLANATIONS
means that either

Explanation for Correct Answer C :

Choice (C) is correct. The equation

or

The solutions of these last two equations are

and

Both

these values of

satisfy

and they are the only two values that satisfy

Explanation for Incorrect Answer A :

Choice (A) is not correct. The values

and

both satisfy

Explanation for Incorrect Answer B :

Choice (B) is not correct. The values

and

Explanation for Incorrect Answer D :

Choice (D) is not correct. The values

but they are the only two values of

Explanation for Incorrect Answer E :

st

Choice (E) is not correct. The values

but they are the only two values of

nR

An amusement park awards tickets that can be exchanged for prizes to reward high scores at an arcade game. Some scores and the corresponding number of prize the up to tickets awarded are shown in the table above. For scores from -point score number of tickets awarded increases by a constant amount for each

increase. How many tickets would be awarded for a score of

(A)
(B)
(C)

eg i

er ed
and

that satisfy

both satisfy

both satisfy

and

both satisfy

that satisfy

file://E:\\e6.htm

2006-11-12

The Official SAT Online Course

5/12

(D)

(E)

ANSWERS

AND EXPLANATIONS
points, there are

Explanation for Correct Answer D : Choice (D) is correct. For a score of

tickets awarded, and

for a score of

there are

tickets awarded. For this

-point score

increase, the increase in the number of tickets awarded is

For scores from

up to

the number of tickets awarded increases by a constant amount for each

-point score increase. Thus, any

-point score increase within this interval

increases the number of tickets awarded by

Therefore, since

tickets are

awarded for a score of

points, it follows that

tickets are

awarded for a score of

points, and

tickets are awarded for a

score of

Explanation for Incorrect Answer A : Choice (A) is not correct. The number of tickets awarded for a score of

re


is

d nR

is

points

is

Explanation for Incorrect Answer B : Choice (B) is not correct. The number of tickets awarded for a score of

Explanation for Incorrect Answer C : -point score increase results in an additional Choice (C) is not correct. Since a

ticket awarded, the number of tickets awarded for a score of

eg

is

but the number of tickets awarded for a score of

te

is

but the number of tickets awarded for a score of

points

is

is

not

Explanation for Incorrect Answer E : Choice (E) is not correct. The number of tickets awarded for a score of

points

but the number of tickets awarded for a score of

is

students in a class, only one of whom is a junior. If There are to the class, how many students in the class will NOT be juniors?

seniors are added

(A)
(B)
(C)
(D)
(E)

file://E:\\e6.htm

2006-11-12

The Official SAT Online Course

6/12

ANSWERS

AND EXPLANATIONS

Explanation for Correct Answer D : students in the class, only one of Choice (D) is correct. There are originally students in the original class are not juniors. Then whom is a junior. Thus, seniors are added to the class; since these students are seniors, they are not seniors are added to the class, there are juniors. Therefore, after the

students in the class who are not juniors.

Explanation for Incorrect Answer A : students in the class were not juniors Choice (A) is not correct. If exactly seniors had been added to the class, then originally there would have after the

been

students in the class who were not juniors. But originally

there were

students who were not juniors.

Explanation for Incorrect Answer B : students in the class were not juniors Choice (B) is not correct. If exactly seniors had been added to the class, then originally there would have after the

been

students in the class who were not juniors. But originally

there were

students who were not juniors.

students in the class were not juniors

been

students in the class who were not juniors. But originally

there were

students who were not juniors.

Explanation for Incorrect Answer E : seniors were added to the class, there were a Choice (E) is not correct. After the

total of

students in the class. However, the question asks how many of these

students were not juniors, which is

nR

eg i

following must be true?
(A)

st e

re

and the

after the

seniors had been added to the class, then originally there would have

d
the point

Explanation for Incorrect Answer C : Choice (C) is not correct. If exactly

The quadratic function graphed above has equation

Which of the

(B)

(C)

is even.

(D)

is odd.

(E)

is the square of an integer.

ANSWERS

AND EXPLANATIONS
is on the graph

Explanation for Correct Answer A :

Choice (A) is correct. Since

of

Thus, the intersection of the graph of

-axis is

units above the

origin. Since the graph of

intersects the positive

-axis, the value of

must be

file://E:\\e6.htm

2006-11-12

The Official SAT Online Course

7/12

may be even, odd, or the square of positive. It cannot be negative. The value of an integer, but it need not be. Therefore, the only one of the five statements that must be true is

Explanation for Incorrect Answer B :

Choice (B) is not correct. The point

is the intersection of the graph of

and

-axis. Since the graph of the cannot be negative.

intersects the positive

-axis, the value of

Explanation for Incorrect Answer C : Choice (C) is not correct. It may be true that be true.

is an even integer, but it need not

Explanation for Incorrect Answer D : Choice (D) is not correct. It may be true that be true.

is an odd integer, but it need not

Explanation for Incorrect Answer E : Choice (E) is not correct. It may be true that need not be true.

is the square of an integer, but it

is te

not

re d


and

nR

(A)

(B)

(C)

(D)

(E)

ANSWERS

Explanation for Correct Answer E : Choice (E) is correct. The length of each side of a triangle is less than the sum of Squaring both sides of this the lengths of the other two sides; thus,

inequality yields

Explanation for Incorrect Answer A : Choice (A) is not correct. The length of each side of a triangle is less than the sum Squaring both sides of this of the lengths of the other two sides; thus,

inequality yields

Explanation for Incorrect Answer B : Choice (B) is not correct. The length of each side of a triangle is less than the sum Squaring both sides of this of the lengths of the other two sides; thus,

eg

Which of the following inequalities is true about the lengths the triangle above?

of the sides of

AND EXPLANATIONS

inequality yields

not

file://E:\\e6.htm

2006-11-12

The Official SAT Online Course

8/12

Explanation for Incorrect Answer C : Choice (C) is not correct. The length of each side of a triangle is less than the sum Squaring both sides of this of the lengths of the other two sides; thus,

inequality yields

not

Explanation for Incorrect Answer D : Choice (D) is not correct. The length of each side of a triangle is less than the sum Squaring both sides of this of the lengths of the other two sides; thus,

inequality yields

not

If a circle has circumference

what is the radius of the circle?

Your Response:

Correct Response(s): 3.5, 7/2

Explanation:

The correct answer is

or

. The circumference of a circle is equal to

where

is the radius of the circle. Thus,

Solving for

The answer may also be gridded as the equivalent decimal

10

If

and

and

are positive integers, what is one possible value of

Your Response:

Correct Response(s): 5 or 10 or 15 or 20

eg

Explanation:

The possible correct answers are

nR

must be a factor of

Therefore, the possible values of

and are values of be gridded as the answer.

11

is
Since

te
and

re

and

gives

d
. If

then

have no prime factors in common,

In addition,

and, since

is positive,

are

and

(The corresponding

respectively.) Any one of

or

may

floors of an The figure above shows, on the left, the call buttons for the buttons labeled A through I for the apartment building, and on the right, the

file://E:\\e6.htm

2006-11-12

The Official SAT Online Course


apartments on each floor of the building. How many different apartments can be called by pressing one button on the left and one button on the right?

9/12

Your Response:

Correct Response(s): 108

Explanation:

floors in the building, and there are . There are The correct answer is floors. Therefore, by pressing one button on the left, apartments on each of the floors, and then one button on the right, for one of the for one of the

apartments on that floor,

different apartments can be called.

12

If the function

is defined by

for what value of

does

Your Response:

Correct Response(s): 66

Explanation:

The correct answer is

. If

then

and

13

nR

above Students at Northern High School are represented in Figure above. Figure students is another way to show which pets these students have. If the same are represented in both figures, what is the total number of students represented by the shaded circle in Figure

eg

is t

er

ed

therefore,

file://E:\\e6.htm

2006-11-12

The Official SAT Online Course

10/12

Your Response:

Correct Response(s): 275

Explanation:

represents all students . The shaded circle in Figure The correct answer is at Northern High School who have cats. This includes students who have cats only of the shows that and students who have both cats and dogs. Figure

students at Northern High School have cats only and

have both cats and

dogs. Thus, the shaded circle in Figure

represents

of the

students at Northern High School. Since both figures represent the same

students, the shaded circle in Figure

represents

of

students, that is,

students.

14

If

is a positive integer, and

what is the value of

Your Response:

re

is

Explanation:

d
to both sides of

Correct Response(s): 25


gives the

The correct answer is

. Adding

equation

dividing both sides of this equation by

Therefore, the value of

15

eg
Your Response:

If

nR

Correct Response(s): 2

Explanation:

is

is

te

gives the equation

what is the value of

The correct answer is

. Use the distributive law.

Since

it follows that

Therefore,

and the value of

16

file://E:\\e6.htm

2006-11-12

The Official SAT Online Course

11/12

inches. What is the volume, in cubic inches, of the The sphere above has radius smallest cube that can contain the entire sphere?

Your Response:

Correct Response(s): 1728

Explanation:

The correct answer is

17

nR eg

If the value of
Your Response:

As shown above, when the sphere is placed inside the smallest cube that can contain it, the greatest distance between two points on the sphere (the diameter of the sphere) must be equal to the shortest distance between two points on opposite faces of the cube (which is equal to the edge length of the cube). Thus, the edge inches. length of this cube will be equal to the diameter of the sphere, which is . The volume of the cube, in cubic inches, will be

is

te

is twice the value of

Correct Response(s): 4

Explanation:

re

d
The correct answer is

what is the value of

. Since the value of

is twice the value of

the

equation

is true. Cross-multiplying

gives

or

It follows that

or

18

file://E:\\e6.htm

2006-11-12

The Official SAT Online Course

12/12

In the

-coordinate plane above,

and

is perpendicular to

If the coordinates of point

are

what is the

-intercept of line

Your Response:

Correct Response(s): 16/3, 5.33

Explanation:

The correct answer is

or

. Since

is perpendicular to

ed

is

follows that

is a right triangle. By the Pythagorean Theorem,

The coordinates of point

so the coordinates of

are

it

are

and the coordinates of

er

is the

are

Thus, the slope of line

Hence the equation

is t

of line

is

where

-intercept of

Point

is on

so

Therefore,

The answer can be

nR

eg

gridded as

or as its rounded decimal equivalent,

Back to Score Report

Copyright 2006 The College Board. All rights reserved.

Privacy Policy

Terms of Use

Contact Us

file://E:\\e6.htm

2006-11-12

The Official SAT Online Course

1/24

Help | Profile | My Organizer | My Bookmarks | Logout

Answers and Explanations

Test Sections

Back to Score Report

Section 1

View Answers and Explanations


Online - Practice Test #5

Section 2

Section 3

Section 4

Section 5

Numismatics, or coin collecting, and becoming popular in the United States around 1857, when the replacement of the large cent by the new flying-eagle cent led enthusiasts to start collecting the earlier coin.
(A)

Section 6

and becoming

Section 8

Section 9

(B)

becoming

Section 10

(C) will become

(D) became

(E)

it became

Explanation for Correct Answer D : Choice (D) is correct. It avoids the error of the original by providing a main verb (became) to perform the action of the sentence (Numismatics became ).

nR

(A)

Explanation for Incorrect Answer B : Choice (B) creates a sentence fragment. The participle becoming cannot serve as a main verb. The other verb, led, is the verb for the subordinating clause beginning with when and cannot serve as the main verb. Therefore, a main verb is needed to complete the action of the sentence.

Explanation for Incorrect Answer C : Choice (C) involves an error in verb tense that results in faulty logic. The action of the sentence takes place in the past (around 1857), so a past-tense verb, became, not a future-tense verb, is needed.

Explanation for Incorrect Answer E : Choice (E) involves an error in pronoun use that results in a sentence fragment. The pronoun it is used unnecessarily, so the subject of the sentence, Numismatics, has no main verb to complete the action of the sentence.

Just as his predecessor Mozart, for whom composing music seemed to come easily, Beethoven always struggled to perfect his work.

Just as

(B)

Even though

(C) Whereas

(D) Unlike with

(E)

Unlike

eg

Explanation for Incorrect Answer A : Choice (A) involves improper coordination that results in a sentence fragment. The coordinating conjunction and is not needed, and the participle becoming cannot serve as the main verb in this sentence. The other verb, led, is the verb for the subordinating clause beginning with when and cannot serve as the main verb. A main verb is needed to complete the action of the sentence.

is

te

re d

ANSWERS

AND EXPLANATIONS

file://E:\\e7.htm

2006-11-12

The Official SAT Online Course

2/24

ANSWERS

AND EXPLANATIONS

Explanation for Correct Answer E : Choice (E) is correct. It avoids the improper word choice of the original by replacing the adverb Just and the preposition as with the appropriate preposition Unlike to indicate that the idea expressed in the prepositional phrase (UnlikeMozart) contrasts with the idea expressed in the independent clause (Beethovenwork).

Explanation for Incorrect Answer A : Choice (A) involves inappropriate word choice that creates faulty logic. The phrase Just as is inappropriate in the context of this sentence because it expresses similarity, not contrast. The sentence explains how Beethoven was not like Mozart, so the preposition Unlike is needed to indicate that the idea expressed in the prepositional phrase (UnlikeMozart) contrasts with the idea expressed in the independent clause (Beethovenwork).

Explanation for Incorrect Answer B : Choice (B) involves the misuse of a subordinating conjunction. The subordinating conjunction even though does express contrast, but it is used inappropriately to introduce a prepositional phrase, not a subordinate clause.

The Incas kept records with knotted cords called quipus, historians believe they were used to document crop production and to aid in tax collection.

nR

(E)
(B)

(A)

historians believe they were used

historians believing in its use

(C) which historians believe they use

(D) which historians believe were used

and historians believe it was used

ANSWERS

Explanation for Correct Answer D : Choice (D) is correct. It avoids the comma-splice error and ambiguous pronoun of the original by turning the second independent clause (historians collection) into a subordinate clause introduced by the relative pronoun which, and by making clear that the relative pronoun which refers to the noun quipus.

Explanation for Incorrect Answer A : Choice (A) involves a comma-splice error and an ambiguous pronoun. Two independent clauses (The quipus and historians collection) are joined only by a comma rather than by a comma and a coordinating conjunction or by a semicolon. Also, the plural pronoun they could refer to Incas, records, or quipus.

Explanation for Incorrect Answer B : Choice (B) contains awkward phrasing and an error in pronounantecedent agreement. The singular possessive pronoun its does not agree in number with the plural noun quipus to which it refers. Also, in the context of this sentence, the use of the preposition in after the participle believing is not idiomatic.

eg

AND EXPLANATIONS

is

te

Explanation for Incorrect Answer D : Choice (D) involves an unnecessary preposition. The preposition Unlike is appropriate to introduce a contrasting idea, but the added preposition with is unnecessary in the context of this sentence.

re

Explanation for Incorrect Answer C : Choice (C) involves the misuse of a subordinating conjunction. The subordinating conjunction whereas does express contrast, but it is used inappropriately to introduce a prepositional phrase, not a subordinate clause.

file://E:\\e7.htm

2006-11-12

The Official SAT Online Course

3/24

Explanation for Incorrect Answer C : Choice (C) involves an error in verb tense and an unclear pronoun reference. The present tense of the verb use is inconsistent with the past tense of the main verb kept. Also, the plural pronoun they could refer either to Incas or (illogically) to historians.

Explanation for Incorrect Answer E : Choice (E) creates an error in pronounantecedent agreement. Although the addition of the coordinating conjunction and corrects the comma-splice error of the original, the singular pronoun its does not agree with the plural noun quipus to which it refers.

While females were not allowed to compete in the Olympic games of ancient Greece; they were not even allowed to watch them.
(A)

While females were not allowed

(B)

Even though females were not allowed

(C) Females were not allowed

(D) They did not allow females

(E)

With females, they were not allowed

ANSWERS

AND EXPLANATIONS

nR

(A)

Explanation for Incorrect Answer B : Choice (B) involves improper subordination. The first clause is a subordinate clause because it is introduced by a subordinate conjunction (Even though), but it cannot stand alone (as indicated by the semicolon). To make the clause an independent clause that is equal in importance to the sentences other independent clause, the subordinating conjunction Even though should be omitted.

Explanation for Incorrect Answer D : Choice (D) involves a pronoun error. Although the first clause has been turned into an independent clause, equal in importance to the other independent clause of the sentence, it now has a pronoun (They) without an antecedent to which it refers.

Explanation for Incorrect Answer E : Choice (E) involves awkward phrasing. The addition of the preposition With and the pronoun they is unnecessary. The noun females can stand on its own as the subject of the first independent clause.

In psychological theory, the term social learning is used to describe learning that is influenced by ones environment and not the influence of innate or internal forces.

and not the influence of

(B)

and not, instead,

(C) rather than by

(D) rather than when it is

(E)

rather than being influenced due to

eg i

Explanation for Incorrect Answer A : Choice (A) involves improper subordination. The first clause is a subordinate clause because it is introduced by a subordinate conjunction (While), but it cannot stand alone (as indicated by the semicolon). To make the clause an independent clause that is equal in importance to the other independent clause, the subordinating conjunction While should be omitted.

st e

re

Explanation for Correct Answer C : Choice (C) is correct. It avoids the improper subordination of the original by deleting the subordinating conjunction While and thereby making the clause an independent clause, which can stand alone.

file://E:\\e7.htm

2006-11-12

The Official SAT Online Course

4/24

ANSWERS

AND EXPLANATIONS

Explanation for Correct Answer C : Choice (C) is correct. It avoids the error in parallelism and faulty logic of the original by replacing the phrase and not the influence of with the conjunction rather than and the preposition by. This revision makes the grammatical structure that precedes the conjunction rather than parallel with the grammatical structure that follows it. This revision also fixes the faulty logic in the phrase learning that is not the influence.

Explanation for Incorrect Answer A : Choice (A) involves a lack of parallelism and faulty logic. The grammatical structure of the phrase after and is made up of an adverb (not) modifying is and a noun (the influence) that functions as a subject complement renaming learning. It does not make sense to say learning that is not the influence. Also, this grammatical structure is not parallel with the first structure (influenced by ones environment). Replacing this phrase with the rather than by fixes both problems.

Explanation for Incorrect Answer D : Choice (D) involves a lack of parallelism. The structure following the conjunction rather than is a subordinate clause introduced by the relative adverb when. It is not parallel with the prepositional phrase (by ones environment) that precedes the conjunction. Also, the phrase when it is is awkward and can be replaced by the simple preposition by.

nR

(A)

Giant stars, or those that weigh 100 times what the Sun weighs, are important to galaxies and the universe because with furious combustion, it produces many important elements that form planets and other bodies, including carbon, oxygen, sodium, and neon.

because with furious combustion, it produces

(B)

because their furious combustion produces

(C) due to their furious combustion, producing

(D) as their furious combustion produced

(E)

in that with furious combustion, they produced

ANSWERS

Explanation for Correct Answer B : Choice (B) is correct. It avoids the pronoun error and imprecise language of the original by removing the pronoun it, which could refer to the Sun, the universe, or combustion.

Explanation for Incorrect Answer A : Choice (A) involves a pronoun error and imprecise language. The antecedent of the pronoun it is unclear; it could be the Sun, the universe, or combustion.

Explanation for Incorrect Answer C : Choice (C) involves faulty logic, imprecise language, and an unclear pronoun

eg

Explanation for Incorrect Answer E : Choice (E) involves an error in verb tense and awkward phrasing. The progressive tense of the verb being is inconsistent with the simple present tense of the verb is. Also, the phrase rather than being influenced due to can be replaced by the much simpler, and more idiomatic, phrase rather than by.

AND EXPLANATIONS

is t

er ed

Explanation for Incorrect Answer B : Choice (B) involves redundancy and a lack of parallelism. The adverbs not and instead are redundant, so instead is not needed. Also, for the structure following and to be parallel with the structure preceding and, the preposition by is needed: is influenced by and not by .

file://E:\\e7.htm

2006-11-12

The Official SAT Online Course

5/24

reference. The prepositional phrase introduced by due to creates faulty logic. It illogically suggests that the giant stars are important because of their combustion, not because of what their combustion produces. Also, the participial phrase introduced by producing describes the subject, Giant stars, and imprecisely suggests that the giant stars, not their combustion, produce many important elements.

Explanation for Incorrect Answer D : Choice (D) contains an incorrect verb tense and ambiguous language. The past tense of produced is inconsistent with the present tense of the main verb are and also illogically suggests that the combustion of giant stars no longer occurs. The subordinating conjunction as could mean when or because. In the context of this sentence, its meaning is not clear.

Explanation for Incorrect Answer E : Choice (E) involves imprecise language and an incorrect verb tense. The subordinate clause introduced by in that is imprecise, suggesting that the giant stars, not their combustion, produce many important elements. Also, the past tense of produced is inconsistent with the present tense of the main verb are and illogically suggests that the combustion of giant stars no longer occurs.

American photographer Annie Leibovitz is renowned for her revealing, eye-catching portraits of celebrities, and characterized by carefully staged settings and the use of vivid primary colors.
(A)

and

(B)

and she is

(C) each of them are

(D) each of which are

(E)

which are

ANSWERS

AND EXPLANATIONS

nR

Explanation for Incorrect Answer A : Choice (A) involves an incorrectly used coordinating conjunction and faulty logic. The coordinating conjunction and that precedes the verb characterized forms a compound verb for the subject of the independent clause, Annie Leibovitz. It illogically suggests that Annie Leibovitz, not her portraits, is characterized by carefully staged settings and the use of vivid primary colors.

Explanation for Incorrect Answer B : Choice (B) involves faulty logic. The addition of the pronoun she, which refers to the subject of the sentence, Annie Leibovitz, and the addition of the singular helping verb is create a second independent clause. This new clause illogically suggests that Annie Leibovitz, not her portraits, is characterized by carefully staged settings and the use of vivid primary colors.

Explanation for Incorrect Answer C : Choice (C) creates a comma-splice error and a subjectverb agreement error. The addition of a subject (each) and a helping verb (are) creates an independent clause connected to the other independent clause (American celebrities) by only a comma. Also, the plural verb are does not agree in number with its subject, the singular pronoun each.

Explanation for Incorrect Answer D : Choice (D) involves an error in subjectverb agreement. Although the addition of the phrase each of which are creates a subordinate clause that modifies portraits, the plural verb are does not agree in number with its subject, the singular pronoun each.

eg i

Explanation for Correct Answer E : Choice (E) is correct. It avoids the errors of the other options by providing a relative pronoun, which, to introduce the subordinate clause that describes portraits and by providing the plural verb are, which is the appropriate helping verb for the passive verb characterized.

st er

ed

file://E:\\e7.htm

2006-11-12

The Official SAT Online Course

6/24

Decorative arts, art forms that have a mainly practical or ornamental purpose, and include ceramics, basketry, furniture making, and glassblowing. (A) and include
(B)

including

(C) includes

(D) include

(E)

which include

ANSWERS

AND EXPLANATIONS

Explanation for Correct Answer D : Choice (D) is correct. It avoids the errors of the original by removing the unnecessary coordinating conjunction and, so that the sentence has a main plural verb, include, that agrees with the plural subject, Decorative arts.

Explanation for Incorrect Answer C : Choice (C) involves an error in subjectverb agreement. The singular verb includes does not agree with the plural noun and subject of the sentence, arts.

nR

(E)
(A)

Explanation for Incorrect Answer E : Choice (E) creates a sentence fragment. The relative pronoun which makes all that follows it a subordinate clause that describes arts, so the sentence has no main verb. Without the relative pronoun, the verb include can serve as the main verb of the sentence.

About 35 percent of the world's orange juice is produced by Florida, compared with Brazil, the worlds largest orange producer, which has nearly 50 percent.

Brazil, the worlds largest orange producer, which has nearly 50 percent

(B)

Brazil, with nearly 50 percent, it is the worlds largest orange producer

(C) nearly 50 percent that are produced by Brazil as the worlds largest orange producer

(D) the production of Brazil, the worlds largest orange producer, with nearly 50 percent

nearly 50 percent produced by Brazil, the worlds largest orange producer

ANSWERS

Explanation for Correct Answer E : Choice (E) is correct. It avoids the illogical comparison of the original by replacing Brazil with 50 percent as the object of the multi-word preposition compared with.

Explanation for Incorrect Answer A : Choice (A) involves an illogical comparison. The prepositional phrase introduced by the multi-word preposition compared with functions as an adjective modifying

eg

AND EXPLANATIONS

is te r

Explanation for Incorrect Answer B : Choice (B) results in a sentence fragment. The participle including introduces a participial phrase that functions as an adjective describing arts, so the sentence has no main verb. To fix this problem, the verb include is needed.

ed

Explanation for Incorrect Answer A : Choice (A) involves an unnecessary coordinating conjunction, and. The conjunction and connects the verbs have and include as compound verbs of the subordinate clause, so the subject of the main clause has no verb to complete the action of the sentence. To make include the verb of the independent clause, the and should be omitted.

file://E:\\e7.htm

2006-11-12

The Official SAT Online Course

7/24

the subject of the sentence, 35 percent, so the amount, 35 percent, is illogically compared with the country, Brazil, rather than with another amount, 50 percent.

Explanation for Incorrect Answer B : Choice (B) involves an illogical comparison and a comma-splice error. The prepositional phrase introduced by the multi-word preposition compared with functions as an adjective modifying the subject of the sentence, 35 percent, so the amount, 35 percent, is illogically compared with the country, Brazil, rather than with another amount, 50 percent. Also, two independent clauses (About percent and it producer) are joined only by a comma rather than by a semicolon or by a comma and a subordinating conjunction.

Explanation for Incorrect Answer C : Choice (C) contains an error in subjectverb agreement and awkward phrasing. The relative pronoun that refers to the singular amount noun 50 percent, so it requires the singular verb is. Also, the long phrase beginning with nearly, which includes an unnecessary subordinate clause (that are produced by Brazil) and preposition (as), can be reduced to the simpler phrase nearly 50 percent produced by Brazil, the worlds largest orange producer.

Explanation for Incorrect Answer D : Choice (D) contains a non-idiomatic preposition and awkward phrasing. In the context of this sentence, the prepositional phrase production of Brazil is not idiomatic and creates nonsense. Brazil is not being produced. The idiomatic preposition in this case is by (production by Brazil). Also, the long phrase beginning with nearly can be reduced to the simpler phrase nearly 50 percent produced by Brazil, the worlds largest orange producer.

10

Many of blues great Bessie Smiths songs describe the experiences of southern Black migrants, especially the struggles of Black women to adjust to urban life in the northern United States.

(B)

how Black women struggled at adjusting

(C) when it was Black women struggling to adjust

(D) Black women, they struggled with adjusting

(E)

for Black women and struggling to adjust

nR
ANSWERS

Explanation for Correct Answer A : Choice (A) is correct. It avoids the errors of the other options by providing an appositive, the struggles of Black women to adjust, to rename the experiences that Bessie Smith sang about.

Explanation for Incorrect Answer B : Choice (B) involves awkward, unidiomatic phrasing. The infinitive to adjust is the idiomatic verb form to follow struggled in this context. In addition, the entire awkward phrase how Black women struggled at adjusting can be reduced to the more precise the struggles of Black women to adjust.

Explanation for Incorrect Answer C : Choice (C) involves unnecessary pronoun use and faulty logic. It does not make sense to use the relative adverb when to describe experiences. In addition, there is nothing in the sentence to which the pronoun it can logically refer.

Explanation for Incorrect Answer D : Choice (D) involves faulty logic. What comes after especially should be an appositive that renames or describes the experiences that Bessie Smith sang about, but people (Black women) cannot logically be experiences.

Explanation for Incorrect Answer E : Choice (E) involves awkward and wordy phrasing, resulting in nonsense. What comes after especially should be an appositive that renames or describes the experiences that Bessie Smith sang about, but the prepositional phrase for

eg

AND EXPLANATIONS

is t

(A)

the struggles of Black women to adjust

er e

d
file://E:\\e7.htm

2006-11-12

The Official SAT Online Course

8/24

Black women cannot logically be experiences. In addition, the entire awkward phrase for Black women and struggling to adjust can be reduced to the more precise phrase struggles of Black women to adjust.

11

The Bretons who began emigrating to the United States from the Brittany region of France in the early twentieth century were distinguished from other French citizens by their Celtic origin, but about 40 percent of those who emigrated spoke a Celtic language closely related to Welsh. (A) but about 40 percent of those who emigrated spoke
(B)

about 40 percent of whom spoke

(C) about 40 percent of which were speaking

(D) with about 40 percent of those who emigrated speaking

(E)

where 40 percent of them were speaking

ANSWERS

AND EXPLANATIONS

12

nR

ANSWERS

Explanation for Incorrect Answer B : Choice (B) involves an unclear pronoun. The relative pronoun whom has no clear antecedent; it could refer to Bretons or to French citizens.

Explanation for Incorrect Answer C : Choice (C) involves errors in pronoun reference. The relative pronoun which has no clear antecedent; it could refer to Bretons or to French citizens. In addition, which should be used to refer to things, not to people.

Explanation for Incorrect Answer E : Choice (E) involves an inappropriate relative adverb and a verb-tense error. The relative adverb where introduces a subordinate clause that functions as an adjective, but the noun to which it refers not clear. It could be United States, Brittany region, or France.

Although they are not fast runners, wolves could have maintained a loping run

for many miles, running throughout the night if necessary. No error

Corrected Sentence: Although they are not fast runners, wolves can maintain a loping run for many miles, running throughout the night if necessary.

Explanation for Correct Answer B :

eg

Explanation for Incorrect Answer A : Choice (A) involves a coordination error. The coordinating conjunction but indicates contrast, but the idea expressed in the clause following but does not contrast with the idea in the clause preceding but. Instead, the clause following but explains how 40 percent of the French immigrants were distinguished from the other French immigrants.

AND EXPLANATIONS

is t

er

file://E:\\e7.htm

ed

Explanation for Correct Answer D : Choice (D) is correct. It avoids the coordination error of the original by deleting the coordinating conjunction but and turning the independent clause (but Welsh) into a prepositional phrase that functions as an adverb modifying the verb were distinguished. The coordinating conjunction but indicates contrast, but the idea expressed in the clause following but does not contrast with the idea in the clause preceding but. Instead, the clause following but explains how 40 percent of the French immigrants were distinguished from the other French immigrants.

2006-11-12

The Official SAT Online Course

9/24

The error in this sentence occurs at (B), where an incorrect verb tense is used. The conditional present perfect tense of could have maintained is not consistent with the simple present tense of the subordinate clause (are).

Explanation for Incorrect Answer A : There is no error at (A). The plural pronoun they agrees in number with the plural noun to which it refers, wolves; the plural verb are agrees with the plural subject of the subordinate clause, they; and the adverb not appropriately modifies the verb are.

Explanation for Incorrect Answer C : There is no error at (C). The preposition for is an appropriate preposition to introduce the prepositional phrase (for many miles) that functions as an adjective describing the noun run. The adjective many appropriately modifies the noun miles.

Explanation for Incorrect Answer D : There is no error at (D). The preposition throughout properly introduces the prepositional phrase throughout the night, which functions as an adverb modifying the participle running.

Explanation for Incorrect Answer E : There is an error in the sentence.

13

The credit for making Franz Kafka

belong internationally to his famous as a writer

friend, novelist Max Brod, who editedKafkas unpublished manuscripts and thenhad

te r

ed

ANSWERS

nR

Corrected Sentence: The credit for making Franz Kafka internationally famous belongs to his friend, novelist Max Brod, who edited Kafkas unpublished manuscripts and then had them published, despite Kafkas dying wishes to the contrary.

Explanation for Correct Answer B : The error in this sentence occurs at (B), where a plural verb (belong) does not agree in number with its singular subject (credit).

Explanation for Incorrect Answer A : There is no error at (A). The adverb internationally appropriately modifies the adjective famous.

Explanation for Incorrect Answer C : There is no error at (C). The relative pronoun who properly refers to the proper noun Max Brod, a person, and properly introduces the subordinate clause (who published) that describes that proper noun.

Explanation for Incorrect Answer D : There is no error at (D). The coordinating conjunction and properly connects the two verb phrases (edited manuscripts and then published) that complete the action of the subordinate clause introduced by the relative pronoun who. The adverb then appropriately modifies the verb had.

Explanation for Incorrect Answer E : There is an error in the sentence.

eg is

them published, despite Kafkas dying wishes to the contrary.

No error

AND EXPLANATIONS

file://E:\\e7.htm

2006-11-12

The Official SAT Online Course

10/24

14

Unlike
country-and-western bands,

which often feature the harmonica, banjo, or

fiddle, rock bands

tended to use electronic equipment, including amplifiers, guitars,

and organs.

No error

ANSWERS

AND EXPLANATIONS

Corrected Sentence: Unlike country-and-western bands, which often feature the harmonica, banjo, or fiddle, rock bands tend to use electronic equipment, including amplifiers, guitars, and organs.

Explanation for Correct Answer C : The error in this sentence occurs at (C), where an incorrect verb tense is used. The past tense of tended is not consistent with the simple present tense of the subordinate clause, feature. The simple present tense (tend) is needed instead.

Explanation for Incorrect Answer D : There is no error at (D). The adjective electronic appropriately modifies the noun equipment.

15

nR

ANSWERS

Explanation for Incorrect Answer E : There is an error in the sentence.

Like her nonfiction, Jean Craighead Georges fiction draws extensively not only from

published material but also she had firsthand observations of animals and ecological

systems. No error

Corrected Sentence: Like her nonfiction, Jean Craighead Georges fiction draws extensively not only from published material but also from firsthand observations of animals and ecological systems.

Explanation for Correct Answer C : The error in this sentence occurs at (C), where there is an error in parallel structure. The correlative conjunctions not only but also require that the grammatical structure following the first half the pair is the same as the grammatical structure following the second half. The independent clause she had firsthand observations is not grammatically parallel to the prepositional phrase, from published material, that follows not only. The preposition from is needed instead (but also from)

eg

AND EXPLANATIONS

is

Explanation for Incorrect Answer B : There is no error at (B). The relative pronoun which properly refers to the noun bands and properly introduces the subordinate clause (which fiddle) that describes bands.

te

re

Explanation for Incorrect Answer A : There is no error at (A). The preposition unlike is an appropriate preposition to introduce the prepositional phrase (Unlike bands) that modifies the subject of the independent clause (rock bands) and to signal that a contrasting idea will follow.

file://E:\\e7.htm

2006-11-12

The Official SAT Online Course

11/24

Explanation for Incorrect Answer A : There is no error at (A). The preposition like is an appropriate preposition to introduce the prepositional phrase (like nonfiction) that modifies the subject of the independent clause, fiction.

Explanation for Incorrect Answer B : There is no error at (B). The singular verb draws agrees in number with its singular subject fiction, and the adverb extensively appropriately modifies the verb draws.

Explanation for Incorrect Answer D : There is no error at (D). The coordinating conjunction and appropriately connects the two objects of the preposition of, animals and systems.

Explanation for Incorrect Answer E : There is an error in the sentence.

16

Most

ships move

through

the Suez Canal under their own power,

so extremely large

ships must be

assisted by

a tugboat.

No error

ANSWERS

AND EXPLANATIONS

Corrected Sentence: Most ships move through the Suez Canal under their own power, but extremely large ships must be assisted by a tugboat.

Explanation for Correct Answer C : The error in this sentence occurs at (C), where an inappropriate coordinating conjunction, so, connects two independent clauses that express contrasting ideas. The coordinating conjunction but is needed to properly express the contrast between the ideas in the sentence.

17

nR

Explanation for Incorrect Answer A : There is no error at (A). The word Most appropriately functions as an adjective modifying the noun ships.

Explanation for Incorrect Answer B : There is no error at (B). The preposition through is an idiomatic preposition to follow the verb move and properly introduces the prepositional phrase (through the Suez Canal) that functions as an adverb modifying the verb move.

Explanation for Incorrect Answer D : There is no error at (D). The past participle assisted is the appropriate verb form to create the present tense of a passive verb, be assisted. The preposition by is the idiomatic preposition to follow a passive verb and to introduce the prepositional phrase that modifies that verb.

Explanation for Incorrect Answer E : There is an error in the sentence.

Ralph Ellisons Invisible Man is the story of a nameless young Black man who

ultimately decides to forge his own identity rather than accept the one assigned to

eg is

te

re

d
file://E:\\e7.htm

2006-11-12

The Official SAT Online Course

12/24

No error
him.

ANSWERS

AND EXPLANATIONS

Corrected Sentence:

Explanation for Correct Answer E : There is no error in this sentence.

Explanation for Incorrect Answer A : There is no error at (A). The singular verb is agrees in number with its singular subject, Ralph Ellisons Invisible Man.

Explanation for Incorrect Answer B : There is no error at (B). The adverb ultimately correctly modifies the verb decides, and the singular decides agrees in number with the singular subject of the subordinate clause, man.

Explanation for Incorrect Answer D : There is no error at (D). The definite article the properly modifies the pronoun one, which, in turn, properly serves as the direct object of accept.

18

The Sun was shining for nearly five billion years and is thoughtto have sufficient

eg is

te

re

Explanation for Incorrect Answer C : There is no error at (C). The phrase rather than is an idiomatic phrase that serves as a conjunction meaning and not. It correctly connects the two phrases forge his own identity and accept the one assigned to him.

thermonuclear fuel in its core to shine for about another five billion.

No error

nR
ANSWERS

Corrected Sentence: The Sun has been shining for nearly five billion years and is thought to have sufficient thermonuclear fuel in its core to shine for about another five billion.

AND EXPLANATIONS

Explanation for Correct Answer A : The error in this sentence occurs at (A), where an incorrect verb tense is used. The past progressive tense of was shining is incorrect for expressing an action that started in the past and is still occurring. The present perfect progressive, has been shining, is needed instead.

Explanation for Incorrect Answer B : There is no error at (B). The singular verb is agrees in number with the singular subject of the sentence, Sun, and the past participle thought is the appropriate verb form to create the present tense of a passive verb, is thought.

Explanation for Incorrect Answer C : There is no error at (C). The adjective sufficient appropriately describes the noun fuel.

Explanation for Incorrect Answer D : There is no error at (D). The preposition in properly introduces the prepositional phrase in its core, which functions as an adjective describing the noun fuel; the singular possessive pronoun its agrees with the singular

file://E:\\e7.htm

2006-11-12

The Official SAT Online Course

13/24

noun, Sun, to which it refers; and the noun core properly serves as the object of the preposition in.

Explanation for Incorrect Answer E : There is an error in the sentence.

19

The study

showed

that children

who consumed small amounts of added sugars

eating

on average one serving more of grains per day

than did children who ate larger

amounts of these same sugars.

No error

ANSWERS

AND EXPLANATIONS

Corrected Sentence: The study showed that children who consumed small amounts of added sugars ate on average one serving more of grains per day than did children who ate larger amounts of these same sugars.

Explanation for Incorrect Answer A : There is no error at (A). The past tense of the verb showed appropriately expresses an action completed in the past.

20

nR

ANSWERS

Explanation for Incorrect Answer B : There is no error at (B). The relative pronoun who appropriately refers to the noun children and properly introduces the subordinate clause (who consumed small amounts of added sugars) that describes children.

Explanation for Incorrect Answer D : There is no error at (D). The conjunction than is necessary to complete the comparison begun with more. Also, the past tense of did is consistent with the past tense of the other verbs in the sentence (showed, consumed, and ate).

Explanation for Incorrect Answer E : There is an error in the sentence.

In many parts of the world where grasses cover vast expanses of land, periodic,

controlled burning is practiced in order to keep woody brush from gaining a foothold

and it stimulates continuing grass growth. No error

Corrected Sentence: In many parts of the world where grasses cover vast expanses of land, periodic, controlled burning is practiced in order to keep woody brush from gaining a foothold and to stimulate continuing grass growth.

eg is te r

AND EXPLANATIONS

Explanation for Correct Answer C : The error in this sentence occurs at (C), where an improper verb form is used. The participle eating cannot serve as a main verb for the subject of this subordinate clause, children, and its present tense is inconsistent with the simple past tense of the other verbs in the sentence (showed, consumed, did, and ate). The past-tense verb ate should be used instead.

ed

file://E:\\e7.htm

2006-11-12

The Official SAT Online Course

14/24

Explanation for Correct Answer D : The error in this sentence occurs at (D), where there is an error in parallelism. The independent clause it stimulates growth is not parallel with the infinitive phrase to keep to which it is connected by the coordinating conjunction and. The infinitive phrase to stimulate is needed instead.

Explanation for Incorrect Answer A : There is no error at (A). The relative adverb where correctly introduces the subordinate clause that modifies the noun parts.

Explanation for Incorrect Answer B : There is no error at (B). The passive verb is practiced is appropriately used in this sentence, and the singular is agrees in number with its singular subject, burning.

Explanation for Incorrect Answer C : There is no error at (C). The preposition from is an idiomatic preposition to follow keep, and the gerund gaining, which is a noun, properly serves as the object of that preposition. In addition, the prepositional phrase appropriately functions as an adverb modifying keep.

Explanation for Incorrect Answer E : There is an error in the sentence.

21

Corrected Sentence: The newspaper reported that even with the increase in the minimum wage, many people are still having trouble making ends meet.

22

nR

Explanation for Correct Answer B : The error in this sentence occurs at (B), where improper phrasing is used. The participle having introduces a participial phrase that incorrectly modifies the subject of the sentence, newspaper. So that the phrase the increase in the minimum wage describes the intended noun, people, the phrase must be introduced by a preposition, in this case, with.

Explanation for Incorrect Answer A : There is no error at (A). The past tense of the verb reported appropriately expresses an action that was completed in the past, and the pronoun that properly introduces the subordinate clause that meet, which functions as the direct object of the independent clause.

Explanation for Incorrect Answer C : There is no error at (C). The adjective many appropriately modifies the noun people, which, in turn, properly functions as the subject of the subordinate clause introduced by that.

Explanation for Incorrect Answer D : There is no error at (D). The participle making properly introduces the participial phrase making ends meet that functions as an adjective describing the noun trouble.

Explanation for Incorrect Answer E : There is an error in the sentence.

eg

ANSWERS

AND EXPLANATIONS

is

te

are still having trouble making ends meet.

re

No error

The newspaper reported that having the increase in the minimum wage, many people

file://E:\\e7.htm

2006-11-12

The Official SAT Online Course

15/24

Used primarily in the United States and Canada, American Sign Language,

which

comprises hand signs, facial expressions, and fingerspelling,

including many regional

differences and slang.

No error

ANSWERS

AND EXPLANATIONS

Corrected Sentence: Used primarily in the United States and Canada, American Sign Language, which comprises hand signs, facial expressions, and fingerspelling, includes many regional differences and slang.

Explanation for Correct Answer C : The error in this sentence occurs at (C), where a participle, including, is used instead of a main verb, includes, which is needed to complete the action of the sentence.

Explanation for Incorrect Answer B : There is no error at (B). The relative pronoun which appropriately refers to the proper noun American Sign Language and introduces the subordinate clause that describes American Sign Language.

Explanation for Incorrect Answer E : There is an error in the sentence.

23

nR

ANSWERS

The United States is the worlds largest producer of cranberries and cranberry

products, most of which is consumed there and in Canada. No error

Corrected Sentence: The United States is the worlds largest producer of cranberries and cranberry products, most of which are consumed there and in Canada.

Explanation for Correct Answer C : The error in this sentence occurs at (C), where an incorrect verb is used. The singular helping verb is does not agree in number with the plural subject of the subordinate clause (most) that refers to the compound plural objects of the preposition, cranberries and cranberry products.

Explanation for Incorrect Answer A : There is no error at (A). The singular noun producer appropriately renames the singular subject of the independent clause, United States. The preposition

eg

Explanation for Incorrect Answer D : There is no error at (D). The plural adjective many idiomatically describes the plural noun differences.

AND EXPLANATIONS

is te

re

Explanation for Incorrect Answer A : There is no error at (A). The past participle used appropriately introduces the participial phrase that modifies the subject of the independent clause, American Sign Language. The adverb primarily properly modifies the participle used, which functions as an adjective.

file://E:\\e7.htm

2006-11-12

The Official SAT Online Course

16/24

of is an idiomatic preposition to follow producer and to introduce the prepositional phrase (of products) that functions as an adjective describing producer.
Explanation for Incorrect Answer B : There is no error at (B). The pronoun most, which can be singular or plural, agrees in number with the compound plural objects of the preposition, cranberries and cranberry products, to which it refers.

Explanation for Incorrect Answer D : There is no error at (D). The coordinating conjunction and appropriately connects two adverbs, there, and the prepositional phrase, in Canada, that modify the verb consumed. The preposition in is an idiomatic preposition to follow the verb consumed.

Explanation for Incorrect Answer E : There is an error in the sentence.

24

Almost all animals

by eating it that hibernate large prepare for during summer

amounts of food,

which they convert to thick layers of fat.

No error

ANSWERS

AND EXPLANATIONS

Explanation for Correct Answer B : The error in this sentence occurs at (B), where an improper pronoun is used. The pronoun it has no antecedent to which it can logically refer.

25

nR

Explanation for Incorrect Answer A : There is no error at (A). The relative pronoun that correctly refers to animals and properly introduces the subordinate clause that describes animals. The plural verb of this subordinate clause, hibernate, agrees in number with the pronoun that, which refers to the plural noun animals.

Explanation for Incorrect Answer C : There is no error at (C). The preposition by is the idiomatic preposition to follow the word prepare, and the gerund eating properly serves as the object of that preposition. The prepositional phrase appropriately serves as an adverb modifying the verb prepare.

Explanation for Incorrect Answer D : There is no error at (D). The relative pronoun which introduces the clause (which they convert) that modifies the noun food. The plural pronoun they agrees in number with the plural noun to which it refers, animals, and the plural verb convert agrees in number with its plural subject, they.

Explanation for Incorrect Answer E : There is an error in the sentence.

Among birds that forage in tree trunks, nuthatches are the only ones that

regularly feed with their heads facing downward, when they often find food in bark

crevices overlooked in their upward search. No error

eg i

st er

Corrected Sentence: Almost all animals that hibernate prepare for hibernation during summer by eating large amounts of food, which they convert to thick layers of fat.

ed

file://E:\\e7.htm

2006-11-12

The Official SAT Online Course

17/24

ANSWERS

AND EXPLANATIONS

Corrected Sentence: Among birds that forage in tree trunks, nuthatches are the only ones that regularly feed with their heads facing downward, often finding food in bark crevices overlooked in their upward search.

Explanation for Correct Answer C : The error in this sentence occurs at (C), where a subordinate clause creates faulty logic. The use of when, an adverb that modifies the verb feed, makes the sentence illogically suggest that nuthatches regularly feed when they often find food.

Explanation for Incorrect Answer A : There is no error at (A). The relative pronoun that correctly refers to birds and properly introduces the subordinate clause that describes birds. The plural verb of this subordinate clause, forage, agrees in number with the pronoun that, which refers to the plural noun birds.

Explanation for Incorrect Answer E : There is an error in the sentence.

26

If you can acquire the necessary calories by drinking gasoline instead of by eating

nR
No error

food, you would be able to run 26 miles on about one-twelfth of a gallon of gas.

ANSWERS

Corrected Sentence: If you could acquire the necessary calories by drinking gasoline instead of by eating food, you would be able to run 26 miles on about onetwelfth of a gallon of gas.

Explanation for Correct Answer A : The error in this sentence occurs at (A), where an incorrect verb tense is used. The present tense of the verb can acquire in the subordinate clause is inconsistent with the past tense of the verb in the independent clause, would be able.

Explanation for Incorrect Answer B : There is no error at (B). The preposition by is the idiomatic preposition to follow acquire, and the gerund drinking, which is a noun, appropriately serves as the object of that preposition. The prepositional phrase appropriately functions as an adverb modifying the verb acquire.

Explanation for Incorrect Answer C : There is no error at (C). The second-person pronoun you properly serves as

eg

AND EXPLANATIONS

is te

Explanation for Incorrect Answer D : There is no error at (D). The past participle overlooked properly functions as an adjective describing the noun food, and the preposition in appropriately introduces the prepositional phrase (in search) that serves as an adverb modifying the participle overlooked.

re

Explanation for Incorrect Answer B : There is no error at (B). The adverb regularly properly modifies the plural verb feed, and the plural verb agrees in number with the relative pronoun that, which refers to the plural noun ones.

file://E:\\e7.htm

2006-11-12

The Official SAT Online Course

18/24

the subject of the independent clause and is consistent with the second-person pronoun you in the introductory subordinate clause (If food).

Explanation for Incorrect Answer D : There is no error at (D). The infinitive to run is the idiomatic verb form to follow the verb able.

Explanation for Incorrect Answer E : There is an error in the sentence.

27

In many large cities in the United States, the presence of a

culturally diverse

population

has led

to repeated calls

wholly or partly that in languages curricula taught

other than English.

No error

ANSWERS

AND EXPLANATIONS

Explanation for Correct Answer C : The error in this sentence occurs at (C), where the relative pronoun (that) is not idiomatic and is incorrectly used to introduce a prepositional phrase. The preposition for is needed instead (calls for curricula).

Explanation for Incorrect Answer A : There is no error at (A). The adverb culturally appropriately modifies the adjective diverse, which, in turn, describes the noun population.

28

nR

ANSWERS

Explanation for Incorrect Answer B : There is no error at (B). The singular helping verb has agrees in number with the subject of the independent clause, presence, and the perfect tense (has led) is appropriate for expressing an action completed in the present time.

Explanation for Incorrect Answer D : There is no error at (D). The compound adverb phrase wholly or partly appropriately modifies the past participle taught, which functions as an adjective describing the noun curricula.

Explanation for Incorrect Answer E : There is an error in the sentence.

Of ancient origin, the game of checkers was played in Egypt during the time of the

pharaohs and is mentioned in the writings of Homer and Plato. No error

Corrected Sentence:

Explanation for Correct Answer E : There is no error in this sentence.

eg is te re d

AND EXPLANATIONS

Corrected Sentence: In many large cities in the United States, the presence of a culturally diverse population has led to repeated calls for curricula taught wholly or partly in languages other than English.

file://E:\\e7.htm

2006-11-12

The Official SAT Online Course

19/24

Explanation for Incorrect Answer A : There is no error at (A). The preposition of is used properly to introduce the prepositional phrase, Of ancient origin, that modifies the subject of the sentence, game.

Explanation for Incorrect Answer B : There is no error at (B). The singular helping verb was agrees in number with the singular subject of the sentence, game, and the past participle played is the appropriate verb form to create the present tense of a passive verb (was played).

Explanation for Incorrect Answer C : There is no error at (C). The preposition during combines with the time to produce an idiomatic phrase that indicates when checkers was played in Egypt.

Explanation for Incorrect Answer D : There is no error at (D). The singular helping verb is agrees in number with the singular subject game, and the past participle mentioned is the appropriate verb form to create the present tense of a passive verb (is mentioned).

29

By

the early 1920's, Louis Armstrong recorded

his first solos as a member of King

which such pieces as Chimes Blues and Tears, Olivers Creole Jazz Band in

he composed with pianist Lil Hardin.

ANSWERS

AND EXPLANATIONS

Corrected Sentence: In the early 1920s, Louis Armstrong recorded his first solos as a member of King Olivers Creole Jazz Band in such pieces as Chimes Blues and Tears, which he composed with pianist Lil Hardin.

(1) The early history of astronomy was full of misunderstandings. (2) Some of

nR

Explanation for Correct Answer A : The error in this sentence occurs at (A), where an inappropriate preposition (By) is used. In the context of this sentence, the preposition In is the appropriate preposition to introduce the prepositional phrase that functions as an adverb modifying the verb recorded.

Explanation for Incorrect Answer B : There is no error at (B). The singular possessive pronoun his agrees in number and gender with the singular noun Louis Armstrong to which it refers, and it appropriately combines with the adjective first to describe the plural noun solos.

Explanation for Incorrect Answer C : There is no error at (C). The adjective such appropriately modifies the noun pieces, which properly functions as the object of the preposition in, and the preposition as appropriately introduces the prepositional phrase that functions as an adjective modifying the noun pieces.

Explanation for Incorrect Answer D : There is no error at (D). The relative pronoun which is used appropriately to refer to the pieces that Armstrong composed with pianist Lil Hardin.

Explanation for Incorrect Answer E : There is an error in the sentence.

eg

is

te re d
No error

file://E:\\e7.htm

2006-11-12

The Official SAT Online Course

20/24

them were funny, it's like the controversy of the "canali" on Mars. (3) In the late 1800s an Italian astronomer named Giovanni Schiaparelli studied Mars. (4) He had a high-powered telescope that he used to look at Mars. (5) Schiaparelli thought he saw channels criss-crossing the planet's surface. (6) He was intrigued: perhaps these channels were evidence that Mars had great flowing rivers like the Earth. (7) Schiaparelli made charts of the surface of Mars and labeled it with the Italian word "canali."

(8) Unfortunately, "canali" can be translated into English as either "channels" or "canals." (9) Channels and canals are two different things because channels are formed naturally by water, while canals are constructed by people. (10) Some people translated "canali" as "canals," word began to spread that the lines Schiaparelli saw through his telescope were actually canals that had been built by intelligent beings. (11) One of them was an amateur astronomer named Percival Lowell. (12) He wrote a series of best-selling books. (13) In these books Lowell publicized the notion that these "canals" were built by Martian farmers who understood irrigation.

(14) In 1965 a U.S. spacecraft flying close to the surface of Mars sent back conclusive pictures. (15) There are no prominent channels anywhere on the planet. (16) Lowell and Schiaparelli saw what they wanted to see. (17) Lowell was wrong, of course, but so was Schiaparelli.

30

Which is the best version of the underlined portion of sentence 2 (reproduced below)?

Some of them were funny, it's like the controversy of the "canali" on Mars.
(A)

(as it is now)

(B)

funny; it's like

(C) funny, like

(D) funny, even

(E)

Explanation for Correct Answer C : Choice (C) is correct. The sentence properly introduces the controversy that is the subject of the passage as one of a number of funny misunderstandings.

31

nR

Explanation for Incorrect Answer A : Choice (A) is unsatisfactory because it joins two independent thoughts with only a comma.

Explanation for Incorrect Answer B : Choice (B) is unsatisfactory because it is illogical; "it's" does not make sense in this context.

Explanation for Incorrect Answer D : Choice (D) is unsatisfactory because "even" suggests that others do not find the "canali" controversy funny. The passage does not indicate this.

Explanation for Incorrect Answer E : Choice (E) is unsatisfactory because "as funny as" puts the emphasis of the sentence on the other misunderstandings, with the expectation that they will be compared to the "canali" controversy. Such a comparison does not happen in the passage.

Which is the best way to combine sentences 3 and 4 (reproduced below)?

In the late 1800s an Italian astronomer named Giovanni Schiaparelli studied Mars. He had a high-powered telescope that he used to look at Mars.

In the late 1800s an Italian astronomer named Giovanni Schiaparelli

eg i

ANSWERS

AND EXPLANATIONS

st

as funny as

er e

file://E:\\e7.htm

2006-11-12

The Official SAT Online Course

21/24

(A)

studied Mars by a high-powered telescope.

(B)

In the late 1800s an Italian astronomer named Giovanni Schiaparelli studied Mars with a high-powered telescope that he used to look at Mars. (C) In the late 1800s an Italian astronomer named Giovanni Schiaparelli studied Mars, he had a high-powered telescope that he used.
(D) In the late 1800s an Italian astronomer named Giovanni Schiaparelli used a high-powered telescope to study Mars.

(E)

In the late 1800s an Italian astronomer named Giovanni Schiaparelli, using a high-powered telescope to look, studied Mars.

ANSWERS

AND EXPLANATIONS

Explanation for Correct Answer D : Choice (D) is correct. The sentence nicely joins the ideas of sentences 3 and 4 without repeating information unnecessarily.

Explanation for Incorrect Answer A : Choice (A) is unsatisfactory because it is unidiomatic to write that Schiaparelli "studied Mars by" a telescope; "with" or "through" would be more appropriate.

Explanation for Incorrect Answer C : Choice (C) is unsatisfactory because it joins two complete thoughts with only a comma.

32

nR

(A)

Which word would be best to insert at the beginning of sentence 10 (reproduced below)?

Some people translated "canali" as "canals," word began to spread that the lines Schiaparelli saw through his telescope were actually canals that had been built by intelligent beings.

Whereas

(B)

However

(C) If

(D) Although

(E)

Because

ANSWERS

Explanation for Correct Answer E : Choice (E) is correct. The word "Because" appropriately signifies the relationship between the translation problem and the misunderstanding about the "canals."

Explanation for Incorrect Answer A : Choice (A) is unsatisfactory because "Whereas" indicates that the mistranslation of "canali" and the misunderstanding about the building of the "canals" were contradictory ideas, when in fact one caused the other.

Explanation for Incorrect Answer B : Choice (B) is unsatisfactory because "However" makes no sense in this context.

eg

AND EXPLANATIONS

is

Explanation for Incorrect Answer E : Choice (E) is unsatisfactory because it unnecessarily separates the act of looking at Mars from the act of studying Mars. As the passage indicates, the activities are the same.

te re d

Explanation for Incorrect Answer B : Choice (B) is unsatisfactory because it repeats information about studying/looking at Mars unnecessarily.

file://E:\\e7.htm

2006-11-12

The Official SAT Online Course

22/24

Explanation for Incorrect Answer C : Choice (C) is unsatisfactory because "If" suggests that there is some doubt as to whether or not people mistranslated "canali" in this way. The implication of the passage is that people did mistranslate the term, resulting in a misconception about the building of the "canals."

Explanation for Incorrect Answer D : Choice (D) is unsatisfactory because "Although" implies that one would not expect word to spread given that the mistranslation occurred, when the passage indicates the opposite.

33

What is the best version of the underlined portion of sentence 11 (reproduced below)?

One of them was an amateur astronomer named Percival Lowell.


(A)

(As it is now)

(B)

One of the most intelligent was

(C) This idea was popularized by

(D) It was spread even more by someone else,

(E)

The person who solved the problem was

ANSWERS

AND EXPLANATIONS

34

nR

(A)

Explanation for Incorrect Answer B : Choice (B) is unsatisfactory because Lowell is not described elsewhere in the passage as intelligent; in fact, his theory is shown to be based on a simple misunderstanding.

Explanation for Incorrect Answer D : Choice (D) is unsatisfactory because it results in an awkward and illogical sentence.

Explanation for Incorrect Answer E : Choice (E) is unsatisfactory because, according to the passage, Lowell created more problems than he solved.

What is the best way to combine sentences 12 and 13 (reproduced below)?

He wrote a series of bestselling books. In these books Lowell publicized the notion that these "canals" were built by Martian farmers who understood irrigation.

In a series of bestselling books, Lowell publicized the notion that these "canals" were built by Martian farmers who understood irrigation.

(B)

He wrote a series of books that was a bestseller and publicized the notion that these "canals" were built by Martian farmers who understood irrigation.

(C) His books that were bestsellers publicized the notion that these "canals" were built by Martian farmers who understood irrigation.

(D) He wrote a series of bestselling books; Lowell publicized the notion that these "canals" were built by Martian farmers who understood irrigation.

(E)

In these books, which were bestsellers, Lowell publicized the notion that these "canals" were built by Martian farmers who understood irrigation.

eg

Explanation for Incorrect Answer A : Choice (A) is unsatisfactory because the pronoun "them" seems to refer to "intelligent beings" who built canals on Mars, and Lowell was clearly not one of these.

is

te

Explanation for Correct Answer C : Choice (C) is correct. It properly signifies the relationship between the idea presented in the previous sentence and the astronomer Lowell.

re

file://E:\\e7.htm

2006-11-12

The Official SAT Online Course

23/24

ANSWERS

AND EXPLANATIONS

Explanation for Correct Answer A : Choice (A) is correct. The resulting sentence maintains the effective structure of the original sentence 13 while adding the important information from sentence 12.

Explanation for Incorrect Answer B : Choice (B) is unsatisfactory because it is improper to describe a series of books as "a bestseller."

Explanation for Incorrect Answer C : Choice (C) is unsatisfactory because, following sentence 11, it is more appropriate for the subject of the sentence to be "Lowell" than to be "his books."

Explanation for Incorrect Answer D : Choice (D) is unsatisfactory because it does not indicate the relationship between the books Lowell wrote and his popularization of the "intelligent beings" idea.

Explanation for Incorrect Answer E : Choice (E) is unsatisfactory because it suggests that Lowell's books have been previously mentioned in the passage ("In these books"), when this is the first mention of them.

35

What would best replace "it" in sentence 15 (reproduced below)?

According to these images, there are no prominent channels anywhere on it.


(A)

the planet

(C) the pictures

(D) these

(E)

them

nR
ANSWERS

Explanation for Correct Answer A : Choice (A) is correct. It corrects the error of the original by clearly specifying the object on which there are no prominent channels: the planet Mars.

Explanation for Incorrect Answer B : Choice (B) is unsatisfactory because, in the context of the passage, it makes no sense to discuss "channels" on a spacecraft.

Explanation for Incorrect Answer C : Choice (C) is unsatisfactory because the "images" mentioned earlier in the sentence are the "pictures" taken by the spacecraft. There is no need to refer to them twice.

Explanation for Incorrect Answer D : Choice (D) is unsatisfactory because the word these does not logically refer to anything before or after sentence 15.

Explanation for Incorrect Answer E : Choice (E) is unsatisfactory because it is unclear to what "them" refers.

eg

AND EXPLANATIONS

is

(B)

the spacecraft

te r

ed

file://E:\\e7.htm

2006-11-12

The Official SAT Online Course

24/24
Back to Score Report

Copyright 2006 The College Board. All rights reserved.

Privacy Policy

Terms of Use

Contact Us

nR

eg i

st

er

file://E:\\e7.htm

ed

2006-11-12

The Official SAT Online Course

1/18

Help | Profile | My Organizer | My Bookmarks | Logout

Answers and Explanations

Test Sections

Back to Score Report

Section 1

View Answers and Explanations


Online - Practice Test #5

Section 2

Section 3

Section 4

Section 5

Section 6

The anthropology professor hoped that his latest book would appeal to popular as well as to ------- readers, thereby earning him ------- in both realms. (A) general. . disdain
(B)

Section 8

lay. . attention

Section 9

(C) academic. . anonymity

Section 10

(D) avid. . remuneration

(E)

scholarly. . acclaim

Explanation for Incorrect Answer A : Choice (A) is incorrect. General, in this context, means not confined by specialization. Disdain is a feeling of contempt or scorn. If one were to insert these terms into the text, the sentence would read The anthropology professor hoped that his latest book would appeal to popular as well as to general readers, thereby earning him disdain in both realms. The phrases as well as and both realms indicate that the professor hoped his book would appeal to two separate groups of readers. The term general does not make logical sense in this context because there is not a clear distinction between the terms general and popular as they refer to similar groups of people. Additionally, the anthropology professor wanted his book to appeal to readers, so it is illogical to suggest that the anthropology professor hoped to earn disdain, or contempt.

nR

Explanation for Incorrect Answer B : Choice (B) is incorrect. Lay means not associated with a profession or any particular knowledge. Attention is observation or notice. If one were to insert these terms into the text, the sentence would read The anthropology professor hoped that his latest book would appeal to popular as well as to lay readers, thereby earning him attention in both realms. The phrases as well as and both realms indicate that the professor hoped his book would appeal to two separate groups of readers. The term lay does not make logical sense in this context because the terms lay and popular both refer to readers from one group, the general public.

Explanation for Incorrect Answer C : Choice (C) is incorrect. Academic means scholarly or related to higher learning. Anonymity is a state of being unknown or unidentified. If one were to insert these terms into the text, the sentence would read The anthropology professor hoped that his latest book would appeal to popular as well as to academic readers, thereby earning him anonymity in both realms. The professor likely hoped his book would appeal to readers in both the popular realm and the academic, or scholarly, realm, but it is illogical to suggest that the professor hoped to earn anonymity. Anonymity, or a state of being unknown, is not usually described as something one earns, and there is no reason to believe that the professor would not want to be known for his work.

eg

is te r

Explanation for Correct Answer E : Choice (E) is correct. Scholarly means learned or academic. Acclaim is praise or applause. If one were to insert this term into the text, the sentence would read The anthropology professor hoped that his latest book would appeal to popular as well as to scholarly readers, thereby earning him acclaim in both realms. The phrases as well as and both realms indicate that the professor hoped his book would appeal to two separate groups of readers. It makes sense to say that the professor hoped to earn acclaim, or praise, from both popular readers and scholarly, or academic, readers.

ed

ANSWERS

AND EXPLANATIONS

file://E:\\e8.htm

2006-11-12

The Official SAT Online Course

2/18

Explanation for Incorrect Answer D : Choice (D) is incorrect. Avid means enthusiastic and vigorous. Remuneration is payment for a service. If one were to insert these terms into the text, the sentence would read The anthropology professor hoped that his latest book would appeal to popular as well as to avid readers, thereby earning him remuneration in both realms. The phrases as well as and both realms indicate that the professor hoped his book would appeal to two separate groups of readers. The terms avid and popular do not necessarily describe separate groups. Popular readers, or readers from the general public, can also be avid, or enthusiastic, readers.

As if intended to squelch rumors of cutbacks, the companys annual celebration was as ------- as ever. (A) sparing
(B)

tawdry

(C) belated

(D) lavish

(E)

sated

ANSWERS

AND EXPLANATIONS

nR

Explanation for Incorrect Answer A : Choice (A) is incorrect. Sparing means restrained, modest, or thrifty. If one were to insert this term into the text, the sentence would read As if intended to squelch rumors of cutbacks, the companys annual celebration was as sparing as ever. The word squelch indicates that the missing term will describe a celebration that might suppress rumors of cutbacks, or reductions in spending. A "sparing," or thrifty, celebration would more likely encourage rumors of cutbacks than squelch them.

Explanation for Incorrect Answer B : Choice (B) is incorrect. Tawdry means cheap. If one were to insert this term into the text, the sentence would read As if intended to squelch rumors of cutbacks, the companys annual celebration was as tawdry as ever. A tawdry, or cheap, celebration would most likely strengthen rumors of cutbacks rather than squelch, or suppress, them.

Explanation for Incorrect Answer C : Choice (C) is incorrect. Belated means delayed or late. If one were to insert this term into the text, the sentence would read As if intended to squelch rumors of cutbacks, the companys annual celebration was as belated as ever. The word "squelch" indicates that the missing term will describe a celebration that might suppress rumors of cutbacks, or reductions in spending. Although a celebration could be belated, it is not logical to suggest that a late celebration might suppress such rumors.

Explanation for Incorrect Answer E : Choice (E) is incorrect. Sated means satisfied almost to the point of excess. If one were to insert this term into the text, the sentence would read As if intended to squelch rumors of cutbacks, the companys annual celebration was as sated as ever. While a person can be described as sated, it does not make logical sense to describe a celebration as sated, or satisfied.

The professors ------- lecture on American history served more to confuse the students than to clarify the difficult material.

eg

is te

Explanation for Correct Answer D : Choice (D) is correct. Lavish means extravagant and elaborate. If one were to insert this term into the text, the sentence would read As if intended to squelch rumors of cutbacks, the companys annual celebration was as lavish as ever. The word squelch indicates that the missing term will describe a celebration that might suppress rumors of cutbacks, or reductions in spending. A lavish, or extravagant, celebration could certainly suppress such rumors.

re

file://E:\\e8.htm

2006-11-12

The Official SAT Online Course

3/18

(A)

perspicacious

(B)

exhaustive

(C) cogent

(D) scintillating

(E)

disjointed

ANSWERS

AND EXPLANATIONS

Explanation for Correct Answer E : Choice (E) is correct. Disjointed means out of order and incoherent. If one were to insert this term into the text, the sentence would read The professors disjointed lecture on American history served more to confuse the students than to clarify the difficult material. A disjointed, or incoherent, lecture would most likely confuse the students more than it would help them to understand the difficult material.

nR

(A)

Explanation for Incorrect Answer C : Choice (C) is incorrect. Cogent means relevant. If one were to insert this term into the text, the sentence would read The professors cogent lecture on American history served more to confuse the students than to clarify the difficult material. There is no reason to believe that a cogent, or relevant, lecture would confuse the students more than it would help them to understand the difficult material.

Explanation for Incorrect Answer D : Choice (D) is incorrect. Scintillating means lively and witty. If one were to insert this term into the text, the sentence would read The professors scintillating lecture on American history served more to confuse the students than to clarify the difficult material. There is no reason to believe that a scintillating, or lively, lecture would confuse the students more than it would help them to understand the difficult material.

Since the foreign correspondent was accustomed to completing his assignments under ------- conditions, the commotion of the subway at rush hour presented no ------- his creativity.

squalid . . boon for

(B)

tranquil . . obstruction to

(C) tumultuous . . impediment to

(D) destructive . . demonstration of

(E)

flagrant . . benefit to

ANSWERS

Explanation for Correct Answer C : Choice (C) is correct. Tumultuous means chaotic and disorderly. An impediment means something that slows down or gets in the way of something

eg is te

Explanation for Incorrect Answer B : Choice (B) is incorrect. Exhaustive means thorough. If one were to insert this term into the text, the sentence would read The professors exhaustive lecture on American history served more to confuse the students than to clarify the difficult material. There is no reason to believe that an exhaustive, or thorough, lecture would confuse the students more than it would help them to understand the difficult material. On the contrary, an exhaustive lecture would be likely to clarify the material.

AND EXPLANATIONS

re

Explanation for Incorrect Answer A : Choice (A) is incorrect. Perspicacious means keen or having acute mental abilities. If one were to insert this term into the text, the sentence would read The professors perspicacious lecture on American history served more to confuse the students than to clarify the difficult material. It is unlikely that a lecture could be both perspicacious and confusing to the students.

file://E:\\e8.htm

2006-11-12

The Official SAT Online Course

4/18

else. If one were to insert these terms into the text, the sentence would read Since the foreign correspondent was accustomed to completing his assignments under tumultuous conditions, the commotion of the subway at rush hour presented no impediment to his creativity. The term Since indicates that there is a relationship between the conditions to which the correspondent was accustomed and his ability to work during the commotion, or noisy confusion, of the subway at rush hour. The noisy confusion of the subway was similar to the tumultuous, or chaotic and disorderly, conditions the correspondent was used to, so the commotion was not an impediment to his creativity.

Explanation for Incorrect Answer A : Choice (A) is incorrect. Squalid means dirty and neglected. A boon is a benefit or favor. If one were to insert these terms into the text, the sentence would read Since the foreign correspondent was accustomed to completing his assignments under squalid conditions, the commotion of the subway at rush hour presented no boon for his creativity. The term Since indicates that there is a relationship between the conditions to which the correspondent was accustomed and his ability to work during the commotion, or noisy confusion, of the subway at rush hour. Because there is not necessarily any connection between squalid, or dirty and neglected, conditions and noisily confusing conditions, there is no reason to believe that the commotion of the subway would or would not have been a boon, or benefit, to the correspondents creativity.

nR

(A)

Explanation for Incorrect Answer D : Choice (D) is incorrect. Destructive means causing harm and destroying. A demonstration is a public display. If one were to insert these terms into the text, the sentence would read Since the foreign correspondent was accustomed to completing his assignments under destructive conditions, the commotion of the subway at rush hour presented no demonstration of his creativity. Although the first term makes sense in this context, the second term does not. While the commotion, or noisy confusion, of the subway could affect the correspondent s ability to be creative, it is illogical to suggest that a commotion could demonstrate his creativity.

Explanation for Incorrect Answer E : Choice (E) is incorrect. Flagrant means obviously offensive. A benefit means an advantage or something that is useful. If one were to insert these terms into the text, the sentence would read Since the foreign correspondent was accustomed to completing his assignments under flagrant conditions, the commotion of the subway at rush hour presented no benefit to his creativity. Although the second term makes sense in this context, the first does not. It is illogical to describe a condition as flagrant, or obviously offensive.

Unlike the ------- presentations of the other students in class, Mary Catherines presentation demonstrated a thorough and mature grasp of the material.

astute

(B)

sophomoric

(C) incredulous

(D) cloying

(E)

scintillating

ANSWERS

Explanation for Correct Answer B : Choice (B) is correct. Sophomoric means very immature and poorly informed.

eg

AND EXPLANATIONS

is

te re

Explanation for Incorrect Answer B : Choice (B) is incorrect. Tranquil means calm and free from disturbance. An obstruction is something that blocks or gets in the way of something else. If one were to insert these terms into the text, the sentence would read Since the foreign correspondent was accustomed to completing his assignments under tranquil conditions, the commotion of the subway at rush hour presented no obstruction to his creativity. A correspondent who is used to working under tranquil, or calm, conditions would likely find the commotion, or noisy confusion, of the subway to be an obstruction to his creativity. Therefore, it is illogical to suggest that the commotion presented no obstruction to, or did not get in the way of, his creativity.

file://E:\\e8.htm

2006-11-12

The Official SAT Online Course

5/18

If one were to insert this term into the text, the sentence would read Unlike the sophomoric presentations of the other students in class, Mary Catherines presentation demonstrated a thorough and mature grasp of the material. The term Unlike indicates that the presentations of the other students differed from Mary Catherines in that they did not demonstrate a thorough, or complete, and mature grasp of the material. Presentations that did not demonstrate a mature and complete grasp of the material would most likely be described as sophomoric, or immature and poorly informed.

Explanation for Incorrect Answer A : Choice (A) is incorrect. Astute means shrewd or clever. If one were to insert this term into the text, the sentence would read Unlike the astute presentations of the other students in class, Mary Catherines presentation demonstrated a thorough and mature grasp of the material. The term Unlike indicates that the presentations of the other students differed from Mary Catherines in that they did not demonstrate a thorough and mature grasp of the material. It is unlikely that presentations that did not demonstrate a thorough, or complete, grasp of the material would be described as astute, or shrewd and clever.

Explanation for Incorrect Answer E : Choice (E) is incorrect. Scintillating means brilliantly lively and witty. If one were to insert this term into the text, the sentence would read Unlike the sophomoric presentations of the other students in class, Mary Catherines presentation demonstrated a thorough and mature grasp of the material. The term Unlike indicates that the presentations of the other students differed from Mary Catherines in that they did not demonstrate a mature and thorough grasp of the material. The term scintillating is somewhat illogical in this context, because a brilliantly lively presentation could certainly demonstrate a mature and complete understanding of the material.

nR

(A)

Sadly, the author never ------- the rewards of literary success during her lifetime; public recognition and appreciation of her talent were completely -------.

predicted . . conclusive

(B)

reaped . . posthumous

(C) acknowledged . . fulsome

(D) appreciated . . gratuitous

(E)

pursued . . discredited

ANSWERS

Explanation for Correct Answer B : Choice (B) is correct. To reap means to obtain. Posthumous means following or occurring after death. If one were to insert these terms into the text, the sentence would read Sadly, the author never reaped the rewards of literary success during her lifetime; public recognition and appreciation of her talent were completely posthumous. The semicolon indicates that the idea presented in the second part of the sentence will provide evidence in support of the idea presented in the first part of the sentence. If public recognition and appreciation of the

eg

AND EXPLANATIONS

is te

Explanation for Incorrect Answer D : Choice (D) is incorrect. Cloying means excessively sweet and sentimental. If one were to insert this term into the text, the sentence would read Unlike the cloying presentations of the other students in class, Mary Catherines presentation demonstrated a thorough and mature grasp of the material. The term Unlike indicates that the presentations of the other students differed from Mary Catherines in that they did not demonstrate a mature and thorough grasp of the material. The term cloying is somewhat illogical in this context, because an excessively sweet and sentimental presentation could still demonstrate a mature and complete understanding of the material.

re

Explanation for Incorrect Answer C : Choice (C) is incorrect. Incredulous means skeptical or expressing disbelief. If one were to insert this term into the text, the sentence would read Unlike the incredulous presentations of the other students in class, Mary Catherines presentation demonstrated a thorough and mature grasp of the material. Although the students could be incredulous themselves, it is illogical to say that the presentations were incredulous or skeptical.

file://E:\\e8.htm

2006-11-12

The Official SAT Online Course

6/18

authors talent were completely posthumous, or following her death, it makes sense to say that the author never reaped, or obtained, the rewards of literary success during her lifetime.

Explanation for Incorrect Answer A : Choice (A) is incorrect. To predict means to foresee something. Conclusive means related to an outcome or conclusion. If one were to insert these terms into the text, the sentence would read Sadly, the author never predicted the rewards of literary success during her lifetime; public recognition and appreciation of her talent were completely conclusive. Although it is plausible to suggest that the author never predicted, or foresaw, that she would achieve literary success in her lifetime, it does not make sense to say that the recognition and appreciation she received were conclusive, or related to a conclusion.

Explanation for Incorrect Answer C : Choice (C) is incorrect. To acknowledge means to take notice of something. Affected means a quality of behavior that is not natural to oneself. If one were to insert these terms into the text, the sentence would read Sadly, the author never acknowledged the rewards of literary success during her lifetime; public recognition and appreciation of her talent were completely affected. The semicolon indicates that the idea presented in the second part of the sentence will provide evidence in support of the idea presented in the first part of the sentence. It is reasonable to suggest that the author never acknowledged, or took notice of, the rewards of literacy success, but this idea is not logically connected to the idea that the recognition she received was unnatural to those who recognized her talent.

Explanation for Incorrect Answer E : Choice (E) is incorrect. To pursue means to seek. Discredited means not accepted as true or accurate. If one were to insert these terms into the text, the sentence would read Sadly, the author never pursued the rewards of literary success during her lifetime; public recognition and appreciation of her talent were completely discredited. The semicolon indicates that the idea presented in the second part of the sentence will provide evidence in support of the idea presented in the first part of the sentence. Although it is plausible to suggest that the author never pursued, or sought, the rewards of literary success, this idea is not logically connected to the idea that the public recognition and appreciation she received was discredited. Additionally, it is illogical to suggest that recognition and appreciation of an authors talent would not be accepted as true.

These two passages discuss the same scholarly book, Strangers from a Different Shore by Asian American historian Ronald Takaki. The first passage is a critique of the book written by another Asian American historian, Li Ling-chi Wang; the second is a defense of the book by its author.

Passage 1

In terms of its approach, Takakis book is similar to Victor and Bret Nees Longtime Californ (1972) because both Line are media through which diverse Asian 5 American voices surface. The major difference is in how they collected and use the voices. The Nees identified and

nR

eg

is

Explanation for Incorrect Answer D : Choice (D) is incorrect. To appreciate means to recognize with gratitude. Gratuitous means unearned or unwarranted. If one were to insert these terms into the text, the sentence would read Sadly, the author never appreciated the rewards of literary success during her lifetime; public recognition and appreciation of her talent were completely gratuitous. The semicolon indicates that the idea presented in the second part of the sentence will provide evidence in support of the idea presented in the first part of the sentence. It is reasonable to suggest that the author never appreciated, or recognized with gratitude, the rewards of literary success and that the recognition and appreciation she received were gratuitous, or unearned, but there is no logical connection between these ideas.

te re

file://E:\\e8.htm

2006-11-12

The Official SAT Online Course

7/18

10

15

20

25

30

40

45

50

55

nR

eg

35

selected representative personalities from different segments of the Chinese American community in San Francisco, whom they laboriously interviewed themselves over a two-year period, and placed these lengthy, in-depth interviews largely verbatim in their book with minimal narrative and analysis. From the analytical standpoint, the book broke no new ground. But it succeeded in giving us the sights, sounds, flavors, perspectives, and feelings of the community that had never before been permitted to surface. For their labor, the book received lavish reviews by the mainstream press. Takakis approach is virtually identical with that of the Nees except the voices used tend to be fragmentary and mostly collected from indirect sources. To begin with, no attempt was made to distinguish voices and quotes from literary works from voices of oral histories, documents, and newspapers. Unlike in the Nees book, only a very tiny percentage of the voices, probably less than 15 percent, are based on personal interviews by Takaki and of these, most are interviews of the authors relatives and colleagues. In this respect, the author depends almost entirely on available primary and secondary sources, leaving readers wondering how representative they are. Also, unlike the Nees, Takakis book offers no analytical breakthrough for our understanding of either Asian American history or the newly emerging communities. Particularly disappointing is Takakis silence on the historic emergence of Asian America in the late 1960s and early 1970s and his ahistorical approach to the ensuing decades in Asian American history. The rise of ethnic and political consciousness, the emergence of new community organizations, and the rise of Asian American studies are among the most important developments in the history of Asians in the United States, in my view. Unfortunately both Takaki and Roger Daniels [in the latters Asian America: Chinese and Japanese in the

is te re

file://E:\\e8.htm

2006-11-12

The Official SAT Online Course

8/18

60 U.S. since 1850] failed to adequately treat

this period of Asian America that gave them the titles for their books, a term that captures the richness of the newly emerged Asian American political and 65 cultural expressions. In this respect, book reviewers are wrong in characterizing Takakis book as a comprehensive history of 150 years of Asian experience in the U.S. It is more 70 accurate to describe the book as a history of Asians in the U.S. up to World War II, with the final two chapters of anecdotal events since then tacked on as an epilogue.

Passage 2
75

Comparing Strangers from a Different Shore to Victor and Brett Nees Longtime Californ, Wang points out that I did not collect the voices myself. However my study is a very different one from the 80 Nees. Theirs studied only one ethnic group and only one geographical communitySan Francisco Chinatown. Mine is comparative and also far more comprehensive: it studies the Chinese, 85 Japanese, Koreans, Filipinos, Asian Indians, VietnameseThus, given the tremendous scope of my study, I had to collect most of the voices from indirect sources such as the Ethnic Studies Oral 90 History Project, and I acknowledged them through the study. But I also directly interviewed people. One of them, Touly Xiong, a Hmong refugee now living in Wisconsin, 95 told me: We hope you will include our stories in your book. Americans need to understand us and what we have gone through. He said that he had fought for the US in Laos and that his brother had 100 been killed by North Vietnamese soldiers. I also interviewed many of my relatives. One of the meaningful things I discovered and I hope other Asian American scholars will learn is that our family histories are 105 tied to the histories of our communities. We, as scholars, are members of communities, and our aunts, uncles, and cousins have stories and voices that

nR

eg i

st

er

ed

file://E:\\e8.htm

2006-11-12

The Official SAT Online Course

9/18

As used in line 1, approach most nearly means


(A)

beginning

(B)

technique

(D) advance

(E)

style

nR
ANSWERS

Explanation for Correct Answer B : Choice (B) is correct. In context, approach most nearly means technique. The author of Passage 1 is comparing two books, saying that both are media through which diverse Asian American voices surfaceclearly a reference to both writers technique, or methodology for treating their similar subject matter.

Explanation for Incorrect Answer A : Choice (A) is incorrect. The author of Passage 1 is clearly comparing two books in their entirety in terms of how they handle their subject matterin other words, their technique, or procedure. The author does not address whether or not the two books begin in the same manner. Their beginnings may or may not be similar.

Explanation for Incorrect Answer C : Choice (C) is incorrect. In comparing two books, the author of Passage 1 is addressing their similar ways of handling their subject matterin other words, their technique, or procedure. The author does not address whether or not the two books share a hypothesis, which in this context would mean a philosophical assumption. The paragraph goes on to compare the books in terms of what they do, not in terms of the assumptions they may or may not share.

Explanation for Incorrect Answer D : Choice (D) is incorrect. While approach as a verb may have some connection to advance as a verb, it does not make sense to say that the two books have similar advances. In this section, the author of Passage 1 is clearly referring to methods or procedures that the authors of both books sharein other words, their

eg

(C) hypothesis

AND EXPLANATIONS

is te

re

belong to Asian American history. 110 But what about Wangs claim that I failed to adequately treat the post-World War II period? I wish I had given more attention to the 1950s, but I felt that the book was already an 115 enormously long one and also that I wanted to focus more on the post-1965 period. And here it is not accurate to describe my book as a history of Asians in the US up to World War II. The last 120 two chapters (out of a total of twelve chapters) are devoted to exactly this time period. As any reader will see, they are not chapters of anecdotal events. . . tacked on as an epilogue, as Wang 125 claims. Rather they are designed to relate the early period to the more recent history in a substantive way. They present an analysis of post-World War II developments.These certainly are not 130 anecdotal events.

file://E:\\e8.htm

2006-11-12

The Official SAT Online Course

10/18

techniques.
Explanation for Incorrect Answer E : Choice (E) is incorrect. The author of Passage 1 is clearly speaking here of the two books similar technique, or methods of handling their subject matter. They may or may not be similar in style, which would more likely refer to literary elements than to scholarly methodology, or approach.

In context, surface (line 20) is closest in meaning to


(A)

arrange

(B)

level

(C) float

(D) emerge

(E)

smooth

ANSWERS

AND EXPLANATIONS

nR

(A)

Explanation for Incorrect Answer B : Choice (B) is incorrect. The author of the passage is speaking here about the information that the Nees book has brought to light, or has allowed to emerge. In this context, it makes little sense to say that the voices in the book have been permitted to level, either in the sense of equalize or tear down.

Explanation for Incorrect Answer C : Choice (C) is incorrect. While an object that has been submerged might be said to float to the surface, the author clearly is not speaking so literally here. Rather, he is speaking of previously obscure voices that have been allowed to emerge, or become evident.

Explanation for Incorrect Answer E : Choice (E) is incorrect. The author of the passage is speaking here about the information that the Nees book has brought to light, or has allowed to emerge. In this context, it makes little sense to say that the voices in the book have been permitted to smooth, or to be become level or flat.

In paragraph 2, the author of Passage 1 implies that a major difference between the Nees book and Takakis book is that

the Nees book draws extensively on interviews conducted by the authors

(B)

Takakis study focuses on U.S. citizens with roots in only one Asian nation

(C) the Nees make no significant analytical advances

(D) Takaki places great emphasis on the rise of Asian American studies

(E)

the Nees book uses quotations from literary works and oral histories

ANSWERS

eg

AND EXPLANATIONS

is

Explanation for Incorrect Answer A : Choice (A) is incorrect. The author of the passage is speaking here about the information that the Nees book has brought to light, or has allowed to emerge. In this context, it makes little sense to say that the voices in the book have been permitted to arrange, or come to an agreement.

te

re

file://E:\\e8.htm

Explanation for Correct Answer D : Choice (D) is correct. The author is saying here that the Nees book provided information that had never before been disclosed about a certain community. It is clear from context (never before been permitted to) that this information has been willfully or deliberately kept obscure; in revealing this information, the Nees have allowed it to emerge, or come forth from obscurity.

2006-11-12

The Official SAT Online Course

11/18

Explanation for Correct Answer A : Choice (A) is correct. In the second paragraph, the author of Passage 1 contrasts Takakis book with the Nees book, saying that the former relies on voices that are mostly collected from indirect sources and, further, that Takaki, unlike the Nees, acquired only a very tiny percentage of his sources from direct interviews. Together, these comparisons imply that, in the view of the author, the Nees rely heavily on interviews that they themselves conducted, an area in which the author considers Takaki somewhat deficient.

Explanation for Incorrect Answer B : Choice (B) is incorrect. In this paragraph, the author does not address the Asian roots of the people interviewed by either Takaki or the Nees; rather, he focuses on what he perceives as the shortage of direct interviews undertaken by Takaki in the preparation of the book.

Explanation for Incorrect Answer C : Choice (C) is incorrect. Though in the first paragraph the author says that the Nees book broke no new analytical ground, he does not address this subject in paragraph 2. Further, the author nowhere implies that analytical advances are made by Takaki and not by the Nees, an assumption that underlies option C.

In context, distinguish (line 28) most nearly means


(A)

mark

nR
(D) judge

(B)

differentiate

(C) analyze

(E)

discover

ANSWERS

Explanation for Correct Answer B : Choice (B) is correct. In this part of the passage, the author is accusing Takaki of neglecting to distinguish one kind of literary voice from another kind in his book; he means that Takaki does not differentiate, or make a distinction, among quotations from literary works and quotations from documentary sources.

Explanation for Incorrect Answer A : Choice (A) is incorrect. Although mark can be said to mean distinguish, in the context of this passage the author is not speaking of physically marking different kinds of quotations in order to keep them distinct; rather, he is speaking of making a distinction among different kinds of quotations in the text of a book.

Explanation for Incorrect Answer C : Choice (C) is incorrect. In this part of the passage, the author is accusing Takaki of neglecting to distinguish, or differentiate, among various types of quotations in the text of his book. It does not make logical sense or syntactic sense to say that Takaki fails to analyze, or methodically examine, one type of quotation or source from another type.

Explanation for Incorrect Answer D :

eg

10

AND EXPLANATIONS

is te

Explanation for Incorrect Answer E : Choice (E) is incorrect. While the author does mention literary works and oral histories in the second paragraph, he attributes their use to Takaki, not to the Nees. This paragraph is devoted to what the author sees as Takakis books deficiencies, among them a reluctance to differentiate among quotations from literary works and quotations from oral histories.

re

Explanation for Incorrect Answer D : Choice (D) is incorrect. Elsewhere in the passage, the author implies that Takaki has placed insufficient emphasis on the rise of the field of Asian American studies. The author does not address this topic at all in the second paragraph of the passage.

file://E:\\e8.htm

2006-11-12

The Official SAT Online Course

12/18

Choice (D) is incorrect. In this part of the passage, the author is accusing Takaki of neglecting to distinguish, or differentiate, among various types of quotations in the text of his book. Although judge can be said to mean distinguish, it is clear from context that the author would have preferred that Takaki make some sort of distinction among various types of quotations, not that Takaki judge, or evaluate, such types.

Explanation for Incorrect Answer E : Choice (E) is incorrect. In this part of the passage, the author is accusing Takaki of neglecting to distinguish, or differentiate, among various types of quotations in the text of his book. It does not make logical sense or syntactic sense to say that Takaki fails to discover, or learn about for the first time, one type of quotation or source from another type.

11

The author of Passage 1 mentions Roger Daniels primarily as an example of


(A)

an historian who wrote an essay critical of Takakis study

(B)

a scholar who was frequently consulted by Takaki

(C) a writer whose research methods the author admires

(D) an author whose book shares a weakness with Takakis

(E)

someone who assisted Takaki in writing his book

12

nR

(A)

Explanation for Incorrect Answer A : Choice (A) is incorrect. Daniels is mentioned as a scholar whose work, in the opinion of the author, has a shortcoming similar to one found in Takakis work. Nowhere in the passage is he identified as writing anything about Takakis work.

Explanation for Incorrect Answer B : Choice (B) is incorrect. Daniels is mentioned as a scholar whose work, in the opinion of the author, has a shortcoming similar to one found in Takakis work. Nowhere in the passage is he identified as either someone Takaki consulted or someone whose work Takaki consulted in preparing his own bookthe latter is likely, of course, but it is not mentioned in the passage.

Explanation for Incorrect Answer C : Choice (C) is incorrect. Daniels is mentioned as a scholar whose work, in the opinion of the author, has a shortcoming similar to one found in Takakis work. If the author of the passage expresses any opinion at all of Daniels work, it is disapproval: [Both books] failed to adequately treat this period.

Explanation for Incorrect Answer E : Choice (E) is incorrect. Daniels is mentioned as a scholar whose work, in the opinion of the author, has a shortcoming similar to one found in Takakis work. The two authors sharing what the author of the passage considers a shortcoming does not imply that the two writers ever worked together, much less assisted one another.

In context, respect (line 66) is closest in meaning to

regard

(B)

appreciation

(C) reference

(D) esteem

eg is te

Explanation for Correct Answer D : Choice (D) is correct. When the author of Passage 1 mentions Roger Daniels in lines 57-65 (Unfortunately . . . expressions), he states that Daniels, along with Takaki, failed to adequately treat this period, that is, a period during which, in the authors view, Asian Americans organized themselves into political and cultural groups. The author considers this omission a shortcoming of both Takakis and Daniels books; in other words, both works share a weakness.

re

ANSWERS

AND EXPLANATIONS

file://E:\\e8.htm

2006-11-12

The Official SAT Online Course

13/18

(E)

recognition

ANSWERS

AND EXPLANATIONS

Explanation for Correct Answer A : Choice (A) is correct. In lines 66 and following, the author of Passage 1 is referring back to the preceding paragraph, where two books are compared in terms of a common flaw. This respect, then, at the beginning of a new paragraph, refers to something like Takakis neglect of Asian American history after World War II, with respect meaning regard, or aspect.

Explanation for Incorrect Answer B : Choice (B) is incorrect. In lines 66 and following, the author of Passage 1 is referring back to the preceding paragraph, where two books are compared in terms of a common flaw. This respect, then, at the beginning of a new paragraph, refers to something like Takakis neglect of Asian American history after World War II. It makes no sense, then, to say In this appreciationor recognition of valuein this particular context.

13

nR

(A)

Explanation for Incorrect Answer E : Choice (E) is incorrect. In lines 66 and following, the author of Passage 1 is referring back to the preceding paragraph, where two books are compared in terms of a common flaw. This respect, then, at the beginning of a new paragraph, refers to something like Takakis neglect of Asian American history after World War II. It makes no sense, then, to say In this recognitionor favorable noticein this particular context.

Which of the following best describes the attitude of the author of Passage 1 toward Takakis book?

disbelieving

(B)

confrontational

(C) exasperated

(D) outraged

(E)

dissatisfied

ANSWERS

Explanation for Correct Answer E : Choice (E) is correct. The author of Passage 1 takes issue with two major elements of Takakis book: its research methodology (paragraph 2); and its treatment of Asian Americans in the U.S. in the 1950s and 1960s (paragraphs 3-5). In using such words as disappointing, unfortunately, failed to treat, and tacked on as an epilogue, the author betrays dissatisfaction, or discontent, with the book. The passage, however, maintains a scholarly detachment in its appraisal of the book, avoiding harsh language and outright attacks.

eg i

Explanation for Incorrect Answer D : Choice (D) is incorrect. In lines 66 and following, the author of Passage 1 is referring back to the preceding paragraph, where two books are compared in terms of a common flaw. This respect, then, at the beginning of a new paragraph, refers to something like Takakis neglect of Asian American history after World War II. It makes no sense, then, to say In this esteemor favorable regardin this particular context.

AND EXPLANATIONS

st

er e

Explanation for Incorrect Answer C : Choice (C) is incorrect. In lines 66 and following, the author of Passage 1 is referring back to the preceding paragraph, where two books are compared in terms of a common flaw. This respect, then, at the beginning of a new paragraph, refers to something like Takakis neglect of Asian American history after World War II. While it might make sense in some contexts to say In this referenceor denotationcritics are wrong, the author makes it clear here that he means respect in the sense of regard, or aspect.

file://E:\\e8.htm

2006-11-12

The Official SAT Online Course

14/18

Explanation for Incorrect Answer A : Choice (A) is incorrect. The author of Passage 1 is clearly not completely satisfied with Takakis book, and finds fault with two of its major elements. But nowhere does the author say or imply that Takaki is less than truthful in his reporting or that he is withholding the truth, so the authors attitude cannot be called disbelieving.

Explanation for Incorrect Answer B : Choice (B) is incorrect. The author of Passage 1 is clearly not completely satisfied with Takakis book, and finds fault with two of its major elements. But nowhere does the author sound confrontational, or hostilethe author does not, for example, challenge Takaki to defend his sources or his methods, which would be confrontational.

Explanation for Incorrect Answer C : Choice (C) is incorrect. While the author of the passage is clearly not completely satisfied with Takakis book, and finds fault with two of its major elements, he nowhere sounds exasperated, or angrily impatient. Throughout the passage, the tone is one of scholarly detachment, avoiding harsh language and outright attacks.

Explanation for Incorrect Answer D : Choice (D) is incorrect. While the author of the passage is clearly not completely satisfied with Takakis book, and finds fault with two of its major elements, he nowhere sounds outraged, or grossly offended. Throughout the passage, the tone is one of scholarly detachment, avoiding harsh language and outright attacks.

14

The author of Passage 2 asserts that the Nees book is inferior to Strangers from a Different Shore in terms of its
(A)

purpose

(B)

readability

(D) accuracy

(E)

range

nR

ANSWERS

Explanation for Correct Answer E : Choice (E) is correct. In the first paragraph of Passage 2, Takaki contrasts Strangers from a Different Shore with Longtime Californ, pointing out that the former studied only one ethnic group and only one geographical community, while the latter is comparative and also far more comprehensive; three lines later, he refers to the tremendous scope of my study. Thus, Takaki is comparing his book to the Nees book in terms of its scope, or range of material covered, and considers his superior.

Explanation for Incorrect Answer A : Choice (A) is incorrect. The author of Passage 2 does not really mention purpose in regard to either book. He does say that Strangers from a Different Shore is different, and more comprehensive, but he is basically addressing range or scope rather than purpose with these descriptions.

Explanation for Incorrect Answer B : Choice (B) is incorrect. The passage does not compare the two books in terms of their relative readability, or ease or pleasure with which they can be read.

Explanation for Incorrect Answer C : Choice (C) is incorrect. While the author of Passage 2 does discuss matters of documentationin, for example, lines 86-91nowhere in the passage does he assert or imply that his methods are superior to those of the Nees, merely that the scope of his work necessitated a different kind of documentation.

Explanation for Incorrect Answer D : Choice (D) is incorrect. The author of Passage 2 does mention accuracy in the last paragraph of the passage, but it is in connection with Li Ling-chi Wang, the

eg

AND EXPLANATIONS

is

(C) documentation

te

re

d
file://E:\\e8.htm

2006-11-12

The Official SAT Online Course

15/18

author of Passage 1, and not with the Nees work.

15

The author of Passage 2 mentions the Ethnic Studies Oral History Project (lines 8990) primarily in order to
(A)

cite an organization that praised his study

(B)

highlight a valuable resource overlooked by the Nees

(C) defend the validity of his use of secondary sources

(D) acknowledge the assistance he received from earlier scholars

(E)

refer to a study he conducted earlier in his career

ANSWERS

AND EXPLANATIONS

Explanation for Correct Answer C : Choice (C) is correct. In this part of the passage, the author is speaking of the tremendous scope of his book, which necessitated his using indirect sources such as the Ethnic Studies Oral History Project rather than relying solely on direct interviews. He is also defending himself against the charge made by the author of Passage 1: that he did not conduct enough personal interviews. Therefore, he is citing the Ethnic Studies Oral History Project by way of defending his use of it and of other secondary sources.

16

nR

(A)

Explanation for Incorrect Answer D : Choice (D) is incorrect. While it can be inferred that Takaki and other scholars draw upon the work of their predecessors, there is no direct mention in the passage of Takakis receiving assistance nor any acknowledgement of the Ethnic Studies Oral History Project as the work of earlier scholars who might have been in a position to provide assistance.

Explanation for Incorrect Answer E : Choice (E) is incorrect. There is no indication in the passage that Takaki was in any way involved with the authorship of the Ethnic Studies Oral History Project, only that he drew on it in documenting Strangers from a Different Shore.

The author of Passage 2 uses the direct quotation in lines 95-98 most probably in order to

discredit an assertion made in the Nees book

(B)

demonstrate that his book has gained a wide readership

(C) provide evidence that indirect sources are as valid as direct sources

(D) introduce claims about an ethnic group not discussed in his book

(E)

counter a criticism made by the author of Passage 1

ANSWERS

Explanation for Correct Answer E : Choice (E) is correct. The quotation, from a Hmong refugee, Touly Xiong, is provided in part to support the claim, But I also directly interviewed people. This assertion is, in turn, made to refute a claim made by the author of Passage 1, paraphrased by Takaki as Wang points that I did not collect the voices myself (lines 77-78). So including the direct quotation is intended to counter, or

eg

Explanation for Incorrect Answer B : Choice (B) is incorrect. While it is clear from context that Takaki considers the Ethnic Studies Oral History Project a valuable resourcehaving used it himselfthere is no indication that the Nees either used it or did not use it in preparing their study.

AND EXPLANATIONS

is

te

Explanation for Incorrect Answer A : Choice (A) is incorrect. There is no indication in the passage that the Ethnic Studies Oral History Project either praised or criticized Takakis study, only that Takaki himself drew on the Project in his work.

re

d
file://E:\\e8.htm

2006-11-12

The Official SAT Online Course

16/18

refute, a criticism made by the author of Passage 1 (that Takaki did not conduct interviews).

Explanation for Incorrect Answer A : Choice (A) is incorrect. While Takaki states in Passage 2 that the Nees book is less ambitious than his own (lines 80-84), he nowhere discredits any of the Nees assertions. The quotation in lines 95-98 can be seen as an attempt to discredit an assertion, but it is an assertion made in the passage, not the Nees book.

Explanation for Incorrect Answer B : Choice (B) is incorrect. The quotation in lines 95-98 is from a person interviewed by Takaki in the preparation of the latters book; there is no indication that the person quoted has read the book, nor is there any indication anywhere in the passage about the scope of Takakis audience.

Explanation for Incorrect Answer C : Choice (C) is incorrect. While in the paragraph preceding this one Takaki defends the use of indirect sources, in lines 95-98 he is quoting a person he actually interviewedin other words, addressing the validity of direct sources.

17

With the parenthetical information in lines 120-121 Takaki intends primarily to


(A)

imply that his book is a relatively lengthy one

(B)

illustrate the significance the post-World War II era has in his book

(C) tell readers where his treatment of the post-World War II era can be found

(D) compare his book to a previous book about the post-World War II era

(E)

describe the arrangement of chapters in his book

nR

ANSWERS

Explanation for Correct Answer B : Choice (B) is correct. In this paragraph, the author is defending his book against Wangs charge that he failed to adequately treat the post-World War II period. In lines 118-119, he quotes Wangs description of Takakis book as a history of Asians in the US up to World War II, then insists that the last two chapters (out of a total of twelve chapters) are devoted to exactly this time periodthat is, the years after World War II. Thus, by saying exactly how much of his book is devoted to this period, Takaki is using the parenthetical information to express how significant this period is to his book as a whole.

Explanation for Incorrect Answer A : Choice (A) is incorrect. The parenthetical information tells how much of Takakis booktwo out of twelve chaptersis devoted to a certain topic. Though he does indicate in lines 114-115 that his book is enormously long, he is not using the information in parenthesis to illustrate the overall length of the book.

Explanation for Incorrect Answer C : Choice (C) is incorrect. While the paragraph as a whole suggests that Takakis treatment of post-World War II Asian American life comes near the end of his book, the parenthetical information is clearly intended to illustrate how great a proportion of the entire book is taken up with this topic, not where in the book the topic can be found.

Explanation for Incorrect Answer D : Choice (D) is incorrect. The parenthetical information in no way refers to any other book; rather, Takaki is here referring to how much of his own booktwo out of twelve chaptersis concerned with a single topic, Asian American history after World War II.

eg is

AND EXPLANATIONS

te re

Explanation for Incorrect Answer D : Choice (D) is incorrect. While Touly Xiong, the person quoted in lines 95-98, is identified as a Hmong refugee who has left Asia and now lives in the United States, it is obvious from context that Takaki is using the quotation to refer to an ethnic group that is discussed in Strangers from a Different Shore.

file://E:\\e8.htm

2006-11-12

The Official SAT Online Course

17/18

Explanation for Incorrect Answer E : Choice (E) is incorrect. While Takaki does say in this paragraph that the chapters of his book that deal with the post-World War II period come at the end of the book, he is clearly using the parenthetical information to demonstrate what proportion of his book is devoted to the topic.

18

The author of Passage 2 uses quotation marks in line 130 most likely in order to
(A)

indicate that he is using a common expression

(B)

highlight an important concept

(C) cite a judgment with which he disagrees

(D) emphasize a humorous statement

(E)

call attention to an unusual phrase

ANSWERS

AND EXPLANATIONS

Explanation for Incorrect Answer A : Choice (A) is incorrect. Quotation marks are more often used to indicate an uncommon expression than a common one; but in this context, it is clear that Takaki is quoting another authors work, and that he takes issue with it.

19

nR

(A)

Explanation for Incorrect Answer D : Choice (D) is incorrect. Since Takaki is quoting a negative assessment of his work here, it is unlikely that he is doing so in a humorous way. The tone of the paragraph as a whole does not support this interpretation.

Explanation for Incorrect Answer E : Choice (E) is incorrect. While quotation marks are often used to call attention to an unusual phrase, it is clear from context that Takaki is refuting a judgment with which he disagrees; he does, after all, quote the judgment just above, in lines 122125.

The authors of Passage 1 and Passage 2 would most probably agree with one another about which of the following statements concerning scholarly studies of Asian American communities?

Asian American scholars should focus on earlier immigrant groups rather than on those who immigrated later.

(B)

Oral histories are less credible than is written documentation of Asian American history.

(C) Interviewing ones own family members is an effective way to represent the voices of ones community.

(D) Post-World War II Asian American history is just as important as the history of earlier periods of Asian American history.

(E)

A study that focuses solely on San Francisco can treat the history of Asian American immigrants in a sufficiently thorough way.

ANSWERS

Explanation for Correct Answer D :

eg

Explanation for Incorrect Answer B : Choice (B) is incorrect. While quotations marks might feasibly be used to highlight an important concept, it is clear in this context that Takaki is quoting another authors work, and that he takes issue with it.

AND EXPLANATIONS

is te re d

Explanation for Correct Answer C : Choice (C) is correct. In this paragraph, Takaki is refuting Li Lin-chi Wangs contention that Takakis book underemphasizes an important period of Asian American history. In lines 122-124, he quotes Wang's description of the book's last two chapters as "Chapters of anecdotal events...tacked on as an epilogue." Takaki, in lines 127-130, insists that the chapters are more substantive than Wang claims, directly quoting Wangs judgment in order to disagree with it.

file://E:\\e8.htm

2006-11-12

The Official SAT Online Course

18/18

Choice (D) is correct. In Passage 1, Wang criticizes Takaki for underplaying the post-World War II era in Strangers from a Different Shore, terming the events of the late 1960s and early 1970's among the most important developments in the history of Asians in the United States (lines 53-55). While Takaki obviously disagrees with Wangs assessment of the book, he does acknowledge the importance of the period, insisting in the last paragraph of Passage 2 that two out of twelve chapters of his book are dedicated to the period. Thus, it can be inferred that both authors agree about the importance of the post-World War II period in the study of Asian American history.

Explanation for Incorrect Answer A : Choice (A) is incorrect. As it happens, both passages spend more time discussing relatively recent immigrants, but there is no indication in either passage that the authors consider earlier immigrants more worthy of study then later immigrants, or vice versa.

Explanation for Incorrect Answer B : Choice (B) is incorrect. Far from considering oral histories less credible than written sources, both authors seem convinced of the importance of oral histories: Wang chides Takaki for failing to distinguish written histories from oral histories and for underusing oral interviews (Passage 1, paragraph 2), and Takaki defends at some length his use of oral histories (Passage 2, paragraph 2).

Explanation for Incorrect Answer E : Choice (E) is incorrect. Both passages mention Longtime Californ by Victor and Bret Nee as a study that focuses solely on Chinese immigrants in San Francisco, but neither Wang nor Takaki says or implies that the book is thoroughly satisfactory: Wang says that it broke no new [analytical] ground (lines 16-17), while Takaki compares his book favorably to the Nees in terms of scope (lines 80-86).

nR

eg i

st er

ed

Privacy Policy

Explanation for Incorrect Answer C : Choice (C) is incorrect. While Takaki defends the scholarly usefulness of interviewing family members (our aunts, uncles, and cousins have stories and voices that belong to Asian American history), Wang seems unenthusiastic about the practice, critically stating that most [of Takakis interviews] are of the authors relatives and colleagues.

Back to Score Report

Copyright 2006 The College Board. All rights reserved.

Terms of Use

Contact Us

file://E:\\e8.htm

2006-11-12

The Official SAT Online Course

1/14

Help | Profile | My Organizer | My Bookmarks | Logout

Answers and Explanations

Test Sections

Back to Score Report

Section 1

View Answers and Explanations


Online - Practice Test #5

Section 2

Section 3

Section 4

Section 5

Section 6

Section 8

Section 9

Section 10

Of the labeled points on the cube shown above, which is farthest from point
(A)
(B)
(C)
(D)
(E)

ANSWERS

nR

Explanation for Correct Answer C : is the length of each edge of the cube, then points Choice (C) is correct. If

eg i

AND EXPLANATIONS

st er

are each distance

ed

The line segments

and

from point

and

are

the hypotenuses of right triangles with both legs of length and and that between points and between points

so the distance must each be

greater than

Furthermore, the line segment between points

and

is the

hypotenuse of the right triangle with legs

and

so the distance between

points

and

must be greater than the distance between points

and

Therefore, of the labeled points, point

is farthest from point

Explanation for Incorrect Answer A : is one of the labeled points that is closest to Choice (A) is not correct. Point The question asks for the labeled point that is farthest from point

Explanation for Incorrect Answer B : Choice (B) is not correct. The line segment between points

and

is the

hypotenuse of the right triangle with legs

and

so the distance between

points

and

must be greater than the distance between points

and

Explanation for Incorrect Answer D : Choice (D) is not correct. The line segment between points

and

is the

hypotenuse of the right triangle with legs

and

so the distance between

points

and

must be greater than the distance between points

and

Explanation for Incorrect Answer E : is one of the labeled points that is closest to Choice (E) is not correct. Point

file://E:\\e9.htm

2006-11-12

The Official SAT Online Course

2/14

point

The question asks for the labeled point that is farthest from

In a scale model of a statue,

inch represents

inches. If the statue is

inches tall, what is the height of the scale model?

(A)

inches

(B)

inches

(C)

inches

(D)

inches

(E)

inches

te re

is

Explanation for Correct Answer A :

Choice (A) is correct. If

inch on the model represents

d
where

ANSWERS

AND EXPLANATIONS

inches on the statue,

and if the statue is

inches tall, then

is the height, in

nR eg

Explanation for Incorrect Answer B : Choice (B) is not correct. If the scale model were

inches tall, then the proportion

is
But

inches, of the scale model of the statue. Solving this proportion for

gives

inches tall and the statue were

would hold. Solving this proportion

gives

not

Explanation for Incorrect Answer C :

Choice (C) is not correct. If the scale model were

inches tall and the statue

were

inches tall, then the proportion

would hold. Solving this

proportion gives

But

is

not

Explanation for Incorrect Answer D :

Choice (D) is not correct. If the scale model were

inches tall and the statue

were

inches tall, then the proportion

would hold. Solving this

file://E:\\e9.htm

2006-11-12

The Official SAT Online Course

3/14

proportion gives

But

is

not

Explanation for Incorrect Answer E : Choice (E) is not correct. If the scale model were

inches tall and the statue

were

inches tall, then the proportion

would hold. Solving this

proportion gives

But

is

not

te re
intersect. If

In the figure above, lines

and

(B)
(C)
(D)
(E)

nR

ANSWERS

Explanation for Correct Answer D : Choice (D) is correct. The angled labeled

eg

AND EXPLANATIONS
and the angle consisting of all three

angles labeled

are vertical angles, so

Explanation for Incorrect Answer A : Choice (A) is not correct. This choice,

is

(A)


what is the value of

is too small. It could be the result of

visually estimating the answer, but the question does not ask for an estimate.

Explanation for Incorrect Answer B : is too small. It could be the result of Choice (B) is not correct. This choice, visually estimating the answer, but the question does not ask for an estimate.

Explanation for Incorrect Answer C : is too small. It could be the result of Choice (C) is not correct. This choice, visually estimating the answer, but the question does not ask for an estimate.

Explanation for Incorrect Answer E : is too large. It could be the result of Choice (E) is not correct. This choice, visually estimating the answer, but the question does not ask for an estimate.

d
file://E:\\e9.htm

2006-11-12

The Official SAT Online Course

4/14

If

is
(A)
(B)
(C)
(D)
(E)

more than twice

what is

in terms of

ANSWERS

AND EXPLANATIONS
is

Explanation for Correct Answer E : Choice (E) is correct. The statement

more than twice

can be written

as

Subtracting

from both sides of this equation gives

now dividing both sides of the resulting equation by

yields

Therefore,

st er

were equal to

Explanation for Incorrect Answer A : were equal to Choice (A) is not correct. If

which is not equivalent to

more than twice

Explanation for Incorrect Answer B : were equal to Choice (B) is not correct. If

which is not equivalent to

more than twice

Explanation for Incorrect Answer C :

eg i

Choice (C) is not correct. If

nR

which is not equivalent to

more than twice

Explanation for Incorrect Answer D :

Choice (D) is not correct. If

were equal to

which is not equivalent to

more than twice

students, the amount of time the The scatterplot above shows, for each of student spent on homework plotted against the amount of time the student spent on

ed

then


would be equal to

then

would be equal to

then

would be equal to

then

would be equal to

file://E:\\e9.htm

2006-11-12

The Official SAT Online Course

5/14

after-school activities last week. According to the scatterplot, which of the following statements is true?
(A)

Each of the students spent the same amount of time on homework as on after-school activities.

(B)

Each of the students spent more time on homework than on after-school activities.

(C) Each of the students spent less time on homework than on after-school activities.

(D) Exactly

students spent

hours on homework.

(E)

Exactly

students spent

hours on after-school activities.

ANSWERS

AND EXPLANATIONS

Explanation for Correct Answer D : hours on homework Choice (D) is correct. The number of students who spent can be found by counting the number of dots that fall on the vertical line hours of homework. There are three such dots, corresponding to representing students spent hours of after-school activities. Therefore, exactly and

hours on homework.

Explanation for Incorrect Answer B : Choice (B) is not correct. It is not true that each of the students spent more time on homework than on after-school activities. For example, the highest rightmost hours on afterhours on homework and dot represents a student who spent school activities.

nR

Explanation for Incorrect Answer C : Choice (C) is not correct. It is not true that each of the students spent less time on homework than on after-school activities. For example, the lowest leftmost dot hour on after-school hours on homework and represents a student who spent activities.

Explanation for Incorrect Answer E : Choice (E) is not correct. It is not true that exactly

eg

is te

then

re

Explanation for Incorrect Answer A : Choice (A) is not correct. It is not true that each of the students spent the same amount of time on homework as on after-school activities. For example, the lowest hour on hours on homework and leftmost dot represents a student who spent after-school activities.

students spent

hours on

hours on after-school after-school activities. The number of students who spent activities can be found by counting the number of dots on the horizontal line that hours of after-school activities. There are only two such dots, one represents

corresponding to homework.

hours of homework and one corresponding to

hours of

If the function

is defined by

what is the value of

(A)
(B)
(C)
(D)
(E)

ANSWERS

AND EXPLANATIONS

Explanation for Correct Answer B :

Choice (B) is correct. If

file://E:\\e9.htm

2006-11-12

The Official SAT Online Course

6/14

Explanation for Incorrect Answer A :

Choice (A) is not correct.

not

Therefore,

not

Explanation for Incorrect Answer C :

Choice (C) is not correct. The value of

cannot be

because

Explanation for Incorrect Answer D : Choice (D) is not correct. This incorrect answer could be the result of incorrectly

computing the sign of

Explanation for Incorrect Answer E :

Choice (E) is not correct.

not

Therefore,

not

nR
(A)

If the areas of the two rectangles in the figure above are equal, which of the following could be the coordinates of point

(B)

(C)

(D)

(E)

ANSWERS

Explanation for Correct Answer D : is in Quadrant IV and the two rectangles in the Choice (D) is correct. Since is positive, the -coordinate of figure are of equal area, it follows that the is negative, and the absolute value of the product of the coordinate of Of the given choices, only is -coordinate of and the coordinate of

satisfies these three conditions.

Explanation for Incorrect Answer A : is in Quadrant IV, its Choice (A) is not correct. Since

eg

positive. Therefore,

AND EXPLANATIONS

is te

re

cannot be the coordinates of

Explanation for Incorrect Answer B :

-coordinate must be

file://E:\\e9.htm

2006-11-12

The Official SAT Online Course

7/14

Choice (B) is not correct. Since

is in Quadrant IV, its

-coordinate must be

positive and its

-coordinate must be negative. Therefore,

cannot be the

coordinates of

Explanation for Incorrect Answer C : Choice (C) is not correct. The upper rectangle in the figure has area

If the

coordinates of point

were

then the lower rectangle would be of area

However, the question states that the two rectangles must be of equal area.

Explanation for Incorrect Answer E : is in Quadrant IV, its Choice (E) is not correct. Since

-coordinate must be

negative. Therefore,

cannot be the coordinates of

l l l l l

The The The The The

first person ordered a salad. second person did not order a salad. third person ordered a hamburger. fourth person ordered the same thing as the first person. fifth person ordered the same thing as the second person.

(A)

must be

(B)

must be

(C)

must be

(D)

must be

or

(E)

must be

or

ANSWERS

Explanation for Correct Answer E : Choice (E) is correct. The first person ordered a salad. The second ordered either a hamburger or a hot dog. The third ordered a hamburger, and the fourth ordered a salad. The fifth person ordered either a hamburger or a hot dog to match the order of the second person. There are only two possibilities for the number of hamburgers ordered, and they depend on whether the second person ordered a hamburger or a hot dog. If the second person ordered a hot dog, the orders were salad, hot dog, hamburger, salad, hot dog. If the second person ordered a hamburger, the orders were salad, hamburger, hamburger, salad, hamburger. In the first case, the In the second case, the number of people who ordered a hamburger was

nR

not true.

These are the only two number of people who ordered a hamburger was must be true. or must be possible cases, so the statement

Explanation for Incorrect Answer A : Choice (A) is not correct. The number of people who ordered a hamburger may be must be is Therefore, the statement but this number could also be

Explanation for Incorrect Answer B : Choice (B) is not correct. The number of people who ordered a hamburger could not be

Explanation for Incorrect Answer C : Choice (C) is not correct. The number of people who ordered a hamburger may be must be Therefore, the statement but this number could also be

is not true.

Explanation for Incorrect Answer D : Choice (D) is not correct. The number of people who ordered a hamburger could be but this number could not be

eg

AND EXPLANATIONS

is

te

re

A lunch stand has three choices: hamburger, hot dog, or salad. Five people from an office ordered one choice each from the lunch stand. The statements above are about is the number of people who ordered a what these five people ordered. If hamburger, which of the following statements is true?

file://E:\\e9.htm

2006-11-12

The Official SAT Online Course

8/14

If

which of the following could be the value of

(A)
(B)
(C)
(D)
(E)

ANSWERS

AND EXPLANATIONS

Explanation for Correct Answer E :

Choice (E) is correct. If

then

Thus, the possible values for

are

and

Therefore, of the choices given, only

Explanation for Incorrect Answer A :

er ed
then

Choice (A) is not correct. If

not

eg

Explanation for Incorrect Answer B :

Choice (B) is not correct. If

nR

However,

Explanation for Incorrect Answer C :

Choice (C) is not correct. If

not

is t

then

could be the value of

However,

then

which is

undefined. However,

is defined and equal to

Explanation for Incorrect Answer D :

Choice (D) is not correct. If

then

However,

not

10

and each term after the first is The first term of a sequence is previous term. Which of the following is an expression for the

more than the term of the

sequence for any positive integer

(A)
(B)
(C)

file://E:\\e9.htm

2006-11-12

The Official SAT Online Course

9/14

(D)
(E)

ANSWERS

AND EXPLANATIONS
term is

Explanation for Correct Answer B : Choice (B) is correct. From the given description of the sequence, the

the

term is

the

term is

and for any positive integer

the

term is

This last formula can

also be written as

Another way to see this is to look at the terms of the sequence. The first five terms Only the formula in choice (B) yields these terms for and are

and

Explanation for Incorrect Answer A : Choice (A) is not correct. If for any positive integer

the

term of the

sequence were equal to

then the first term of the sequence would be

But the first term of the sequence is

Explanation for Incorrect Answer C : Choice (C) is not correct. If for any positive integer

ed
the

sequence were equal to

then the first term of the sequence would be

Explanation for Incorrect Answer D : Choice (D) is not correct. If for any positive integer

te r

and

But the first term of the sequence is


the

term of the

term of the

sequence were equal to

Explanation for Incorrect Answer E : Choice (E) is not correct. If for any positive integer

eg is

But the first term of the sequence is

then the first term of the sequence would be

the

term of the

sequence were equal to

then the first term of the sequence would be

But the first term of the sequence is

11

nR

at random from shaded regions?


(A)
(B)
(C)

In the figure above,

are all squares. If a point is chosen

what is the probability that the point will be from one of the

file://E:\\e9.htm

2006-11-12

The Official SAT Online Course

10/14

(D)
(E)

ANSWERS

AND EXPLANATIONS

Explanation for Correct Answer D : Choice (D) is correct. The probability that a point chosen at random from will be from one of the shaded regions is equal to the ratio of the total area of the is equal to The area of shaded regions to the area of

and the area of the shaded region

is

Since

is a square, it follows that

Since

is also a

square, it follows that

so

Thus, the area of

is

equal to

Similarly, the area of the second shaded region,

is

equal to

Therefore, the probability that the chosen point

will be from one of the shaded regions is

from the shaded region

Explanation for Incorrect Answer B :

Choice (B) is not correct. This choice is equal to

Explanation for Incorrect Answer C : Choice (C) is not correct. This is the ratio of

eg

nR

the probability that a point chosen at random from shaded regions.

is
gives

probability that a point chosen at random from shaded regions.

te

re

to

Explanation for Incorrect Answer E :

Choice (E) is not correct.

If

will be from one of the unshaded regions, but the question asks for the probability that the point will be from one of the shaded regions.

is the probability that a point chosen at random from

12

which of the following must be true?

(A)
(B)
(C)
(D)
(E)

ANSWERS

AND EXPLANATIONS
gives

Explanation for Correct Answer B :

Choice (B) is correct. Expanding the left-hand side of

Explanation for Incorrect Answer A : Choice (A) is not correct. This is the probability that the point chosen at random but the point could also be will be from the shaded region from

but that is not equal to the

will be from one of the

but that is not equal to

will be from one of the

Subtracting

from both sides yields

now

multiplying both sides by

file://E:\\e9.htm

2006-11-12

The Official SAT Online Course

11/14

Explanation for Incorrect Answer A : must be Choice (A) is not correct. The value of

not

Explanation for Incorrect Answer C :

Choice (C) is not correct. The value of equation holds given any value for

does not have to be

if

the

Explanation for Incorrect Answer D : Choice (D) is not correct. It is the value of

that must be

not the value of

Explanation for Incorrect Answer E : Choice (E) is not correct. It is the value of

that must be

not the value of

13

If

what is the value of

(A)
(B)
(C)
(D)
(E)

It cannot be determined from the information given.

Explanation for Correct Answer A :

st er

not

ANSWERS

AND EXPLANATIONS

Choice (A) is correct. Since

Therefore,

ed

then
it follows that


Thus,

Explanation for Incorrect Answer B :

eg i

But

nR

Choice (B) is not correct. If

equals

were equal to

would be

equal to

and so

Explanation for Incorrect Answer C :

Choice (C) is not correct. The value of

the constant terms

is not equal to the sum of

and

Explanation for Incorrect Answer D :

Choice (D) is not correct.

implies that

Therefore, each factor

in the product

is positive, and so the product cannot be equal to

Explanation for Incorrect Answer E : can be determined from the condition Choice (E) is not correct. The value of

Therefore, the value of the product

can also be

determined.

14

What is the ratio of the diameter

of a circle to half the circumference of the circle?

(A)
(B)
(C)
(D)

file://E:\\e9.htm

2006-11-12

The Official SAT Online Course

12/14

(E)

ANSWERS

AND EXPLANATIONS
has circumference

Explanation for Correct Answer C : Choice (C) is correct. A circle with diameter

Thus, half

the circumference of the circle is

Therefore, the ratio of the diameter to half

the circumference is

which is equal to

or

Explanation for Incorrect Answer A : is the ratio of the radius of the circle to the Choice (A) is not correct. circumference of the circle.

Explanation for Incorrect Answer B : is the ratio of the diameter of the circle to the entire Choice (B) is not correct. circumference, but the question asks for the ratio of the diameter to half the circumference.

15

nR
(B)
(C)

(A)

(D)

(E)

ANSWERS

Explanation for Correct Answer B : Choice (B) is correct. Each carton contains

eg

plates. If the boxes of paper plates, and each box contains A carton contains dollars, what is the cost per paper plate, in dollars, when the plates carton costs are bought by the carton?

AND EXPLANATIONS
boxes of paper plates, and each box

is

contains

plates, so each carton contains

te r

Explanation for Incorrect Answer E : is the ratio of the circumference of the circle to the Choice (E) is not correct. radius of the circle.

dollars and contains

plates, the cost per plate is

Explanation for Incorrect Answer A : Choice (A) is not correct. Each carton contains

box contains

plates, so each carton contains

dollars and contains

plates, the cost per plate is

dollars.

ed

Explanation for Incorrect Answer D : is the ratio of the circumference of the circle to the Choice (D) is not correct. diameter of the circle.

plates. Since each carton costs

dollars.

boxes of paper plates, and each

plates. Since each carton costs

dollars, not

file://E:\\e9.htm

2006-11-12

The Official SAT Online Course

13/14

Explanation for Incorrect Answer C :

Choice (C) is not correct. This choice,

is the number of plates per dollar, not

the number of dollars per plate.

Explanation for Incorrect Answer D : Choice (D) is not correct. Each carton contains

boxes of paper plates, and each

box contains

plates, so each carton contains

plates. Since each carton costs

dollars and contains

plates, the cost per plate is

dollars, not

Explanation for Incorrect Answer E : Choice (E) is not correct. Each carton contains

boxes of paper plates, and each

box contains

plates, so each carton contains

plates. Since each carton costs

dollars and contains

plates, the cost per plate is

dollars, not

16

to

nR
(B)

(A)

(C)

(D)

(E)

ANSWERS

Explanation for Correct Answer B :

eg

to

Triangles

and

in the figure above are equilateral. What is the ratio of

to

to

to

to

AND EXPLANATIONS
and

is
Let

te
and

re

are all

d
Choice (B) is correct. Since

it follows that

is the

perpendicular bisector of

the point where

and

intersect, as

shown in the figure below.

Then

triangles. Thus,

file://E:\\e9.htm

2006-11-12

The Official SAT Online Course

14/14

the ratio of

to

is

to

Since

it follows that

and

Therefore,

Explanation for Incorrect Answer A :

Choice (A) is not correct. Let

the point where

and

intersect. If the

ratio of

to

were

to

then the ratio of

to

would also be

to

But then, by the Pythagorean Theorem, the ratio of

to

would

be

to

and so the ratio of

to

would be

to

But it is given

that

is equilateral, so this cannot be true.

Explanation for Incorrect Answer C :

Choice (C) is not correct. Let

the point where

and

intersect. Then

is a

triangle with

having measure

and

having measure

Thus,

so

Therefore, the ratio of

to

cannot be

to

Explanation for Incorrect Answer D :

Choice (D) is not correct. Let

the point where

Explanation for Incorrect Answer E :

ed

and

of

to

is

to

but the ratio of

to

Choice (E) is not correct. Let

the point where


and

intersect. The ratio

is

to

intersect. If the

is te r

to

ratio of

to

were

to

then the ratio of

to

would also be

to

But then, by the Pythagorean Theorem, the ratio of

to

would

be

to

and so the ratio of

would be

to

But it is

given that

is equilateral, so this cannot be true.

nR

eg

Back to Score Report

Copyright 2006 The College Board. All rights reserved.

Privacy Policy

Terms of Use

Contact Us

file://E:\\e9.htm

2006-11-12

The Official SAT Online Course

1/11

Help | Profile | My Organizer | My Bookmarks | Logout

Answers and Explanations

Test Sections

Back to Score Report

Section 1

View Answers and Explanations


Online - Practice Test #5

Section 2

Section 3

Section 4

Section 5

The first 10,000 United States patents, they were known as the X-patents, were burned in a fire in 1836. (A) they were known
(B)

Section 6

Section 8

which they knew

Section 9

(C) which they know

Section 10

(D) to be known

(E)

known

nR

Explanation for Incorrect Answer A : Choice (A) involves a comma splice. This sentence contains two independent clauses joined only by a comma: The first 10,000 United States patents were burned in a fire in 1836 and they were known as the X-patents. However, adding a coordinating conjunction after the first comma or replacing the first comma with a semicolon will not correct the problem because one independent clause comes between the subject (patents) and verb (were burned) of the other independent clause. The error can be corrected by turning the interrupting clause into a subordinate participial phrase, known as the Xpatents, that modifies the subject (patents) of the other independent clause.

Explanation for Incorrect Answer B : Choice (B) creates a pronoun error. This revision corrects the comma-splice error of the original by turning one independent clause (they were known as the Xpatents) into a dependent clause introduced by which, but the revision creates a pronoun error. The pronoun they has no antecedent to which it can logically refer.

Explanation for Incorrect Answer C : Choice (C) creates a pronoun error. This revision corrects the comma-splice error of the original by turning one independent clause (they were known as the Xpatents) into a dependent clause introduced by which, but the revision creates a pronoun error. The pronoun they has no antecedent to which it can logically refer.

Explanation for Incorrect Answer D : Choice (D) involves faulty logic. Although this revision corrects the comma-splice error of the original by turning one independent clause (they were known as the X-patents) into a subordinate infinitive phrase (to be known), it also creates faulty logic. The infinitive phrase to be known illogically suggests that the action will occur at some point in the future rather than now, in the present.

Winston knew that if he practiced often enough he would one day be able to play the piano as well as his brothers playing.

his brothers playing

eg

is

te

Explanation for Correct Answer E : Choice (E) is correct. It avoids the comma splice of the original by replacing the independent clause they were known as the X-patents with the participial phrase (known as the X-patents) that describes the subject of the independent clause, patents.

re

ANSWERS

AND EXPLANATIONS

file://E:\\e10.htm

2006-11-12

The Official SAT Online Course

2/11

(A)
(B)

that of his brother

(C) his brothers

(D) his brother could

(E)

what his brother did

ANSWERS

AND EXPLANATIONS

Explanation for Correct Answer D : Choice (D) is correct. It avoids the illogical comparison and mixed construction of the original by replacing the gerund phrase his brothers playing with the clause his brother could. This change also untangles the mixed construction of the phrase as well as by turning as well into an adverb modifying to play and as his brother could play into an adverb clause modifying the adverb well.

Explanation for Incorrect Answer E : Choice (E) results in faulty logic. With the phrase what his brother did serving as the object of the preposition as, the construction illogically suggests that Winston could play the piano as well as something that his brother did could play.

nR

(A)

In both his longer and his shorter works of fiction, Gabriel Garca Mrquez achieves the rare feat to be accessible to the common reader while satisfying the most demanding of sophisticated critics.

to be

(B)

for being

(C) of being

(D) that he is

(E)

that they are

ANSWERS

Explanation for Correct Answer C : Choice (C) is correct. It avoids the improper phrasing of the original by replacing the infinitive phrase to be with an idiomatic participial phrase (of being) to modify the noun feat.

Explanation for Incorrect Answer A : Choice (A) involves improper phrasing. The preposition of being, instead of the infinitive to be, is the idiomatic construction to modify the noun feat.

Explanation for Incorrect Answer B :

eg

AND EXPLANATIONS

is t

Explanation for Incorrect Answer C : Choice (C) results in faulty logic. With the phrase his brothers serving as the object of the preposition as, the construction illogically suggests that Winston could play the piano as well as his brothers piano could play.

er

Explanation for Incorrect Answer B : Choice (B) results in faulty logic. The pronoun that refers to the noun piano, so this construction illogically suggests that Winston could play the piano as well as his brothers piano could play.

ed

Explanation for Incorrect Answer A : Choice (A) involves faulty logic. The sentence is meant to compare the way that Winston would one day be able to play the piano with the way that his brother already could play, but instead it illogically suggests that Winston could play the piano as well as his brothers playing could.

file://E:\\e10.htm

2006-11-12

The Official SAT Online Course

3/11

Choice (B) involves improper phrasing. The preposition of, instead of the preposition for, is the idiomatic preposition to follow feat.

Explanation for Incorrect Answer D : Choice (D) involves an awkward construction. Although a dependent clause introduced by the relative pronoun that can modify a noun, the construction is not idiomatic in this sentence. The prepositional phrase of being is the idiomatic construction to modify the noun feat.

Explanation for Incorrect Answer E : Choice (E) involves an awkward construction and a pronoun error. Although a dependent clause introduced by the relative pronoun that can modify a noun, the construction is not idiomatic in this sentence. The prepositional phrase of being is the idiomatic construction to modify the noun feat. Also, in the context of this sentence, the pronoun that serves as the subject of the dependent clause (they) should refer to the singular subject of the main clause, Marquez, not to the object of the preposition, the plural noun works.

In 1977, Jann Wenner, the founder of Rolling Stone, moved the magazines offices from San Francisco to New York City, there he began developing a slicker, more commercial style of magazine and began cultivating relationships with major advertisers.
(A)

there

(B)

then

(E)

which is when

ANSWERS

Explanation for Correct Answer C : Choice (C) is correct. It avoids the comma-splice error of the original by using the relative pronoun where to introduce a subordinate clause that describes what happened when the magazine moved its offices to New York City.

nR

Explanation for Incorrect Answer A : Choice (A) creates a comma splice. The sentence presents two complete ideas, but the ideas are joined using only a comma. Two independent clauses should never be joined by only a comma. This problem can be fixed by using the relative pronoun where to introduce a dependent clause that describes what happened when the magazine moved its offices to New York City.

Explanation for Incorrect Answer B : Choice (B) creates a comma splice. The sentence presents two complete ideas, but the ideas are joined using only a comma. Two independent clauses should never be joined by only a comma. This problem can be fixed by using the relative pronoun where to introduce a dependent clause that describes what happened when the magazine moved its offices to New York City.

Explanation for Incorrect Answer D : Choice (D) creates a comma splice. The sentence presents two complete ideas, but the ideas are joined using only a comma. Two independent clauses should never be joined by only a comma. This problem can be fixed by using the relative pronoun where to introduce a dependent clause that describes what happened when the magazine moved its offices to New York City.

Explanation for Incorrect Answer E : Choice (E) involves awkward phrasing and ambiguous pronoun reference. While the pronoun which is used correctly to introduce a dependent clause, the first part of the sentence (In 1977, Jann Wenner, the founder of Rolling Stone, moved the magazines offices from San Francisco to New York City) would have to be recast so that the pronoun when would unambiguously refer to a time (1977) rather than to a place (New York City).

eg is te

AND EXPLANATIONS

re

(D) so

(C) where

file://E:\\e10.htm

2006-11-12

The Official SAT Online Course

4/11

Covering about 120 square miles, the New York City borough of Queens is almost as large as if you combine Manhattan, the Bronx, and Staten Island. (A) if you combine Manhattan, the Bronx, and Staten Island
(B)

when Manhattan, the Bronx, and Staten Island are combined

(C) Manhattan, the Bronx, and Staten Island combined

(D) if Manhattan were to combine with the Bronx and Staten Island

(E)

combining Manhattan, the Bronx, and Staten Island

ANSWERS

AND EXPLANATIONS

Explanation for Correct Answer C : Choice (C) is correct. It avoids the awkward phrasing and unnecessary shift in person of the original by replacing the subordinate clause introduced by if with a noun phrase, Manhattan, the Bronx, and Staten Island combined, which makes the comparison logical by comparing a noun (Queens) with other nouns. This revision also avoids the unnecessary shift to the second person by deleting you.

Explanation for Incorrect Answer B : Choice (B) involves awkward phrasing. In the comparative construction as as, the first as is an adverb that modifies the adjective (large) that follows it, and the second as is a preposition, which requires an object. An object of a preposition must be a noun. The clause when combined acts as an adverb, not a noun, so it cannot serve as the object of a preposition.

nR

(A)

Explanation for Incorrect Answer D : Choice (D) involves awkward phrasing. In the comparative construction as as, the first as is an adverb that modifies the adjective (large) that follows it, and the second as is a preposition, which requires an object. An object of a preposition must be a noun. The clause if Island acts as an adverb, not a noun, so it cannot serve as the object of a preposition.

Explanation for Incorrect Answer E : Choice (E) makes an illogical comparison. In the comparative construction as as, the first as is an adverb that modifies the adjective (large) that follows it, and the second as is a preposition, which requires an object. An object of a preposition must be a noun. Although the gerund combining is a noun, it results in an illogical comparison: Queens, a borough, is compared to combining, not to the other boroughs.

While cotton is still one of Georgias chief cash crops, the number of acres devoted to its cultivation grows smaller every year.

devoted to its cultivation grows

(B)

it devotes to the cultivation of it grow

(C) they devoted to its cultivation can grow

(D) devoted to its cultivation growing

(E)

they devoted to the cultivation of it are growing

ANSWERS

Explanation for Correct Answer A : Choice (A) is correct. It avoids the errors of the other options by using concise phrasing (its cultivation) in place of awkward wording, by removing

eg is te r

AND EXPLANATIONS

Explanation for Incorrect Answer A : Choice (A) involves awkward phrasing and an unnecessary shift in person. In the comparative construction as as, the first as is an adverb that modifies the adjective (large) that follows it, and the second as is a preposition, which requires an object. An object of a preposition must be a noun. The clause if Island acts as an adverb, not a noun, so it cannot serve as the object of a preposition. In addition, the use of the second-person pronoun you is an unnecessary shift in person.

ed

file://E:\\e10.htm

2006-11-12

The Official SAT Online Course

5/11

unnecessary pronouns (it and they), and by providing a singular verb grows to agree with the singular subject the number.

Explanation for Incorrect Answer B : Choice (B) involves awkward phrasing and subject-verb disagreement. The awkward phrase it devotes to the cultivation of it can be reduced to devoted to its cultivation. The plural verb grow does not agree with the singular subject the number. The singular verb grows should be used instead.

Explanation for Incorrect Answer C : Choice (C) involves illogical pronoun use and an error in verb tense. There is nothing in the sentence to which the plural pronoun they can logically refer, and the use of the past tense (devoted) is not consistent with the use of the present tense elsewhere in the sentence (is).

Explanation for Incorrect Answer D : Choice (D) results in a sentence fragment. As is, the sentence contains two dependent clauses. To fix this problem, a main verb is needed (the number . . . is growing).

Unlike flying squirrels, which may leave their young in a tree cavity while foraging, the babies of flying lemurs are usually carried with them.
(A)

the babies of flying lemurs are usually carried

(B)

the flying lemurs babies are usually carried

(C) the flying lemur is usually carrying its babies

(D) flying lemurs usually carry their babies

(E)

flying lemurs babies are usually carried

nR
ANSWERS

Explanation for Correct Answer D : Choice (D) is correct. It avoids the illogical comparison of the original by replacing the subject of the main clause (babies) with flying lemurs so that flying squirrels and flying lemurs are compared.

Explanation for Incorrect Answer A : Choice (A) involves an illogical comparison. Because the subject of the independent clause is babies, adult flying squirrels are illogically compared to babies rather than to flying squirrels.

Explanation for Incorrect Answer B : Choice (B) involves an illogical comparison. Because the subject of the independent clause, babies, is modified by the singular possessive flying lemurs, adult flying squirrels are illogically compared to babies rather than to flying lemurs.

Explanation for Incorrect Answer C : Choice (C) creates an unparallel comparison and a verb-tense error. The singular subject of the independent clause, flying lemur, is compared to the plural noun flying squirrels. Also, the simple present tense (carries), not the present progressive tense (is carrying), should be used in statements of general truth such as this one.

Explanation for Incorrect Answer E : Choice (E) involves an illogical comparison. Because the subject of the independent clause, babies, is modified by the plural possessive flying lemurs, adult

eg

AND EXPLANATIONS

is te re

Explanation for Incorrect Answer E : Choice (E) involves illogical pronoun use, awkward phrasing, and subject-verb disagreement. There is nothing in the sentence to which the plural pronoun they can logically refer, the awkward phrase devoted to the cultivation of it can be reduced to devoted to its cultivation, and the plural verb are does not agree in number with the singular subject the number.

file://E:\\e10.htm

2006-11-12

The Official SAT Online Course

6/11

flying squirrels are illogically compared to babies rather than to flying lemurs.

Born Charles Hardin Holley, Buddy Holly recorded some of the most distinctive and influential songs in rock-and-roll music, which includes such classics as That'll Be the Day, Rave On, and Peggy Sue. (A) which includes
(B)

they include

(C) also including

(D) including

(E)

these include

ANSWERS

AND EXPLANATIONS

Explanation for Correct Answer D : Choice (D) is correct. Its avoids the verb-tense error of the original. The tense of the verb in the dependent clause should be consistent with the past tense of the main verb recorded. Replacing the dependent clause (which Peggy Sue) with a participial phrase (including Peggy Sue) to modify the noun songs fixes this inconsistency in verb tenses.

nR

(A)

Explanation for Incorrect Answer C : Choice (C) involves faulty logic and unclear modification. The adverb also used to introduce the participial phrase (also Peggy Sue) suggests that the songs named in the participial phrase are in addition to other songs already named, but no other songs have been named. Also, the noun that this participial phrase modifies is unclear. It could modify either songs or music in the main clause.

Explanation for Incorrect Answer E : Choice (E) creates a comma splice. The two independent clauses Born Charles Hardin Holley, Buddy Holly recorded some of the most distinctive and influential songs in rock-and-roll music and these include such classics as That'll Be the Day, Rave On, and Peggy Sue are joined only by a comma rather than by a semicolon or a comma and a coordinating conjunction.

Chipmunks do not technically hibernate, but they do remain in their dens or burrows during cold periods.

hibernate, but they do remain

(B)

hibernate, but remaining

(C) hibernate, but they remained

(D) hibernate, they remain

(E)

hibernate, remaining

ANSWERS

Explanation for Correct Answer A :

eg

Explanation for Incorrect Answer B : Choice (B) creates a comma splice. The two independent clauses Born Charles Hardin Holley, Buddy Holly recorded some of the most distinctive and influential songs in rock-and-roll music and they include such classics as That'll Be the Day, Rave On, and Peggy Sue are joined only by a comma rather than by a semicolon or a comma and a coordinating conjunction.

AND EXPLANATIONS

is te

Explanation for Incorrect Answer A : Choice (A) involves a verb-tense error. The tense of the verb in a dependent clause should be consistent with the tense of the verb of the main clause. The present tense of the verb includes is inconsistent with the past tense verb recorded.

re

d
file://E:\\e10.htm

2006-11-12

The Official SAT Online Course

7/11

Choice (A) is correct. The two independent clauses Chipmunks do not technically hibernate and they do remain in their dens or burrows during cold periods are correctly connected with a comma and coordinating conjunction (, but), and the conjunction but clearly expresses the contrast between the ideas of the two clauses. Also, the two clauses are parallel in structure, and their verbs are consistent in tense.

Explanation for Incorrect Answer B : Choice (B) involves a lack of parallelism. The two grammatical units connected by a coordinating conjunction (but) should be parallel, but in this construction, an independent clause (Chipmunks do not technically hibernate) is connected with a coordinating conjunction to a participial phrase (remaining in their dens or burrows during cold periods).

Explanation for Incorrect Answer C : Choice (C) involves an improper verb tense. The two independent clauses in this sentence (Chipmunks hibernate and they periods) are statements of general truth. Statements of general truth should be expressed in the present tense. The past-tense verb of the second clause, remained, should be the present-tense verb remain.

10

nR eg

(A)

Cryonics the technique used to store human bodies at extremely low temperatures with the hope of one day reviving them are being performed today, but their technology is still in its infancy.

are being performed today, but their technology

(B)

are being performed today, while the technology

(C) which is being performed today, using technology that

(D) is being performed today, but their technology

(E)

is being performed today, but the technology

ANSWERS

AND EXPLANATIONS

Explanation for Correct Answer E : Choice (E) is correct. It avoids the errors in subjectverb agreement and pronounantecedent agreement of the original. It replaces the plural verb are being performed with the singular verb is being performed to agree with the singular subject Cryonics. This revision also replaces the plural pronoun their, for which there is no logical antecedent.

Explanation for Incorrect Answer A : Choice (A) involves errors in subjectverb agreement and pronounantecedent agreement. Cryonics is a singular noun and requires the singular verb is being performed, not the plural verb are being performed. Also the pronoun their can not logically refer to anything in the sentence.

Explanation for Incorrect Answer B : Choice (B) involves subjectverb disagreement and imprecise word choice.

is

Explanation for Incorrect Answer E : Choice (E) creates faulty logic. Although this sentence is grammatically correct, it is not logical. The independent clause Chipmunks do not technically hibernate and the participial phrase remaining in their dens or burrows during cold periods' contradict other." This faulty logic can be corrected by connecting the two ideas with a contrasting conjunction, as in the original.

te r

ed

Explanation for Incorrect Answer D : Choice (D) creates a comma splice and involves faulty logic. The two independent clauses Chipmunks do not technically hibernate and they remain in their dens or burrows during cold periods are connected only by a comma rather than by a semicolon or a comma and a coordinating conjunction. Also, without the conjunction but to express the contrast between the ideas of the two clauses, the two clauses seem to contradict each other. The first clause states that chipmunks do not hibernate, and the second clause suggests that they do.

file://E:\\e10.htm

2006-11-12

The Official SAT Online Course

8/11

Cryonics is a singular noun and requires the singular verb is being performed, not the plural verb are being performed. Also, the use of the subordinate conjunction while to introduce the clause while the technology is still in its infancy does not clearly express the contrast between the idea expressed in the dependent clause and the idea of the main clause (Cryonics is being performed today).
Explanation for Incorrect Answer C : Choice (C) creates a sentence fragment. Although the subjectverb and pronoun antecedent agreement errors are corrected, this revision creates a sentence fragment. The use of which turns the verb phrase is being performed today into a subordinate clause, which cannot stand alone. Also the participle using turns all that follows into a subordinate phrase, which cannot stand alone. Therefore, the subject Cryonics has no main verb to complete the sentence.

Explanation for Incorrect Answer D : Choice (D) involves an error in pronounantecedent agreement. There is nothing in the sentence to which the plural pronoun their can logically refer.

11

In science fiction stories, robots are generally depicted as working in the service of humanity, often escaping the control of their human masters and doing them harm. (A) often escaping the control of their human masters and doing
(B)

often escaping being controlled by their human masters, which does

(C) but they often escape the control of their human masters and do

(E)

but often escaping their controlling human masters, which does

ANSWERS

AND EXPLANATIONS

nR

Explanation for Incorrect Answer A : Choice (A) inappropriately uses a participial phrase (often doing) that does not precisely express the contrast between the two ideas in the sentence (robots humanity and often harm).

Explanation for Incorrect Answer B : Choice (B) involves imprecise phrasing and faulty logic. The participial phrase often being controlled by their human masters does not clearly express the contrast between the idea of the main clause (robots humanity) and the idea of this subordinate clause. Also, the subordinate clause which does them harm modifies escaping, and the pronoun them refers to robots. This clause illogically suggests that the robots cause themselves harm by escaping human control, instead of suggesting that the robots harm their human masters when they escape human control.

Explanation for Incorrect Answer D : Choice (D) involves errors in verb tense and word choice and faulty logic. The past tense of the verb escaped is inconsistent with the present tense of the main verb are depicted. Also, the adjective controlling, used to modify human masters, illogically changes the meaning from humans in general to only those humans who control robots. In addition, the pronouns it and them in the independent clause and it does them harm illogically suggests that the robots cause themselves harm by escaping human control, instead of suggesting that the robots harm their human masters when they escape human control.

Explanation for Incorrect Answer E : Choice (E) involves improper word choice and faulty logic. The adjective controlling, used to modify human masters, illogically changes the meaning from humans in general to only those humans who control robots. In addition, the subordinate clause which does them harm modifies escaping, and the pronoun them refers to robots. This clause illogically suggests that

eg

Explanation for Correct Answer C : Choice (C) is correct. It avoids the errors of the other options by precisely expressing the contrast between the idea of the main clause (robots humanity) and the idea of the subordinate phrase (often harm).

is

te

re

(D) but they often escaped their controlling human masters, and it does

file://E:\\e10.htm

2006-11-12

The Official SAT Online Course

9/11

the robots cause themselves harm by escaping human control, instead of suggesting that the robots harm their human masters when they escape human control.

12

The first Indian woman to win the Booker Prize, Arundhati Roys debut novel, The God of Small Things, received the honor in 1997.
(A)

The first Indian woman to win the Booker Prize, Arundhati Roys debut novel, The God of Small Things, received the honor in 1997.

(B)

Arundhati Roy, the first Indian woman to win the Booker Prize for her debut novel The God of Small Things, receiving the honor in 1997.

(C) The first Indian woman to receive the Booker Prize, Arundhati Roy won the honor in 1997 for her debut novel, The God of Small Things.

The Booker Prize was given to Arundhati Roys debut novel The God of (D) Small Things, winning her the first such honor for an Indian woman in 1997.
(E)

Arundhati Roy, winning the Booker Prize in 1997 for her debut novel The God of Small Things, made her the first Indian woman to do so.

ANSWERS

AND EXPLANATIONS

13

nR

(A)

Explanation for Incorrect Answer D : Choice (D) involves faulty logic and faulty modification. The participial phrase winning her the first such honor modifies the subject (Booker Prize) of the main clause. It does not make sense to say that the prize won the prize. In addition, the prepositional phrases for an Indian woman and in 1977 combine to illogically suggest that Roy won the prize for an Indian woman in 1977, not for her own novel, The God of Small Things.

Explanation for Incorrect Answer E : Choice (E) creates faulty logic. In its simplest subjectverbdirect object pattern, this sentence states Arundhati Roy made her, which illogically suggests that the author made herself.

Pilots at the airline, angered at the prospect of seeing their pension plans replaced with less generous versions, vowed to use legal means to fight it.

it

(B)

them

(C) this

(D) such a move

(E)

that from happening

ANSWERS

Explanation for Correct Answer D : Choice (D) is correct. It avoids the pronoun error of the original by replacing the pronoun it, which has no clear antecedent, with the noun phrase such a

eg

Explanation for Incorrect Answer B : Choice (B) creates a sentence fragment. Neither the infinitive to be nor the participle receiving can serve as a main verb for the subject Arundhati Roy. A main verb is needed to complete the action of the sentence.

AND EXPLANATIONS

is t

Explanation for Incorrect Answer A : Choice (A) involves an error in apposition. In this sentence, the introductory phrase, The first Indian woman to receive the Booker Prize, illogically renames novel, the subject of the main clause, rather than the author, Arundhati Roy.

er

ed

Explanation for Correct Answer C : Choice (C) is correct. It avoids the apposition error of the original by replacing the subject of the main clause, novel, with the subject Arundhati Roy so that the introductory appositive, The first Indian woman to receive the Booker Prize, properly renames the author, not the novel.

file://E:\\e10.htm

2006-11-12

The Official SAT Online Course

10/11

move.

Explanation for Incorrect Answer A : Choice (A) involves a pronoun error. The pronoun it has no antecedent to which it clearly refers. Because the pronoun is singular, it can refer to either of the two singular nouns (prospect or airline).

Explanation for Incorrect Answer B : Choice (B) involves a pronoun error. The plural pronoun them has no antecedent to which it clearly refers. Because it is plural, them could refer to any of the plural nouns in the first part of the sentence: pilots, plans, or versions.

Explanation for Incorrect Answer C : Choice (C) involves a pronoun error. The pronoun this has no antecedent to which it clearly refers. Because the pronoun is singular, this can refer to either of the two singular nouns (prospect or airline).

14

This basic document is stating the liberties guaranteed to the English people, the Magna Carta, signed in 1215 by Englands King John, proclaims rights that have formed the foundation of the constitutions of every English-speaking nation.
(A)

This basic document is stating

(B)

This basic document states

(D) A basic document that states

(E)

A basic document, it stated

nR
ANSWERS

Explanation for Correct Answer D : Choice (D) is correct. It avoids the comma-splice error of the other options by turning the first independent clause, This basic document is stating the liberties into an appositive. An appositive is a subordinate noun phrase that renames a noun. In this revision, A basic document is the appositive that renames Magna Carta, and the dependent clause that states the liberties modifies a basic document.

Explanation for Incorrect Answer A : Choice (A) involves a comma splice. This long sentence contains two independent clauses (This basic document the English people and the Magna Carta, signed nation) joined by only a comma. The error can be corrected by turning the first independent clause into an appositive that renames the subject of the second independent clause, the Magna Carta. In addition, the progressive tense of the verb is stating should be changed to the simple present states to be consistent with the present tense of the independent clause (proclaims).

Explanation for Incorrect Answer B : Choice (B) involves a comma splice. This long sentence contains two independent clauses (This basic document the English people and the Magna Carta, signed nation) joined by only a comma. The error can be corrected by turning the first independent clause into an appositive that renames the subject of the second independent clause, the Magna Carta.

Explanation for Incorrect Answer C : Choice (C) involves a comma splice. This long sentence contains two independent

eg

(C) A basic document, it states

AND EXPLANATIONS

is

te re

Explanation for Incorrect Answer E : Choice (E) involves a pronoun error and awkward phrasing. The pronoun that has no antecedent to which it clearly refers. Because the pronoun is singular, that can refer to either of the two singular nouns (prospect or airline). In addition to its pronoun problem, the phrase fight that from happening is not idiomatic. The idiomatic phrase is keep that from happening, but this correction still does not correct the pronoun problem.

file://E:\\e10.htm

2006-11-12

The Official SAT Online Course


clauses (A basic document, it the English people and the Magna Carta, signed nation) joined by only a comma. The error can be corrected by turning the first independent clause into an appositive that renames the subject of the second independent clause, the Magna Carta.

11/11

Explanation for Incorrect Answer E : Choice (E) involves a comma splice. This long sentence contains two independent clauses (A basic document, it the English people and the Magna Carta, signed nation) joined by only a comma. The error can be corrected by turning the first independent clause into an appositive that renames the subject of the second independent clause, the Magna Carta.

Back to Score Report

Copyright 2006 The College Board. All rights reserved.

Privacy Policy

Terms of Use

Contact Us

nR

eg i

st er

file://E:\\e10.htm

ed

2006-11-12

Vous aimerez peut-être aussi